Download as pdf or txt
Download as pdf or txt
You are on page 1of 94

Microbiological diagnostics of rise in temperature.

Gr “+” cocci, which


staphylococcal infection formed grape like clusters, were revealed in
1. In surgical department arise a suspicion milk products; but it was not possible to
about hospital staphylococcal infection allocate an agent of infection from patient’s
presence, seat of disease is a medical organism. What method gives the most
personal. Which medium is necessary to probable result for definition an etiology of
inoculate with material from nasopharynx of disease?
department personal for defining a carrier of A. Biological test
pathogenic staphylococcus? B. Neutralization test on animals
A. Endo’s medium C. Precipitation test in test tube
B. Yolk-salt agar D. Precipitation test in gel
C. Meat-and-peptone broth E. PHAT with antibodies
D. Russell’s medium diagnosticum
E. Blood agar 6. 26 years old man, at third day after
2. Child with a diagnosis “staphylococcal appendectomy, has a hyperemia and
sepsis” was been hospitalized. On what of infiltration on a stitch course, pus is allocated
medium is necessary to inoculate a patient’s from a wound at pressing. Gr “+” cocci
blood for purpose of agent revealing? formed an abnormality colonies were
A. Ploskirev’s medium revealed in pus. What drug is necessary to use
B. Meat-and-peptone agar for a local specific therapy?
C. Sugar-and-peptone broth A. Antistaphylococcal plasma
D. Buchin’s medium B. Antibiotic to which the infection
E. Yolk-saline agar agent is sensitive
3. А staphylococcal infection is educed as C. Staphylococcal antiphagin
complication at 36 yeared man, who has a D. Fluid staphylococcal
chronic active hepatitis, S.aureus is very bacteriophage
sensitive to tetracycline, penicillin and E. Staphylococcal autovaccine
streptomycin. What drug is necessary to 7. Golden staphylococcus was revealed on
prescribe for treatment of this patient? mucous of reproductive organs of young
A. Streptomycin woman with 8 monthed pregnancy. What
B. Penicillin drug is most necessary to use for specific
C. Tetracycline prevention of possible postnatal
D. Complexly using of all drugs of complications?
this list A. Local using of antiseptic drugs
E. Any drug of this list B. Staphylococcal autovaccine
4. Some children, who have eat from the C. Staphylococcal anatoxin
milk-kitchen, has been revealed with illness, D. Staphylococcal bacteriophage
which characterized by a nausea, vomiting, E. Antibiotic to which the infection
rise in temperature. Gr “+” cocci, which agent is most sensitive
formed grape like clusters, were revealed in 8. Gr “+” cocci located in grumps, gived a
milk products; but it was not possible to positive plasmacoagulase test, fermented
allocate an agent of infection from patient’s mannitol in anaerobic conditions and had a
organism. The most probable cause of disease lecithinase activity were revealed at time of
is an accumulation in products: sets of stomatological instruments sterility
A. α-toxin control. What microorganism was revealed in
B. Toxic protein A this case?
C. Endotoxin A. S. pyogenes
D. Enterotoxin B. S. epidermidis
E. Erythrogenic toxin C. S. saprophyticus
5. Some children, who have eat from the D. S. aureus
milk-kitchen, has been revealed with illness, E. Corynebacterium xerosis
which characterized by a nausea, vomiting,

1
9. Enteritis-ill child’s excrements were C. Take a research of nutrition unit
emulsifying in physiological solution, and equipment
drop of emulsion was inoculated on elective D. Define a presence of antibodies at
media: 10% milk-saline or yolk-saline agar. ill children
What microorganisms are provided to E. Take allergic test
allocate? 14. The microbiological research in neonatal
A. Klebsiella department of maternity home was made in
B. Coliform bacillі view of suspecting on a hospital infection.
C. Streptococcі The golden staphylococcus was revealed at
D. Staphylococcі some children and on some comfort items.
E. Enterococcі What properties of allocated cultures give a
10. Microorganisms, located as a cluster of chance to characterize their origin from the
grapes were revealed in a pus from a furuncle. general source?
What organisms were revealed? A. Antibioticogram
A. Micrococci B. Chromogenesis
B. Diplococci C. Antigen structure
C. Staphylococci D. Biochemical activity
D. Streptococci E. Phage type
E. Tetracocci 15. The doctor detected Gr “+” cocci, what
11. There are causes of purulent postoperative locate in bunch-shaped clumps at time of
staphylococcal nature complications have microscopy of investigated material, taken
become frequent in surgical hospital. How it from patient with acute purulent periostitis.
is possible to define a source of What microorganisms have this
staphylococcal infection in hospital? morphological signs?
A. Defining of phage varies A. Staphylococcuses
B. Defining of haemotoxin B. Sarcines
C. Defining of aggression enzyme C. Tetracocci
D. Defining of biovares D. Candida
E. Defining of antibiotic sensitivity E. Streptococci
12. Food toxic infection eruption was 16. Gr “+” cocci which have a round form,
registered in a city which has arising after form S-form colonies with golden pigment on
taking of cream pastry. A pathogenic a dense nutrient medium, produce a
microorganism was revealed in the rests. lecithinase, plasmocoagualase, haemolysin
What microorganism is most possibly can and ferment a mannitol in anaerobic condition
cause this toxic infection? were revealed at patient with purulent
A. E. coli phlegmona of maxillofacial region. What kind
B. C. perfringens of microorganism is causes suppuration?
C. S. aureus A. S. aureus
D. S. typhimurium B. S. pyogenes
E. S. enteriditis C. S. mutans
13. There are almost all children had D. S. epidermidis
gastroenteritis signs in a few hours after E. S. sanguis
cheese curd intake in the kindergarten. At 17. Gr “+”round microorganisms which have
time of bacteriological examination of a lecitinase activity, coagulate rabbit’s plasma
vomiting masses and wastes of the cheese and split a mannitol in anaerobic condition
curd the golden staphylococcus was revealed. were revealed from purulent exudates of
How is expedient to continue a research for a patient with odontogenous phlegmona. What
specification of diagnosis? of next microorganisms can cause a purulent
A. Make a lysotyping of allocated complication?
strains A. S. epidermidis
B. Define an ability of strains to B. S. aureus
produce toxins C. S. pyogenes

2
D. S. viridans D. Lactic-acid bacteria
E. S. mutans E. Bifidumbacteria
18. The children disease characterized by 22. An agent, which gives round-form yellow
acute beginning, nausea, vomiting, and colonies on blood agar was isolated from
diarrhea has developed after cheese intake in patient with skin pustules. These colonies
kindergarten. Gr “+” microorganisms, which have a medium size and surrounded by active
located in bunch-shaped clumps were area. Gr “+” cocci located by ungeometrical
revealed at microscopy of dab, made of clumps were revealed in dabs from colonies.
cheese and vomiting masses. What will be This culture is oxydase-positive, catalase-
your following actions for etiology definition positive, ferment mannitol and synthesize
of this food intoxication eruption? plasmocoagualase. What pathogenic agent is
A. In addition to apply a revealed?
bacteriological method of A. Staphylococcus aureus
diagnostics B. Staphylococcus saprophyticus
B. To make the conclusion that it is C. Staphylococcus epidermidis
staphylococcus D. Staphylococcus agalactiae
C. In addition to take an allergic test E. Staphylococcus pyogenes
D. In addition to define antibodies in 23. Choose microorganism caused purulent
blood serum diseases of oral cavity mucous membrane:
E. In addition to define a phagothype A. Borrelia, Treponema
of staphylococcus B. Corynebacteria, Fusobacteria
19. The staphylococcus differs from a strain, C. Bacteroides, Enterobacteria
which defined at patients, but has an R- D. Candida and Aspergillus
plasmid was revealed from nasopharynx of E. Staphylococcus, Streptococcus
nurse at time of hospital infection eruption. 24. There is a frequency of infection-
What sing can take a hospital strain as a result inflammatory diseases caused by
of conjugation? Staphylococcus aureus was increased in
A. Antibiotic resistance maternity hospital. What of epidemiological
B. Enterotoxin producing method is using for detection of source of this
C. Fecundity infection?
D. Staphylokines synthesis A. Defining of phage vary
E. Formation of colonies B. Defining of serovary
20. S. aureus culture was revealed as a result C. Defining of colicinotype
of bacteriological examination of sour cream D. Defining of antibioticogramm
material. How to prove what the isolated E. Defining of biovary
colony of S. aureus is an agent of food 25. Gr “+” cocci located by clusters in
poisoning which arisen at sour cream preparation were isolated from patient with
consumers group? osteomyelitis of maxilla. They ferment a
A. Detection of enterotoxin mannitol to acid in anaerobic conditions,
B. Detection of plasmcoagualase coagualate a plasma of rabbit. It is necessary
activity to classify this pure culture.
C. Detection of haemotoxin A. Staphylococcus aureus
D. Detection of saccharolytic B. Staphylococcus saprophyticus
properties C. Streptococcus pyogenes
E. Detection of lecithinase activity D. Streptococcus mutans
21. The examination of bacteria carriers has E. Staphylococcus epidermidis
been made for estimation of sanitary state in 26. The sanitary-microbiological examination
maternity hospital. What microorganisms are of air in medical establishment must be done
pathogenic for human? every 4 months. Which of next
A. Pathogenic staphylococcus microorganisms is classifyed to sanitary-
B. Micrococcus representative air microorganisms of enclosed
C. Sarcines space?

3
A. C. perfrignens С. Streptococcus pyogenes
B. E. coli D. Staphylococcus epidermidis
C. E. faecalis E. Staphylococcus aureus
D. P. aureuginosa
E. S. aureus
27. It’s necessary to find a source of infection Microbiological diagnostics of
in surgical department of stomatologic clinic streptococcal infection
in view of postoperative purulent 31.Confinement to bed patient has a rise in
complications cases, caused by golden temperature of body, cough occurrence with a
staphylococcus. What examination is lot of sputum secretion, pain in the chest.
necessary to make among the personnel? Gr“+” cocci, located in pairs were revealed in
A. Examination of dysbacteriosis the sputum. What microorganism is the most
B. Test of staphylococcal immunity probable agent of infection?
intensity A. Legionella pneumophilla
C. Inoculation of blood to a sterility B. Staphylococcus aureus
D. Detection of carriage C. Klebsiellа pneumonia
E. Examination of outwash of hands D. Mycoplasma pneumonia
28. Examination of a patient with pustular E. Streptococcus pneumoniае
skin lesions allowed to isolate a causative 32.The infection of gallbladder was caused by
agent that forms in the blood agar roundish organisms, what have an oval form, is located
yellow middle-sized colonies surrounded by in pairs or short chains. Proceeding from
haemolysis zone. Smears from the colonies results of serological tests, these organisms
contain irregular-shaped clusters of gram- were identified as “D”group streptococci.
positive cocci. The culture is oxidase- and How they are called?
catalase-positive, ferments mannitol and A. Enterococci
synthesizes plasmocoagulase. What causative B. Green streptococci
agent was isolated? C. Aerococci
А. Staphylococcus saprophyticus D. β-hemolytic streptococci
В. Streptococcus agalactiae E. Pneumococci
С. Streptococcus pyogenes 33. Dick’s toxin was intravenous injected to
D. Staphylococcus epidermidis 20 children, who had contacts with scarlet
E. Staphylococcus aureus fever patient in the kindergarten. Three
29. In the surgical department of a hospital children have tumescence and reddening in a
there was an outbreak of hospital infection place of injection; other children haven’t it.
that showed itself in often postoperative Estimate the results of reaction.
wound abscesses. Bacteriological A. Children with positive Dick’s
examination of pus revealed aurococcus. reaction are the vectors of hemolytic
What examination shall be conducted to find streptococcus
out the source of this causative agent among B. Children with positive Dick’s
the department personnel? reaction have an allergy to toxin of hemolytic
A. Biochemical identification streptococcus
B. Microscopical examination C. Children with positive Dick’s
C. Serological identification reaction have carried a scarlet fever in past
D. Estimation of antibiotic D. Children with positive Dick’s
susceptibility reaction have no antibodies to toxin
E. Phagotyping of hemolytic streptococcus
30. A 65-year-old man has purulent abscess E. At children with positive Dick’s
on his neck. Analyses revealed a culture of reaction will occurrence symptoms
gram-positive cocci with plasmocoagulase of scarlet fever in few days
activity. This culture relates most likely to: 34. After carried scarlet fever of upper
А.Staphylococcus saprophyticus respiratory tracts, the child have a pain in
В. Streptococcus agalactiae joints during long time, which amplified in

4
autumn. What microorganism is the most C. Micrococcus
probable agent of infection? D. Diplococcus
A. S. aureus E. Tetracoccus
B. S. pyogenes 39. A rheumatic heart’s disease has developed
C. C. diphtheriae at 12 years old boy after carried angina. Every
D. B. pertussis next streptococcal infection caused a
E. Flu virus deterioration of patient’s condition. What
35. Doctor has suspected a scarlet fever at 2 drug is rationality to use for complications
yeared child with catarrhal appearance and prevention?
eruption on skin. A little dose of serum to A. Autovaccine
erythrogenic toxin of streptococcus was B. Streptococcal anatoxin
inoculated intradermal and eruption was C. Streptococcal bacteriophage
disappeared in the injection place. What D. Donor’s γ-globulin
means results of reaction? E. Penicillin
A. Final diagnosis is confirmed 40. Streptococcus of “A” group was isolated
B. Child has a high sensitivity to from the patient with glomerulonephritis.
erythrogenic toxin What of streptococcal antigens provides a
C. Disease is not caused by typical specificity and virulence?
hemolytic streptococcus A. Vi-antigen
D. The serum must be enjected B. M-protein
intravenously C. T-antigen
E. Child’s immunity system is very D. P-antigen
poor E. OF-antigen
36. The culture of coccal bacteria was 41. The pathological material (a smear of
revealed from a nasopharynx of a boy with tonsils) was taken from 7 years old child who
chronic tonsillitis. They are located by chains has an angina, and inoculated on blood agar.
in the smear. What microorganisms it is can The medium became transparent around
be? colonies of streptococcus in next day. The
A. Vibrio presence what of pathology factor of an
B. Staphylococcus infectious agent was revealed?
C. Escherichia A. Hemolysin
D. Clostridia B. Endotoxin
E. Streptococcus C. Neuraminidase
37. Blood from a patient with suspicions of a D. β-lactamase
sepsis was inoculated on a sugar broth. The E. Leukocidin
long chain-located Gr“+” cocci was detecting 42. Gr “+”oval located in pairs
in the smear, what taken from sediment. The microorganisms were revealed at microscopic
small, transparent, round colonies with active diagnostics of patient’s sputum. What
hemolytic area were grown on a blood agar. microorganisms were revealed in sputum
What organisms is presence in patient’s more probably?
blood? A. Streptococcus pneumoniae
A. Streptococcus B. Neisseria meningitidis
B. Micrococcus C. Klebsiella pneumoniae
C. Staphylococcus D. Staphylococcus aureus
D. Tetracoccus E. Neisseria gonorrhoeae
E. Sarcina 43. The physician has suspected a progress of
38. The short chain-located, round disease at 8 years old boy, who often have an
microorganisms were revealed in a smear of angina caused by Streptococcus. What of
tonsils mucous. What microorganisms were microbiological method is need to use for
revealed? diagnosis confirmation?
A. Staphylococcus A. Serological
B. Steptococcus B. Bacterioscopical

5
C. Bacteriological A. Microscopical
D. Allergic B. Bacteriological
E. Biological C. Serological
44. At examination of 2 years old boy, D. Express-method
physician has see an eruption on neck and E. Biological
upper part of chest, and also tongue erythema 49. Gr “+” cocci, which can produce a
(“crimson tongue”) and give a diagnosis capsule were revealed from sputum of patient
“scarlet fever”. What organism is an infection with suspicion on lung fever. The
agent of this disease? microorganisms fermented an inulin and had
A. Streptococcus pyogenes been lysed by bile at identification of pure
B. Escherichia coli culture. What organisms were isolated from
C. Neisseria gonorrhoeae this patient?
D. Mycobacteria tuberculosis A. Staphylococcus aureus
E. Corynebacterium diphtheriae B. Streptococcus haemolyticus
45. Gr “+” cocci, which have a little oblong C. Streptococcus pneumoniae
form, located in pairs or short chains, produce D. Streptococcus viridans
a capsule, cause an α-hemolysis in blood agar E. Streptococcus pyogenes
were revealed from oral cavity of clinical- 50. Blood of a patient with presumable sepsis
healthy 25 years old man. A vector what of was inoculated into sugar broth. There
disease he is? appeared bottom sediment. Repeated
A. Streptococcus salivarius inoculation into blood agar caused growth of
B. Streptococcus pyogenes small transparent round colonies surrounded
C. Streptococcus pneumoniae by hemolysis zone. Examination of a smear
D. Streptococcus feacalis from the sediment revealed gram-positive
E. Peptostreptococcus cocci in form of long chains. What
46. Gr “+”prolonged diplococci with pointed microorganisms are present in blood of this
opposite ends were revealed in sputum of patient?
patient with suspicion on pneumonia. What A. Streptococci
microorganisms were revealed in smear? B. Micrococci
A. Staphylococcus aureus C. Staphylococci
B. Streptococcus pneumoniae D. Tetracocci
C. Klebsiella pneumoniae E. Sarcina
D. Neisseria meningitidis 51. A 7 year old child often suffers from
E. Neisseria gonorrheae streprococcic angina. Doctor suspected
47. At microscopy of sputum from patient development of rheumatism and administered
with pneumonia, a lot of Gr “+” lancet shaped serological examination. The provisional
diplococci, which are surrounded by capsule, diagnosis will be most probably confirmed by
were revealed. Which microorganisms were presence of antibodies to the following
revealed in smear? streptococcic antigen:
A. Klebsiella pneumoniae A. O-streptolysin
B. Streptococcus pneumoniae B. C-carbohydrate
C. Chlamidia pneumoniae C. M-protein
D. Staphylococcus aureus D. Erythrogenic toxin
E. Escherichia coli E. Capsular polysaccharide
48. In lumps of pus, taken from sputum of 52. During examination of a patient a dentist
patient with clinical diagnosis “lung fever” revealed a lot of "white spots" - zones of
the blue-violet lancet shaped diplococci, enamel demineralization. What
which surrounded by capsule, were revealed. microorganisms take part in the development
The bacteriologist`s conclusion was “S. of this process?
pneumonia is in the sputum”. What method A. Streptococcus mutans
was used for confirmation of clinical B. Streptococcus salivarius
diagnosis? C. Streptococcus pyogenes

6
D. Veilonella parvula 57. Two children from kindergarten are in
E. Staphylococcus epidermidis hospital with a diagnosis - meningococcal
53. There was an episode of the hospital meningitis. It is necessary to conduct
infection caused by S.pyogenes in the surgical examination of other children and personnel
department. What media can be used for of the kindergarten for the revealing of the
pathogenic Streptococci isolation? effaced forms of meningococcal disease and
A. Endo`s media meningococcal carriers. What kind of
B. Yolk-salt agar diagnostic should be used?
C. MPB A. Bacterioscopy of cerebrospinal
D. Ressel`s media fluid
E. Bloody agar B. Revealing of meningococcal
54. The child with a diagnosis “Streptococcal antigen in cerebrospinal fluid
sepsis” was hospitalized. What media can be C. Serum research
used for pathogenic Streptococci isolation D. Cultural research of
from blood? nasopharyngeal mucus
A. Yolk-salt agar E. Skin allergic test
B. Endo`s media 58. The 5-years old child recovers after acute
C. MPB meningococcal meningitis. When will it be
D. Bloody agar able to visit kindergarten again?
E. Saccharine broth A. After complete clinical
55. A doctor has suspected scarlet fever of convalescence
a 5-year-old child with catarrh and skin B. When serum reactions will become
rash. A small quantity of serum against negative
streptococci erythrogenic toxin has been C. If all children in a group will
injected intracutaneously. In the place of receive a vaccination against
injection the rash disappeared. What do meningitis
reaction results indicate? D. If meningococci will not be
A. The whole doze of serum can revealed in cerebrospinal fluid
be injected intravenously. E. If the result of bacteriological
B. The child is hypersensitive to examination of nasopharyngeal mucus
erythrogenic toxin. is negative
C. The disease has been caused by 59. There was examination of children and
a nonhemolytic streptococcus. personnel with the purpose of revealing of
D. The clinical diagnosis has meningococcal carriers in a kindergarten.
proved to be true. Choose the method of microbiological
E. Immune system of the child research:
is considerably impaired. A. Bacteriological
56. There are cases of purulent postoperative B. Allergological
complications of the streptococcal nature C. Bacterioscopical
which have become frequent in a surgical D. Biological
hospital. How to determine a source of a E. Serological
streptococcal infection in a hospital? 60. A bacteriologist at research of
A. Determination of phagovaries nasopharyngeal mucus adhered to the set rule
B. Determination of serovaries of maintainance of aetiological agent in
C. Determination of enzymes material. At bacterioscopical research the
D. Determination of biovaries presence of gram-negative cocci which
E. Determination of sensitiveness to remind coffee grains and located by pairs and
antibiotics tetrads was determined. Name an aetiological
agent which was revealed by bacteriologist.
Microbiological diagnostics of A. Stаphyloсoссus аurеus
meningococcal infection B. Neisseria menigitidis
C. Neisseria gonorrhoeae

7
D. Morаxеllа lасunаtа D. Microscopical
E. Асinеtobасtеr саlсoасеtiсus E. Bacteriological
61. Gr- diplococci, having the same form as 65. The pure culture of gram-negative
coffee grains were revealed during diplococci was isolated from the child, which
bacterioscopical research of nasopharyngeal visited kindergarten where a case of
mucus of 2,5-years old child with meningococcal infection took place. Which
nasopharyngitis. What organs of child most facts do testify that this microorganism is
probably will be affected if these Neisseria menigitidis?
microorganisms will get to blood? A. Does not grow at 300C on the
A. Meninges serum agar
B. Mitral valves B. Grows at 370C on the serum agar
C. Renal glomerules C. Creates pigment
D. Urogenital tract D. Grows on MPA
E. Lymphatic nodes E. Ferments saccharose
62. From a child with cerebrospinal 66. The sick child`s diagnosis was “epidemic
meningitis a turbid cerebrospinal fluid which cerebrospinal meningitis” on the basis of the
contains plenty of leucocytes is received. conducted inspection and using of
What serum reaction does it cost to take microbiological examination. What
advantage of for express-diagnostic of microorganism caused this disease?
disease? A. Neisseria gonorroeae
A. Precipitation reaction B. Stаphyloсoссus аurеus
B. Agglutination reaction C. Strеptoсoссus pyogenes
C. Complement binding assay D. Neisseria menigitidis
D. Hemagglutination reaction E. Psеudomonas aeruginosa
E. Neutralization reaction 67. Gram-negative diplococci located in
63. A 5-years old patient complains of great leucocytes were revealed in the smear made
headache and vomiting. Objectively: rigidity from cerebrospinal liquid of patient with
of the occipital muscles, continual vomiting, festering meningitis. What does provide the
nausea, herpetic rash on face, fever. What uncompleteness of Neisseria menigitidis
pathological material is needed to use for phagocytosis?
bacteriological confirmation of the A. Capsule polysaccharide antigens
preliminary diagnosis - cerebrospinal B. Capsule protein antigens
meningitis? C. Peptidoglycanes of the cell wall
A. Puncture of cerebrospinal fluid, D. Proteins of the cell wall
which flows out force-feed and has an E. Enzymes of pathogenicity
unpleasant smell 68. There is a child with diagnosis
B. Isolating of urinoculture of “meningococcal nasopharyngitis” in the
Neisseria menigitidis from excrements kindergarten. What vaccine can be used with
C. Isolating of Neisseria menigitidis the purpose of emergency specific
from sputum prophylaxis of meningococcal infection for
D. Research of vomiting masses contact children?
E. Isolating of Neisseria menigitidis A. Anatoxin
from the mucus membrane of the B. Alive attenuated
urinogenital system C. From killed microorganisms
64. A doctor diagnosed “meningococcal D. Chemical
nasopharyngitis” to the child with E. Combined
nasopharyngitis. What method of laboratory 69. The young woman `s temperature
diagnostics is the best for confirmation of suddenly rose to 39°C and great headache has
diagnosis? appeared. At a review rigidity of the occipital
A. Allergological muscles is determined. Spinal puncture was
B. Biological performed. There are huge amount of
C. Serological neutrophils and gram-negative diplococci

8
staned by Gram in a smear from a E. Trychomonadosis
cerebrospinal liquid. Which of the following 74. A doctor-ophthalmologist examined a sick
bacteria could be the reason of this illness? child with the phenomena of purulent cerato-
A. Stаphyloсoссus аurеus conjuctivitis and supposed that it is a
B. Neisseria menigitidis blenorrhea. What methods of laboratory
C. Hаеmophilus influеnzае diagnostics can be used to confirm the
D. Streptococcus pneumoniae diagnosis?
E. Psеudomonas aeruginosa A. Microscopic and bacteriological
70. The culture of meningococci was defined B. Microscopic and biological
in the sediment of cerebrospinal liquid of the C. Microscopic and phage diagnostics
patient with meningitis. What serum reaction D. Bacteriological and biological
must be used for determination of serum E. Phage diagnostics and allergic
group? 75. Mucous consistency, transparent, even-
A. Agglutination reaction edged protuberant colonies have grown on the
B. Precipitation reaction special media after the pus from an urethra
C. Indirect hemagglutination test inoculation. Gram-negative, bean-shaped
D. Complement binding assay diplococci were found during the microscopy
E. Immune sorbent assay of preparations from these colonies. What
71. The bacteriologist revealed small located kind of disease can they cause?
in pairs bean-shaped microorganisms in a A. Chlamidiosis
smear from nasopharyngeal mucus. They B. Gonorrhоea
became pink after Gram staining. Name these C. Syphilis
bacteria. D. Rabbit-fever
A. Streptococcus E. Melioidosis
B. Staphylococcus 76. A pus from a patient’s urethra with
C. Neisseria suspicion of gonorrhea, that independently
D. Enterococcus treated himself by peniсillin, was taken for
E. Micrococcus research. During the research the L-shaped
72. From a child with cerebrospinal gonococci were found. How does penicillin
meningitis there was obtained muddy spinal influence on gonococci?
fluid with numerous leukocytes. Which test A. Destroys the synthesis of cellular
should be used for express-diagnostics of the wall
disease? B. Destroys the synthesis of
a. Complement fixation test. albumens
B. Agglutination. C. Destroys the synthesis of amino
C. Immunofluorescence test. acid
D. Hemagglutination. D. Destroys the synthesis of
E. Neutralization. cytoplasmic membrane
E. Destroys adhesines
Microbiological diagnostics of gonococcal 77. During the bacteriological examination of
infection the patient with suspicion on a gonorrhoea
73. Bacteriological examination of purulent growth of microorganisms on special nutrient
discharges from the urethra revealed mediums was not revealed. What methods of
gram-negative bacteria looking like coffee diagnostics need to be used to confirm or
beans. They were localized in the leukocytes exclude the preliminary diagnosis?
and could decompose glucose and maltose to A. Precipitation test
acid. These are the causative agents of the B. Use the biological method
following disease: C. To conduct microscopic research
A. Syphilis D. Use IFT
B. Venereal lymphogranulomatosis E. Use CBT, allergic test
C. Gonorrhoea 78. 28 year-old woman, who several months
D. Chancroid ago suffered from an acute gonorrhea and

9
treated herself independently by penicillin, hemagglutination assay
has an inflammation of appendages of uterus. B. (R)CFT- Reiter's complement
The reaction Borde-Gangou is positive. What fixation test
other organs and systems can be affected, if C. RDHA - Reverse direct
the course of medical treatment is not hemagglutination assay
conducted? D. Immunoblot analysis
A. Joints E. IFA - Immunofluorescence assay
B. Liver 83. The patient has addressed to the doctor-
C. Vessels dermatologist. Gramnegative bean-shaped
D. Immune system diplococcі inside and outside of leucocytes
E. Central nervous system were detected on bacteriological examination
79. Gramnegative bean-shaped diplococcі of the purulent exudates from the cervix of
inside and outside of leucocytes were detected the uterus. What results of process are
on bacteriological examination of the purulent observed in a preparation?
exudates from the cervix of the uterus. Name A. Capsuleforming
the causative agent of purulent inflammation B. Metabolism
of the cervix of the uterus. C. Phagocytosis
A. Chlamidia trachomatis D. Sporeforming
B. Neisseria gonorroeae E. Malignisation
C. Haemophilus vaginalis 84. The sick woman has the clinical diagnosis
D. Trichomonas vaginalis "gonorrhoea". What from listed below
E.Calymmatobacterium granulomatis researches can be applied to diagnosis
80. During the bacteriological tests of the acknowledgement?
purulent secreta from urethra there were A. Microscopy of pathological
found bacteria, which according to Gram material
were negatively staining, looked like coffee B. Infection of laboratory animals
beans. These bacteria were splitting glucose C. Test with phages
and maltose to acid, they were located inside D. Hemagglutination assay
the leucocytes. Name these microorganisms. E. Reaction of immobilisation
A. Neisseria meningitides 85. A patient had delivered to the hospital
B. Neisseria gonorrhoeae with previous diagnosis “chronic gonorrhea”.
C. Staphylococcus aureus What type of the double-system serological
D. Streptococcus pyogenes reaction can be used for specific antibodies
E. Enterococcus faecalis revealing in the serum?
81. A doctor made the diagnosis of A. Agglutination test
gonorrhoea to the young man. It was known B. Neutralization test
from the anamnesis that a patient had had C. Complement’s binding test
gonorrhoea before and he had been treated D. Radio-immune analysis
completely. What type of infection can this E. Immunoenzyme analysis
new disease be attributed to? 86. Gramnegative bin-shaped diplococcus
A. Superinfection inside and outside of leucocytes were detected
B. Reinfection on bacteriological examination of the purulent
C. Secondary infection exudates from the cervix of the uterus. Name
D. Relapse the causative agent of purulent inflammation
E. Autoinfection of the cervix of the uterus.
82. A patient who came to the doctor because A.Calymmatobacterium
of his infertility was administered to make granulomatis
tests for toxoplasmosis and chronic B. Chlamidia trachomatis
gonorrhoea. Which reaction should be C. Haemophilus vaginalis
performed to reveal latent toxoplasmosis and D. Trichomonas vaginalis
chronic gonorrhoea in this patient? E. Neisseria gonorroeae
A. RIHA - Reverse indirect 87. Clinical diagnosis of a female patient was

10
gonorrhoea. What examination method can be raspberry color with metallic glitter have
applied for confirmation of this diagnosis? grown. Which of diagnostic serums the
a. Infection of laboratory animals positive reaction of agglutination of the
B. Microscopy of pathological isolated culture is the most possible?
material A. Polyvalent OK-serum
C. Test with bacteriophage B. Salmonellas O- serum
D. Hemagglutination reaction C. Salmonellas N- serum
E. Immobilization reaction D. The O-cholera serum
E. Serum for Yesinia, serotype 03
Microbiological diagnostics of 92. The child with coli-enteritis has arrived to
escherichiosis the infectious hospital. Colon bacilla was
88. E. coli culture of O-111 serotype was isolated from excrements. How to establish
revealed in the excrements of a sick child, the bacilla accessory to pathogenic variants?
who had been given artificial food. A. In reaction of agglutination with
Which diagnosis is correct? O-serum
A. Coli-enteritis B. On the basis of biochemical
B. Gastroenteritis properties
C. Cholera-like diseases C. By phage typing
D. Food poisoning D. Microscopy of the stained smears
E. Disentery-like diseases E. Due to growth on Endo media
89. During bacteria conjugation experiment 93. 55 yeared man was hospitalized to a
the next strains were used: 1- streptomycin surgical clinic with suspicion on a sepsis.
resistant auxotroph by lysine (F+) and 2 – What material must be taken from patient
streptomycin sensitive prototroph (F-). What for research and what media it should be
media should be taken for isolation of used?
recombinants? A. Blood, saccharine broth
A. With lysine and streptomycin B. Liquor, serum agar
B. With lysine without streptomycin C. Urine, meat peptone agar
C. Without lysine, but with D. Pus, yolk-salt agar
streptomycin Е. Lymphatic node puncture, cysteine
D. Without lysine and streptomycin agar
E. With factors F+ and F - 94.7 yeared boy has the cholerae-like disease
90. The causes of meningitis which were (vomiting, diarrhoea). The same crimson
caused by a colon bacillus were noted in colour colonies with metal shine have grown
the department for new-borns. At the on Endo media after excrements inoculation.
sanitary-bacteriological investigation of What microorganism is the most possible
rooms a colon bacillus was also revealed. agent of disease?
What research should be made for A.Yersinia enterocolitica
identification of strains? B. Salmonella enteritidis
A. Morphological properties studying C. Escherichia coli
B. Cultural properties studying D. Shigella sonnei
C. Biochemical properties studying E. Vibrio
D. Pathogenicity tests 95.On bacteriological examination of the
E. Colicinotyping defecation of a 4-months-old baby with the
91. 18 yeared patient has general weakness, symptoms of acute bowel infection there were
dizziness, nausea, vomiting, diarrhea ( revealed red colonies spread in the large
defecation up to a 10 time per a day). A quantity in the Endo medium. What
doctor made a previous diagnosis microorganism can it be?
"dysentery". However at the A. Escherichia
bacteriological research of vomiting B. Salmonella
masses and excrements Shigella was not C. Staphylococcus
found. On Endo’s media the colonies of D. Streptococcus

11
E. Shigella C Bacteriophages
96. The 4 yeared child has arrived to the D Autovaccines
infectious clinic with the preliminary E Eubiotics
diagnosis “coli-enteritis”. Symptoms of the 101. Among junior children of an orphanage
general intoxication, vomiting and diarrhoea an outbreak of intestinal infection with signs
were observed. At bacteriological research of of colienteritis was registered. In order to
excrements the red colour colonies with metal identify isolated causative agent it is
shine have grown on Endo media. They were necessary to:
agglutinated by polyvalent ОК-serum on A Study antigenic properties of the causative
slide. How to establish the pathogenicity of agent
the colon bacilla? B To determine sensitivity to antibiotics
A. Due to antigenic properties. C To study sensitivity to bacteriophages
B. Due to morphological properties. D To study biochemical properties of the
C. Due to cultural properties. causative agent
D. Due to toxigenic properties. E To study virulence of the causative agent
E. Due to phage sensitiveness
97. The 6-monthed child was entered to the Microbiological diagnostics of Salmonella
infectious hospital with diagnosis “acute infection
gastroenteritis”. On what nutrient medium it 102. During the repeated Widal's
is necessary to sow faeces of ill child? agglutination test it was noticed that the ratio
A. Moncur’s medium of antibody titers and O-antigens {S.typhi} in
B. Blood agar the patient's serum had increased from 1:100
C. Endo medium to 1:400. How would you interpret these
D. Ploskirev’s medium results?
E. Serum agar A The patient has typhoid fever
98. From the defecation of a 6-year-old ill B The patient is an acute carrier of typhoid
child, who has artificial feeding, the intestinal microbes
bacillus with antigen structure 0-111 is C The patient is a chronic carrier of typhoid
excreted. What is the diagnosis? microbes
A Food poisoning D The patient previously had typhoid fever
B Gastroenteritis E The patient was previously vaccinated
C Cholera-like diseasis against typhoid fever
D Coli-enteritis 103. A patient with complaints of 3-day-long
E Disentery-like diseasis fever, general weakness, loss of appetite came
99. Sanitary bacteriological research on water to visit the infectionist. The doctor suspected
by the membrane filter method revealed two enteric fever. Which method of laboratory
red colonies on a membrane filter (Endo agar) diagnosis is the best to confirm the
through which 500 ml of analyzed water were diagnosis?
passed. Calculate the coli index and coli titer A Detachment of myeloculture
of the analyzed water: B Detachment of blood culture
A 2 and 500 C Detachment of feces culture
B 4 and 250 D Detachment of urine culture
C 250 and 4 E Detachment of pure culture
D 500 and 2 104. On bacteriological study of rinsing
E 250 and 2 water of the patient with food poisoning, the
100. As a result of durative antibiotic therapy pure bacterial culture was inoculated with the
a 37-year old patient developed intestinal following properties: gram-negative motile
dysbacteriosis. What type of drugs should be bacillus in the Endo environment grows like
used in order to normalize intestinal achromic colony. Representative of what
microflora? genus has caused the illness?
A Vitamins A Yersinia
B Sulfanilamides B Shigella

12
C Salmonella reaction in dilutions from 1:10 to 1:1280 and
D Escherichia erythrocytes, which were sensitized by
E Citrobacter microbe antigen. What serologic reaction was
105. Effective diagnostics of the intestinal used?
infections agents is based on antibodies to A. Direct agglutination
bacteria antigens revealing in indirect B. Precipitation
hemagglutination test. What standard C. Passive hemagglutination
preparation must be used in this reaction? D. Coomb`s
A. Erythrocyte diagnosticum with adsorbed E. Opsonization
antigens of bacteria 110. The pure culture of bacteria was isolated
B. Antibodies against immunoglobulins of the from a pathological material. It was partially
main classes identified by morphological, tinсthorial,
C. Monoclone antibodies cultural and biochemical properties of
D. Mono-receptor diagnostic serums bacteria in bacteriological laboratory. The
E. Ram’s erythrocytes and hemolytic serum type-specific adsorbed agglutinative serum
106. Reaction of passive hemagglutination was chosen for final identification. What type
conducted with erythrocytic typhoid Vi- of agglutination test should be used?
diagnosticum A. Reaction of slide agglutination
helped to reveal some antibodies in the B. Reaction of agglutination (variant of
dilution of the patient's serum at a ratio of Widal)
1:80 that exceeds the diagnostic titer. Such C. Reaction of agglutination (variant of
result witnesses of: Gruber)
A Being ill with acute typhoid fever D. Reaction of hemagglutination
B Being a potential carrier of typhoid bacilli E. Reaction of passive hemagglutination
C Typhoid fever recurrence 111. Serological diagnostics of thyphoid is
D Incubation period of typhoid fever found on specific interaction of antigens and
E Reconvalescence of a patient ill with antibodies. How reaction is called when
typhoid fever electrolyte’s presence microorganisms are
107. A 50-year-old patient with typhoid fever stuck together under a specific antibodies’
was treated with Levomycetin, the next day influence?
his condition became worse, temperature A. Reaction of precipitation
rised to 39,60С. What caused worthening? B. Reaction of agglutination
A Reinfection C. Reaction of complement’s fixation
B Allergic reaction D. Reaction of hemadsorption
C Irresponsiveness of an agent to the E. Reaction of neutralization
levomycetin 112. Serological diagnostics of infectious
D Secondary infection addition diseases is found on specific interaction of
E The effect of endotoxin agent antigens and antibodies. How reaction is
108. The pathogen was isolated from patient’s called when highly dispersive antigens are
organism (patient suffers from acute adsorbed on erythrocytes?
gastroenteritis). It must be identified by A. Reaction of neutralization
antigen structure. What serologic reaction B. Reaction of precipitation
must be used? C. Reaction of complement’s fixation
A. Reaction of agglutination D. Reaction of hemadsorption
B. Reaction of complement’s binding E. Reaction of indirect (passive)
C. Reaction of neutralization hemagglutination
D. Reaction of precipitation 113. The pure culture of bacteria was isolated
E. Reaction of opsonization at bacteriological research of patient with the
109. For serologic diagnostics of infectious food poisoning. It has such properties:
disease patient’s blood was delivered for gramnegative mobile rods on an Endo’s
analysis. Blood sera was dissolved by isotonic media forms the colourless colonies. Name
solution. Patient’s serum was used for their genus.

13
A. Shigella D. Formation of hemolysis on bloody
B. Iersinia agar
C. Esherichia E. Formation of tender tape on
D. Citrobacter alkaline pepton water
E. Salmonella 118. A laboratory got the blood of patient
114. For the serological diagnostics of with typhoid (15th day of illness) for
typhoid the reaction, when to a different rising of agglutination reaction. Indicate a
solubilisations of patient’s sera the reagent, which would give a positive
diagnosticums of three types of reaction.
microorganisms are added is used, and the A. The typhoid H-diagnosticum
result of which is estimated on the presence of B. Typhoid O-diagnosticum
sediment from agglutinated bacteria. This C. Diagnosticum typhoid with Vi-
reaction name is: corpuscules
A. Ascoli D. Sera to H-antigens of S. typhi
B. Borde-Gangou E. Sera to O-antigens of S. typhi
C. Wasserman 119. Patient with a suspicion on typhoid
D. Wright was sent to the bacteriological research
E. Widal of blood by the doctor. What is observed
115. Bacteriological examination of a patient during the first weeks of typhoid-
with food poisoning required inoculation of a paratyphoid?
pure culture of bacteria with the following A. Septicopyemia
properties: gram-negative movable bacillus B. Bacteriemia
that gro on the Levin's medium in form of C. Toxinemia
colourless colonies. A representative of which D. Virusemia
species E. Septicemia
caused this disease? 120. Tо infectious department of hospital a
A Esherichia person of 37 years old with the clinical
B Shigella signs of typhoid was admitted. Тerm of
C Iersinia disease is 5 days. Bacteriological research
D Salmonella of patient`s feces gave a negative result.
E Citrobacter What method of diagnostics will turn out
116. 55-year-old patient with typhoid fever most expedient for clarification of clinical
was treated by chloramphenicol. The next day diagnosis?
the patient's condition deteriorated, the A. Bacteriological research of blood
temperature rose to 39.6 C. How do you B. Bacterioscopical research of blood
explain the deterioration condition of the C. Serological research of blood
patient? D. Bacteriological research of urine
A. Allergic reaction E. Bacteriological research of bile
B. The effect of endotoxin 121. A woman of 32 years old has a fever, a
C. Irresponsiveness of an agent to antibiotic headache during a week , she is freezing, a
D. Secondary infection addition liver and a spleen are megascopic, the
E. Reinfection elements of roseol rash are noticeable on
117. From the patient`s blood the culture of the skin of stomach. A doctor diagnosed
typhoid agent was isolated. What are the "typhoid". What pathological material is
cultural characteristics of this bacteria? necessary to send for clarification of
A. Formation of colourless colonies diagnosis?
on Bismuth agar A. Sera of blood
B. Formation of red colonies with B. Stroke of blood from a finger
metallic brilliance on the Endo’s C. Blood from an elbow vein, 10 ml
media D. Bile
C. Formation of colourless colonies on E. Feces
an Endo’s and Ploskirev’s media

14
122. A man of 57 years old, who has typhoid A. Titer of antibodies considerably increases
for 2 years, rejects to pass the inspection only at the secondary immune response
of carrying the bacteria. He points that B. Input of a typhoid vaccine forms the
there is no way he can be a transmitter of cellular immunity, instead of humoral one
Salmonella typhus. A doctor revised his C. Presence of natural specific immunity
medical card and agreed with him. What interferes with formation of postvaccinal
is the conclusion based on? immunity
A. A man had the easy form of D. During the course of preparation the
typhoid vaccine has lost immunogenic properties
B. A man got the inoculation by E. It is necessary to inoculate a vaccine
TABte vaccine together with adjuvant for more active
C. Typhoid carrying of bacteria immunity formation
proceeds for half-year 127. Special antibodies were appeared in the
D. A patient had is promoted acidity patient with typhoid fever on the second week
of gastric juice of illness. What are the mechanisms of their
E. A patient has no gall-bladder protective action?
123. Patient came to doctor-infectionist with A. Opsonization, complement activation
complaints to 3 days long fever, general B. The neutralization of exotoxins
weakness, worsening of appetite. Doctor C. Activation of T-killers
supposed that it was enteric fever. Which D. Activation of B-lymphocytes
method of laboratory diagnosis is the best to E. Activation of T-suppressors
confirm the diagnosis? 128. Reaction of passive hemagglutination
A. Detachment of blood culture conducted with erythrocytic typhoid Vi-
B. Detachment of myeloculture diagnosticum was used to reveal some
C. Detachment of feces culture antibodies in the dilution of the food business
D. Detachment of urine culture workers's serum at a ratio of 1:80 during
E. Detachment of pure culture planned survey. Is this man a carrier?
124. Patient was admitted to the infectious A.Yes, he is
clinic with a previous diagnosis “typhoid”. B. He is reconvalescent
He feels sick during three days. What C. This is disease
method will give possibility to make a D. He is not a carrier
diagnosis? E. He was vaccinated
A. Isolation of haemoculture 129. In the area where the expected outbreak
B. Isolation of urineculture of typhoid fever, it is necessary to carry out
C. Rising of Widal`s reaction school children prevention. Which drug is
D. Biological method more appropriate to use?
E. Isolation of coproculture A. TABte vaccine
125. Agglutination test was used for serologic B. Vi-antigen riched typhoid vaccine
diagnostics of typhoid. What component C. The typhoidal polyvalent bacteriophage
need to be used for this reaction except D. Donor gammaglobulin
patient serum? E. The correct antibiotic
A. Anatoxin 130. 37 yeared man with clinical symptoms
B. Diagnostic serum of typhoid fever delivered to the infectious
C. Complement department of the hospital, duration of disease
D. Hemolytic system is 5 days. Bacteriological examination of
E. Diagnosticum patient feces gave a negative result. What
126. Inactivated typhoid vaccine was checked method of diagnosis would be more
on rabbits and shows such results: antibodies appropriate to clarify the clinical diagnosis?
titer before immunization was 1:5 and after A. The bacteriological examination of blood
immunization was 1:5. How it is possible to B. Bacterioscopic study of blood
explain it? C. Serological study of blood
D. Bacteriological study of urine culture

15
E. Bacteriological study of bile D. The inhibition of hemagglutination
131. A 32-yeared woman has a high fever, E. Vi-hemagglutination
severe headache, fever, liver and spleen were 136. There are sporadic cases of typhoid fever
enlarged and visible elements roseola rash on in the city A. What method of microbiological
the skin of the abdomen during the week. The diagnosis can not be used to identify people
doctor diagnosed “typhoid fever?”. What infected with S. typhi?
pathological material should be sent for A. Biological
examination to clarify the diagnosis? B. Bacteriological
A. Feces C. Serological
B. Blood smear from a finger D. Allergotest
C. Serum 137. A patient`s blood with a preliminary
D. Bile diagnosis "Typhoid fever" is studied in the
E. Blood from the cubital vein,10 ml. laboratory. After which time, since the
132. The patient was arrived to the hospital on serological diagnostics method of infectious
the eighth day with complaints of headache diseases can be most effective?
and weakness. Blood was taken for A. One month after
serological examination. Widal agglutination B. After 3 days
showed positive result in a dilution 1:200 with C. After 12 hours
typhoid O-diagnosticum. What is the D. After a week
diagnosis can be made on the E. From the onset of the disease
basis of this study? 138. The pure culture of bacteria isolated
A. Tuberculosis from a patient with suspected typhoid blood is
B. Dysentery studied in the bacteriological laboratory.
C. Cholera What serological tests should be used to study
D. Leptospirosis the antigenic structure?
E. Typhoid fever A. Precipitation
133. After inoculation of feces specimen from B. Agglutination
a patient with typhoid fever onto Endo media C. CBT
colonies of different size and colour – big red D. ELISA
and medium colourless – have grown. Name E. Flocculation
the functional type of this media. 139. An outbreak of food poisoning
A. Special associated with consumption of confectionery
B. Elective products which are stored at room
C. Differential diagnostic temperature and manufactured from duck
D. Selected eggs was registered. Which organisms can
E. Enriched cause a poisoning?
134. Widal agglutination test showed patients A. E. coli
serum antibodies titer to S.typhi Vi-antigens B. Salmonella
rise from 1:100 to 1:1800. How can we C. Staphylococci
explain these results? D. Legionella
A. Has typhoid E. Vibrio cholerae
B. An acute typhoid bacteria carrier 140. The gramnegative medium size motile
C. A chronic carrier of typhoid bacteria rods with rounded ends which are
D. Had typhoid agglutinated with salmonella group B O-
E. Previously been vaccinated against typhoid serum were revealed in patient at
135. In connection with the outbreak of bacteriological study of vomiting mass. The
typhoid has become necessary to survey identical organisms were revealed in meat the
employees of a café. What reaction is used to salad which recently was eaten by all patients.
diagnose the carrier? A pathogen of which diseas is it in this case?
A. The reverse hemagglutination A. Salmonella-agent of acute gastroenteritis
B. The indirect hemagglutination B. Salmonella pathogen of typhoid
C. Latex agglutination C. Salmonella paratyphi A pathogen

16
D. Esherhii causing foodborne negative result?
E. Proteus foodborne pathogen A. Presence of pathogen in an intestine
141. The boy, aged 12, remains in hospital B. Presence of pathogen in a blood
with suspected food poisoning. A large C. Presence of antibodies in a blood
number of colorless colonies were grew on D. Absense of typhoid in a patient
Endo medium after patient faeces inoculation. E. Presence of pathogen in bilious
Which organisms can most likely be 146. Inspection of school dining-room staff
eliminated with the number of possible for typhoid carrying has been carried out.
causative agents of disease? There were revealed antibodies to Vi-
A. Pseudomonas aeruginosa antigen in the blood serum of the cook.
B. Salmonella enteritidis Which test has been used?
C. Proteus vulgaris A. Complement fixation test.
D. Escherichia coli B. Widal's t est.
E. Yersinia entercolitica C. Indirect hemagglutination test.
142. The majority of guests who attended the D. Enzyme immunoassay.
banquet had vomiting, abdominal pain, E. Immunofluorescence test.
diarrhea and fever after 12 hours. The 147. Identification of food toxical
anamnesis revealed that all of them ate meat infection causative agent has shown that
salad, pickled cucumbers, zucchini, potatoes, according to biochemical properties it
biscuits with cream and fruit juices. Which of belongs to Salmonella genus. What property
organisms can be identified by bacteriological of the causative agent testifies its specific
examination of material from the patients? belonging?
A. S.enteritidis A. Pathogenicity for laboratory animals.
B. C.botulinum B. Phagotype.
C. S.aureus C. Culture properties.
D. E.coli D. Antigenic structure.
E. C.perfringens E. Morphologic and staining properties.
143. The pure culture of bacteria isolated 148. Feces of a restaurant cook without any
from a patient with food poisoning. It was clinical symptoms of disease are examined
identified as with the help of bacteriological methods.
Salmonella enteridis. In this case we are On bismuth-sulfite agar little black
talking about: colonies with metallic sheen have grown.
A. Serotype What microorganisms can it be?
B. Species A. Salmonellae.
C. Genus B. Shigellae.
D. Hemovary C. Escherichiae.
E. Biovary D. Staphylococci.
144. The serological test was used for E. Streptococci.
bacterial culture seroindentification in the
bacteriological laboratory with the material
from a patient with suspected salmonella Microbiological diagnostics of Shigellosis
gastroenteritis. What type of reaction can be 149. Sh.sonnei were isolated from feces of
supplied with this antigen? patient. Which additional researches must be
A. The reaction of neutralization used for establishment of the source of
B. Agglutination infection.?
C. The reaction of precipitation А. To applay the phagotyping of isolated
D. The reaction of indirect agglutination pure culture
E. The reaction of flocculation В. To applay the antibioticogramm
145. Widal's agglutination test was used for С. To applay the reaction of agglutination RA
patient with suspected typhoid after 2 weeks D. To applay the reaction of indirect
from the beginning of disease. The reaction hemagglutination RIHA
was negative. What can be the reason of E. To applay immunofluorescence reaction

17
IFR D. Hemolysis
150. The culture of Shigella was isolated in E. Bacteriolysis
patient with dysentery. It was established, for 154. Shigella with ability to produce
what phages it was sensible to. However the exotoxins were isolated from a patient
using of these phages as the method of diagnosed with dysentery. What species of
specific therapy turned out vain. What is the Shigella talking about?
most credible reason of failure? A. Shigella boydii
A. A pathogen can live inside the cells B. Shigella sonnei
B. In the humans’ organism a pathogen does C. Shigella flexneri
not form receptors for attachment of D. Shigella dysentery
bacteriophage E. Shigella New Castle
C. Used phages are moderate 155. In the infectious ward hospitalized
D. An infectious process is supported due to patient complaining of nausea, liquid
mutants emptying with slime and blood streaks, fever,
E. In humans’ organism phages are weakness. The doctor suspected dysentery.
inactivated What is the most expedient method of
151. In nursery school the flash of acute laboratory diagnosis to confirm
intestinal infection was registered. Children, the diagnosis?
who fell ill, suffered of diarrea, feces with A.Bacteriological
mucus and blood, a temperature was B.Serological
promoted to 380С. They have stomach-ache, C.Mycological
frequent vicious urges on defecation. Indicate D.Microscopic
the most credible pathogen: E. Protozoological
A. Escherishia coli 156. In patients with symptoms of colitis pure
B. Salmonella enteritidis culture of bacteria, which on morphological,
C. Shigella flexneri culturing and biochemical properties
D. Entamoeba dysenteriae attributed to the genus Shigella were isolated.
E. Yersinia enterocolitica Which of these reactions should apply for the
152. The bacteriological method must be used serological identification?
to identify sources of infection during A. Agglutination with diagnostic serums
outbreaks of shigellosis. The research yielded B. Complement fixation
no result. It was decided to use C. Indirect hemagglutination
phagediagnostics. Such research includes: D. Precipitation
A. Determination of phagocytes activity of E. Delays hemagglutination
examined blood 157. The patient was taken to the hospital
B. Detection of bacteriophages in with complaints of headache, fever, frequent
pathological material bowel movements, abdominal pain with
C. Determination of phage type tenesmus. The doctor made a clinical
D. Performances bacteriophage titer increase diagnosis of "dysentery" and sent
reaction pathmaterial (excrement) in the
E. Detection of functional abnormalities in the bacteriological laboratory. Which method for
digestive system diagnosing physician-bacteriologist was to
153. To resolve the issue retrospective confirm or refute the diagnosis?
diagnosis of bacterial dysentery was A. Allergotest
transferred assigned to serological study of B. Biological
blood serum to determine antibody titer to C. Bacterioscopic
Shigella. Which of these reactions is D. Serologically
appropriate for this? E. Bacteriological
A. Passive hemagglutination 158. Pure culture of dysentery agent was
B. Complement fixation isolated from patient in the laboratory. What
C. Precipitation research should be carried out for final
serological identification?

18
A. Use agglutination test with the standard B. Excludes the diagnosis of dysentery
serums C. Previously suffered dysentery
B. Carry out agglutination test with serum of D. Pseudoreaction
the patient E. Vaccination reaction
C. Use indirect hemagglutination reaction 163. Shigella Sonnei. was isolated from
D. Carry out the reaction of molecular patient`s defecation with intestinal infection.
hybridization Which of the following serological tests were
E. Find thermostable antigens in ring used to identify an isolated pure culture?
precipitation test A. Agglutination test
159. In a patient who became ill 3 days ago B. The reaction of precipitation
and complained of fever (38 C), abdominal C. Fixation of complement
pain, frequent bowel movements, presence of D. Neutralization reaction
blood in the stool, a physician clinically E. Lysis reaction
diagnosed bacterial dysentery. What method 164. Phage prevention of shigellosis must be
of microbiological diagnostics should be done to group of students were in contact with
apply in this case and what material should be patients during outbreaks. What mechanism
taken from a patient? will ensure their protection?
A. Bacterioscopic - blood A. Phage typing
B. Bacterioscopic - excrements B. Phagocytosis
C. Bacteriological - excrements C. Phage lysis
D. Bacteriological - urine D. Pinocytosis
E. Antibodies - blood E. Small diffusion
160. In 4 days 10 children fell ill with clinical 165. Multiple brownish-green layers,
signs of acute intestinal infection in different haemorrhage: the gut lumen mucus, a small
age groups of kindergarten. Sonnei dysentery amount of blood were revealed in the mucosa
agent was revealed in feces of these patients. of rectum and sigmoid colon of the 46 yeared
Due to the unfavorable epidemiological man body after opening, histologically -
situation in this children's group contact fibrinous colitis. S. sonne was revealed at
among children is necessary to conduct bacteriological examination of the intestinal
prevention activities. Which drug for specific contents. What is the most likely diagnosis?
prevention should be prescribed to children A. Salmonellosis
that were in contact with these patients? B. Cholera
A. Dyzentery bacteriophage C. Crohn's disease
B. Antibiotics D. Yersiniosis
C. Sulfanilamides E. Dysentery
D. The vaccine TABte 166. The culture of microorganisms, which
E. Immunoglobulin were isolated from a patient was brought in
161. The patient recovered after dysentery the conjuctival sack of guinea-pig with the
Sonnei and reinfected by the same pathogen. purpose of diagnostics of bacterial dysentery
As this infection is called? by a biological method. A result turned out
A. Chronic infection positive – conjunctivitis was developed in an
B. Relapse animal. The presence of what factor of
C. Superinfection pathogenicity of dysenteric bacteria was
D. Persistent infection exposed by this way?
E. Reinfection A. Enterotoxin
162. Shigella were not revealed in feces of B. Cytotoxin
patient with typical clinical signеs of C. Endotoxin
dysentery due to early use of antibiotics. D. Capsule
Antishigella antibodies titer increased 4 times E. Pili
in the reaction of direct hemagglutination in 167. Colitis was exposed in the time of
paired sera of this patient. What this means? dissection of the deceased child, who suffered
A. Confirms the diagnosis of dysentery of diarrhoea. In the smear-imprint of mucus

19
membrane gram-negative bacteria were delivered to a hospital in grave condition and
revealed. What is the most credible diagnosis? died 2 days later. Autopsy of the body
A. Staphylococcus intestinal infection revealed the following diphtheritic colitis with
B. Cholera multiple irregularly-shaped ulcers of different
C. Salmonellosis depth in both sigmoid colon and rectus.
D. Dysentery Bacteriological analysis revealed
E. Typhoid Shigella.What was the main disease?
168. For the purpose of retrospective A. Nonspecific ulcerous colitis
diagnostics of recent bacterial dysentery it B. Salmonellosis
was decided to perform serological C. Crohn's disease
examination of blood serum in order to D. Yersiniosis
determine antibody titer towards Shiga bacilli. E. Dysentery
What of the following reactions should be
applied?
A Bacteriolysis Microbiological diagnostics of cholerae
B Bordet-Gengou test 173. In the feces of a patient with acute
C Precipitation gastroenteritis there were revealed motile,
D Hemolysis slightly curved Gram-negative rods, which grow
E Passive hemagglutination onto alkaline 1% peptone water in the form
169. A patient recovered after Boyd of tender bluish pellicle. What
dysentery and was once more infected with microorganisms have such properties?
the same causative agent. What such infection A. Spirilla.
form is called? B. Spirochetes.
A Recidivation C. Clostridia.
B Reinfection D. Bacilli.
C Superinfection E. Vibrios.
D Persisting infection 174. As a result of feces inoculation onto 1%
E Chronic infection alkaline peptone water after 8 hours'
170. The pathogen of dysentery was isolated incubation with 37° C temperature a growth
in the child with an acute intestinal in the form of tender bluish pellicle was
infection. What morphological and revealed. Microscopy revealed Gram-negative
tinctorial signs are characteristic for this curved rods. What disease could these
pathogene? microorganisms cause?
A. Gram-negative, immobile monobacteria A. Shigellosis.
B. Gram-negative, mobile monobacteria B. Plague.
C. Gram-positive, monobacilli C. Typhoid fever.
D. Gram-positive, streptobacilli D. Paratyphoid fever.
E. Gram-negative, vibrios E. Cholera.
171. At the bacteriological laboratory a 175. A patient is hospitalized to an
request for acquisition of preparations for infectious department with cholera
diagnostics of intestinal infections is aquared. suspected. What basic method of research is
Which of the preparations adopted below was necessary to use for the confirmation of the
included to the list uncorrectly? diagnosis?
A. A serum choleraic O1 A. Immunological.
B. A phage choleraic El-Tor B. Bacteriological.
C. A luminiscent typhoid serum C. Biological.
D. Dysenteric polyvalent phage D. Serologic.
E. Erythrocyte diagnosticum with Shigella E. Allergic.
flexneri 176. Feces of a patient with cholera were
172. A 71-year-old man had been presenting delivered to a laboratory of extremely
with diarrhea for 10 days. The feces had dangerous infections. What method of
admixtures of blood and mucus. He was

20
microbiological diagnostics is to be used to D. Method of peptone water inoculation
confirm or deny the diagnosis? E. Method of peptone agar inoculation
A. Virological. 181. 42-years old man has symptoms of
B. Allergic. cholera. Bacterioscopic analysis of feces and
C. Bacterioscopic. serological analysis of serum was confirmed a
D. Biological. clinical diagnosis, while the repeated attempts
E. Bacteriological. to isolate the pure culture of Vibrio cholerae on
177. From the vomit mass of a patient there ordinary for choleric vibrio media were not
were isolated very motile, slightly curved, successful. What is the most probable reason of
Gram-negative rods which react positively failures?
with Inaba's diagnostic serum. What symptoms, A. The tool that was used for the inoculation
most probably, will appear with the of the pathological material had traces of
treatment absent? disinfectants
A. Endotoxic shock. B. The anaerobic conditions of growing were
B. Bacteremia. not provided
C. Fluid loss. C. The agent is auxotrophic mutant
D. Skin rash. D. Culture was infected with virulent
E. Ulcerous damages of intestine. bacteriophage
178. A patient with complaints of E. Isolated culture only morphologically
repeated diarrhea and vomiting, pain in similar to Vibrio cholerae
muscles of legs, general weakness, and 182. Culture of cholera vibrios was isolated
vertigo is hospitalized to an infectious from feces and vomit of patient. Conducting
department. After examination a doctor has of which reaction will determine the type of
previously diagnosed cholera. Which method microbe that caused the disease?
of investigation of the specimen from the A. Agglutination with serum containing O-
patient should be used for express- antibodies
diagnostics? B. Agglutination with serum containing H-
A. Immunofluorescence test. antibodies
B. Agglutination test. C. Passive hemagglutination with erythrocyte
C. Bacteriological. diagnostics
D. Allergic. D. Agglutination by Widal
E. Biological. E. Precipitation
179. Patient with diarrhoea was admitted to 183. The outbreak of cholera was marked in
the infection unit. Gramnegative curved rod- the region. It is necessary to reveale which
like biovary of Vibrio cholerae is a pathogen.
bacteria were founded on bacterioscopic Which of the following methods give the
examination of faecal masses. What is the most reliable results?
most likely disease in this patient? A. Biochemical identification
A. Cholera B. Serological identification
B. Typhoid fever C. Bio test
C. Salmonellosis gastroenteritis D. The polymyxin test
D. Diphtheria E. Morphological
E. Intestinal form of plague 184. The 10-yeared child with diarrhea,
180. The one of identification stages of vomiting, dry skin, cyanosis, ischuria (urinary
diagnostics of the patient with previous retention) was hospitalized to infectious
diagnosis “Cholera” is the revealing of department of the hospital. Pathmaterial was
infectious agent monotrichal mobility. What delivered to the laboratory. What is the most
method of determining is used for this probable result of pathmaterial inoculation
purpose? onto medium?
A. Method of the "hanging" or "crushed" drop A. Blue colonies on alkaline agar
B. Gram’s method B. Red colonies on Endo medium
C. Loeffler’s method C. Colourless colonies on Endo medium

21
D. Yellow colonies on yolk salt agar of cholera gene action is:
E. Large mucous colonies on meat pepton A. Blocks transferase-2
broth B. Causes damage of erythrocytes
185. Comma shape gram negative bacteria C. Causes salts hypersecretion
were revealed in the smears from the patient`s D. Increases fluid and electrolyte balance
feces. What properties need to be studied E. Activates adenilatcyclase
firstly with microscope use for additional 190. The pure culture of Gram-negative,
information about microorganisms? slightly curved, motile rods that fermented
A. The presence of cysts mannose and sucrose to acid and was
B. The presence of capsules agglutinated by O1 choleraic anti-sera was
C. The presence of spores isolated from patient with acute gastroenteritis
D. Mobility feces. What media was used for isolated pure
E. The primary fluorescence culture?
186. Exo-and endotoxins, enzymes of A. 1% peptide alkaline water, TCBS media
aggression play significant role in the B. Blood meat pepton agar, Ploskirev`s
pathogenesis of cholera. The main syndrome medium
of the disease is dehydration. Select which of C. 1% pepton alkaline water, Endo media
these pathogenic factors cause dehydration is D. Meat peptide agar, meat pepton broth
the main. E. Sugar and meat agar, alkaline meat pepton
A. Defect of membrane phospholipids agar
B. Splitting of neuraminic acid 191. Feces of a patient C. with cholera
C. Destruction of hyaluronic acid were delivered to a laboratory of
D. Adenilatcyclase activating extremely dangerous infections. What
E. Destruction of mucine method of express diagnostics is to be used
187. Patients was hospitalized to the to confirm the diagnosis?
infectious department with suspected cholera. A. Immunofluorescence
What is the primary diagnostics method B. Complement fixation test
should be used to confirm the diagnosis? C. Agglutination test
A. Immunologic D. Precipitation
B. Bacteriological E. Haemagglutination test
C. Biological 192. The patient with witnessed recurrent
D. Serological diarrhea, vomiting, rice broth like feces,
E. Allergic lowered body temperature and heart
188. The laboratory of extremely insufficiency was hospitalized to the infection
dangerous infections was performed ward. His skin is bluish and wrinkled. Which
microscopic studies of smears made from film pathogenicity factor lead to the development
that has grown on 1% pepton water within 6 of these disorders?
hours of feces taken from patients with A. Endotoxin
suspected cholera cultivation. On the basis of B. Enterotoxin
which the morphological and tinctorial C. Invasiveness
properties it is possible to do a preliminary D. Aggression
conclusion about the presence of Vibrios in E. Hemolysins
the grown film? 193. Sanitary Epidemiological Station of
A. Mobile, Gram-negative curved bacillus town S. is controlling the possibility of
B. Movable, curved gram-variable bacillus V.cholerae detection in seawater. What is the
C. Movable, curved gram-positive bacillus epidemiology of cholera in this feature is
D. Mobile, Gram-negative straight fusiform taken into account?
rods A. Probability of hospital infection
E. Mobile, gram-positive pair bacillus B. Zoonotic
189. Cholerogen is a main pathogenesis factor C. Sapronotic
of cholera. It determines dehydration of D. Probability of iatrogenic infection
patients organisms with cholera. Mechanism E. Vector borne transmission

22
194. Microscopy study of patient with bloody diarrhea. Gram-negative helical
diarrhea defecation was revealed the bent rod bacteria were revealed during bacteriological
organisms that look like fish flocks. Which examination of feces. They do not form
bacteria were revealed? spores and capsules, are microaerophilic,
A. Vibrio catalase positive, urease positive and mobile.
B. Bacteria Name the bacteria, which are characterized by
C. Bacillus these properties.
D. Spirillum A. Proteus mirabilis
E. Actinomycetes B. Escherichia coli
195. The cholera vibrios pure cultures isolated C. Haemophilus influenzae
from 31 yeared carrier K. were studied in a D. Helicobacter pylori
laboratory of extremely dangerous E. Salmonella typhy
infections. Which properties of cholera 200. The bacteriological examination was
agent can be determined in unfixed proposed to patient to establish the etiology of
preparation? gastric ulcers. What microorganisms would
A. Monotryhic mobility be reavealed?
B. Peritryhic mobility A. Salmonella
C. Tinctorial properties B. Listeria
D. Capsule formation C. Leptospira
E. The arrangement of bacteria D. Shigella
196. The bacteriophage titer increase reaction E. Helicobacter
with the standard cholera bacteriophages was 201. The biopsies of the ulcer in mucous
used for sea water quality examination. The membrane were took in patient with stomach
reaction was positive. This result is evidence ulcer during fibrogastroscopy. Gram-negative
of: helical bacteria were revealed during
A. Escherichia coli presence microscopy examination of biopsy smears
B. Absence of cholera pathogens in samples with positive urease activity test. Which
C. Presence of organic residues in samples bacteria have been revealed?
D. Absence of organic water pollution A. Campylobacter jeuni
E. Presence of cholerae pathogens in samples B. Spirilla minor
197. In the feces of a patient with intestinal C. Shigella flexneri
decease there were revealed Vibrio. What D. Treponema pallidum
group of morphological properties this E. Helicobacter pylori
microorganisms belong to? 202. The patient with acute colitis with
A. Spiral symptoms of mild intoxication, diarrhea,
B. Clostridium blood in stool was hospitalized. Gram-
C. Cocci negative helical, microaerophilic, catalase
D. Bacteria positive, urease positive and mobile bacteria
E. Bacillus without spores and capsules were revealed
198. Diagnosis “Helicobacteriosis” was put to during bacteriological examination. They
the patient after bioptate resemble the wings of a gull. Name these
esophagogastroduidenoscopy. What bacteria.
characteristics of isolated from this patient A. Escherichia coli
bacteria was taken into account at cultivating? B. Helicobacter pylori
A. Lack of spores and capsules C. Haemophilus influenzae
B. The presence of the urease enzyme D. Proteus mirabilis
C. Colonization of the gastric type mucosal E. Salmonella typhy
cell 203. The motile, gram-negative bacilli were
D. Microaerophilic revealed in the suspected foods. They showed
E. The presence of six polar flagella creeping growth on meat peptone agar
199. Patient S., aged 28, was hospitalized (MPA)in the 18-hour culture. Isolates
with symptoms of mild intoxication and produced gas H2S and indole, fermented

23
glucose, maltose and sucrose to acid and did The infectionist suspected brucellosis. Using
not ferment lactose, mannitol. Bacteriological what reaction is it possible to diagnose
study showed that the isolated bacteria belong brucellosis?
to the genus: А. Wright's.
A. Proteus B. Wasserman`s test.
B. Escherichia C. Coombs test.
C. Pseudomonas D. Widal's test.
D. Salmonella E. Ouchterlony test.
E. Shigella 208. The territory of the burial ground of
204. The symptoms of acute diarrhea were cattle, which has not been used for more
developed in 7 patients from the group of than 50 years, is planned for house building.
tourists (27 people) which used water from However, soil examination has shown the
the pond after 2 days. The material was presence of viable spores of an especially
delivered to the bacteriological laboratory to dangerous disease causative agent. Name
study etiology of this disease. Which material the microorganism which could have been
was used for the diagnosis of the disease preserved in soil during such a long time.
study? A. Yersinia pestis.
A. Food B. Francisclla tularensis.
B. Blood of patients C. Bacillus anthracis
C. Urine D. Brucella abortus.
D. Water and feces of patients E. Mycobacteriuni bovis.
E. Sputum 209. During a biological test in touch smears
205. Patient with diarrhoea was admitted to from the organs of an animal
the infection unit. Gramnegative curved rod- streptobacteria surrounded with a capsule
like were revealed. It gives the basis to
bacteria were founded on bacterioscopic diagnosis:
examination of faecal masses. What is the A. Brucellosis
most B. Tularemia
likely disease in this patient? C. Plague
A Cholera D. Anthrax
B Typhoid fever E. Crupous pneumonia
C Salmonellosis gastroenteritis 210. During a scheduled examination
D Diphtheria milkmaids had a Burnet's intracutaneus
E Intestinal form of plague allergy test. This test is used to detect
206. Patient with vomite was admitted to the hypersensitivity to:
infection unit. Gramnegative curved rod-like A. Brucellin
bacteria were revealed at bacterioscopic B. Tuberculin
examination of vomiting masses. What is the C. Koch's tuberculin
most likely disease in this patient? D. Tularin
A. Salmonellosis E. Anthraxin
B. Cholera 211. Veterinary attendant, working at a cattle
C. Crohn's disease farm complaints on joint pain, fever,
D. Yersiniosis indisposition and sweating at nighttime that
E. Dysentery he has been experiecing for a month. Giving
the regard to such presentations and
Microbiological diagnostics of brucellosis occupational history the doctor suspected
and anthrax brucellosis, despite the patient works on a
207. During the swing of flu epidemic a cattle farm and despite all complaints. What
milkmaid referred to a doctor with material taken from this patient is to be
complaints of high body temperature, analyzed in an ordinary microbiological
general weakness, absence of appetite, pain laboratory?
in joints. During 10 days she had treated flu. A. Feces

24
B. Spinal liquid 216. A man referred to the reception ward
C. Vomit masses of an infectious disease hospital, having
D. Urine received by mail an envelope with suspicious
E. Blood serum powder. The man was isolated, and the
212. A 34-year-old patient complained of powder was sent to the laboratory for
carbuncle on his face. Examination detecting the presence of anthrax causative
revealed a painless thin edema of agent's spores. Which is the fastest method
subcutaneous fatty tissue with a black of detecting these microorganisms?
eschar in the center, and vesicular eruption A. Precipitation in gel
on the periphery. Microbiologic B. Complement-fixation test
examination revealed nonmotile capsule- C. Luminescence immunoassay
forming streptobacilli. What microorganisms D. Pure culture isolation
are the causative agents of this disease? E. Biological assay on mice
A. Bacillus megaterium 217. A child has diagnosis “Brucellosis”.
B. Staphylococcus aureus He/she (child) was not in contact with the
C. Bacillus athracoides infected animals. How could the child was
D. Bacillus subtilis infected?
E. Bacillus anthracis A. Through fresh milk
213. A black-stained carbuncle has appeared B. Through unwashed fruits and vegetables
on the veterinary doctor's cheek after a dead C. Through water
cow examination. During a microscopic D. Through dirty hands
diagnostics the gram-positive bacilli in chains E. During the injections
looks like a bamboo stick were revealed. 218. A strobiloid red stained infiltrate with
What pathogen has these morphological and edema was found by doctor on the skin hand
tinctorial properties? of 42-yeared butcher. There is a black stained
A. P.vulgaris scab in the centre of the infiltrate. What
B. C.perfingens disease is it?
C. Y.pestis A. Flegmona
D. B.anthracis B. Abscess
E. F.tularensis C. Furunculosis
214. The doctor has put a preliminary D. Plague
diagnosis “Anthrax. Dermal form” to the E. Anthrax
examined patient. In this case microscopy of 219. A bacteriologist completed such
carbuncle’s exudates smears stained by Gram processes for a complete confirmation of the
will show: preliminary diagnosis: 1) Inoculation of the
A. Big violet bacilli with spores, which are pathological material onto liver and sugar
arranged in chain broth; 2) Isolators’ sensibility checking
B. Violet bacilli, which are arranged angularly (before antilinear staining agent’s action); 3)
to each other Serological reactions of Wright and
C. Pink bacilli, which are randomly arranged Haddlson; 4) Burnet’s allergen skin test. What
D. Pink bacilli with bipolar coloring diagnosis was confirmed with the help of
E. Slightly curved pink bacilli those bacteriological methods?
215. A veterinary doctor with the preliminary A. Q-fever
diagnosis “brucellosis” was delivered to the B. Tularemia
infectious hospital. On the basis of what C. Typhoid
serological test this diagnosis can be D. Salmonellosis
confirmed? E. Brucellosis
A. Wright’s agglutination reaction 220. A patient with brucellosis has a
B. Vidal’s agglutination reaction positive Burnet's intracutaneus allergy test.
C. Ascoli’s precipitation reaction Which immune system factor can induce
D. Veigl’s agglutination reaction inflammatory reaction in the site of
E. Complement-fixation reaction brucellin introduction?

25
A. Ig A D. Plague pathogen
B. Sensitized T-lymphocytes E. Bacteroides
C. Ig E 226. A patient complained about a carbuncle
D. Ig C on his face. Examination results: neither
E. Ig D dense nor painful edema of subcutaneous
221. A soluble thermostable antigen is cellular tissue, there is black crust in the
prepared in water-salt extract from raw- middle of the carbuncle and peripheral
material in order to check the animal raw- vesicular rash around it. Bacteriological
materials for the anthrax agent presence. examination revealed presence of immobile
What reaction is used for this? streptobacilli able of capsulation. What
A. Neutralization microorganisms are causative agents of this
B. Precipitation in agar disease?
C. Agglutination A. Bacillus antracis
D. Passive hemagglutination B. Staphylococcus aureus
E. Ring precipitation C. Bacillus subtilis
222. Anthrax pathogen was inoculated into D. Bacillus antracoides
gelatin and “inverted fir-tree” liquifaction of E. Bacillus megaterium
gelatin was observed after incubation. What 227. A 34 year old male patient consulted a
properties were studied in this case? doctor about face carbuncle. Objectively: a
A. Proteolytic loose, painless edema of hypodermic tissue:
B. Sacharolytic black crust in the centre of carbuncle,
C. Fibrinolytic vesicular rash around it. Microbiological
D. Hemolytic examination revealed static streptobacilli
E. Cultural capable building. What microorganisms are
223. Rodlike bacilla with capsule, located in the a causative agents of this disease?
chains were found during a microscopic A. Bacillus antracis
examination of carbuncle from patient with B. Bacillus megaterium
anthrax. What microbes were revealed? C. Staphylococcus aureus
A. Monobacillus D. Bacillus antracoides
B. Streptobacillus E. Bacillus subtilis
C. Monobacteria
D. Streptobacteria Microbiological diagnostics of plague and
E. Diplobacteria tularemia
224. In a laboratory the precipitation test is 228. A patient with the symptoms of
used (Askoly test) for the examination of tularemia was delivered to a city infectious
animal skins. An albescent ring formed in hospital. Which method can be used for early
some minutes after adding of the skin diagnostics of disease?
extract to the immune serum. What does A. Allergy test
this result indicate? B. Biological
A. Presence of Bacillus anthracis antigens C. Serological (agglutination reaction)
B. Presence of Clostridium perfringens toxin D. Bacteriological (pure culture isolation)
C. Presence of brucellosis causative agent E. Microscopic
D. Presence of Escherichia surface antigen 229. A patient with symptoms of tularemia
E. Presence of Salmonella Vi-antigen was delivered to the infectious hospital.
225. A patient complained of painless Which method is used for an early diagnostics
carbuncle on his face. Microbiologic nowadays?
examination revealed nonmotile capsule- A. Bacteriological
forming streptobacilli. What microorganisms B. Biological
are the causative agents of this disease? C. Serological
A. Anthrax pathogen D. RIF (express-method)
B. Pathogenic Clostridies E. Allergic
C. Fusobacteries

26
230. The lymph nodes material of 36-year-old A. CTI
man with a suspicion on plague were B. Alive vaccine EV
punctuated and inoculated on MPA in Petri C. Toxoid
dishes. Sterile round areas (1-1,5 mm) with D. Combined vaccine
homogeneous growth of microorganisms have E. Chemical vaccine
appeared after one day. How to explain the 234. What must be done to the hunter
appearance of these sterile spots? hospitalized on the 5-th day of the disease for
A. Wrong selective medium confirming the “tularemia” diagnosis on the
B. Pathological material had a small quantity early stages of diagnostics:
of causative agent A. RPHA
C. Cells in old cultures give an autolysis B. RA
D. Microorganism- antagonist was isolated C. Allergic test
together with causative agent D. CFR (Complement-fixation reaction)
E. Culture is lysogenic E. RIF
231. Immunofluorescence reaction was used 235. Dwellers of a village noticed mass
for recognizing a plague bacteria in the smear. mortality of rodents in some farms. There is It
The preparation was processed with anti- was suspected that the animals might have
plague serum. Glowing bacteria were revealed died from plague. What post-mortal
during luminescent microscopy. It was examination should be conduced in order to
estimated as presence of a plague causative establish the causative agent of the infection
agent inside the smear. Luminescence of as soon as possible?
plague bacteria is connected with: A. Complement fixation reaction
A. Anti-plague antibodies got connected with B. Agglutination reaction
the surface antigens of plague bacteria and C. Passive agglutination reaction
produce glow, because they are connected D. Ring precipitation reaction
with luminescent staining agent E. Neutralization reaction
B. An antigen-antibody reaction took place on 236. During diagnostics of plague the
the surface of bacteria credibility of bacteriological examination
C. Luminescent staining agent covered increases while using immunofluorescence
bacteria inside the smear reaction. Describe the microscopic picture at
D. Plague bacteria produce their own this reaction using:
luminescence A. Small coccal type pink bacteria
E. An antigen-antibody complex fixed the B. Small ovoid rods with bright green glow
complement on the surface of plague bacteria C. Big violet bacillus with chopped off limbs
232. Gram-negative ovoid bacteries with D. Small pink rods with rounded limbs
bipolar filling were isolated from sputum of E. Slightly arched red bacillus, situated
the patient with high temperature, shiver, angularly to each other
headache and cough. Bacteries are located in 237. A punctate of groin lymph nodes was
chains forms inside the smear of broth culture taken from a patient with suspicion on plague.
and create R-form colonies on agar. Which Material was inoculated into solid nutritious
disease is it? media. What looks must the colonies have if
A. Plague “plague” diagnosis is confirmed?
B. Tuberculosis A. “Patterned kerchief”
C. Anthrax B. “Drops of mercury”
D. Diphteria C. “Drops of dew”
E. Streptococcal angina D. “Shagreen skin”
233. A group of Ukrainian dentists must go to E. “Lion’s mane”
Africa on assignment. But it is known that 238. Ovoid microorganisms about 2 mcm
several hundreds persons have plague every lenth intensively stained on poles were
year in this country. What vaccine (from revealed during microscopy of the patient’s
listed below) must be used for plague sputum (the previous diagnosis is “acute
prevention?

27
pneumonia”). What is the most possible final D. Brucellosis
diagnosis? E. Leishmaniasis
A. Pulmonary plague form 242. A microbiological examination of
B. Pneumococcal pneumonia pathologic material stained by Gins-Burri
C. Staphylococcal pneumonia taken from a patient with a suspicion on
D. Klebsiella pneumonia plague and delivered to the laboratory of
E. Diphtheria extremely dangerous infectious diseases was
239. In a mountain settlement mass death of done. What property of causative agent this
rodents was observed. Simultaneously the method allows to study?
inhabitants of this area were ill. The illness A. Acidity
was accompanied by the fast rise of body B. Spore formation
temperature up to 40° C, apparent C. Capsule formation
intoxication, increase of inguinal lymph D. Alkaline resistance
nodes. In the touch smears of cadaveric E. Presence of volutin granules
material Gram-negative bipolarly stained
ovoid rods were revealed. What Microbiological diagnostics of tuberculosis
microorganisms are the causative agents of and actinomycosis
this infectious disease? 243. A 7-year-old child has an acute positive
A. Staphylococci tuberculin Mantoux test for the first time.
B. Costridia What does this result testify about?
C. Causative agents of tularemia A. About Hansens’ bacillus infection
D. Causative agents of anthrax B. About tubercule bacillus infection
E. Causative agents of plague C. About BCG vaccination
240. A doctor has suspected a bubonic form D. About Mantoux test had put before
of tularemia in the patient and sent the E. About tuberculosis disease
examined material to bacteriological 244. During official registration of the child
laboratory for bacteriological method of to school for the decision of question about
diagnostics. What is special for this method in the necessity of revaccination Mantoux test
this case? was negative. What does the given result
A. A pure culture is isolated from infected testify about?
laboratory animals A. About absence of antibodies to the
D. A pure culture is isolated on solid tubercular bacteria
nutritious media B. About the presence of cellular immunity to
C. A pure culture is isolated using the elective tuberculosis
media C. About absence of cellular immunity to
D. Isolated culture is identified using it’s tuberculosis
antigen structure D. About absence of antitoxic immunity to
E. A pure culture is isolated on fluid media tuberculosis
241. A muskrat-hunter’s temperature has E. About the presence of antibodies to the
increased to 39 degrees, headache and tubercular bacteria
weakness have appeared. A small exponent 245. At the study of phlegm, taken from a
appeared on his neck skin. A preparation was patient with suspicon on tuberculosis,
maid from it’s scraping. Rodlike gram- preparation was made and stained by Ziehl-
negative bacteria, that are very small without Neelsen. What microscopic picture is seen at
capsules and evenly stained were revealed. confirmation of the supposed diagnosis?
What causative agent can be spoken of, A. Red bacilli on a green background
considering microorganism morphology, B. Microorganisms with the kernel of ruby-
clinical picture and type of the patient’s red color and blue cytoplasm
activity? C. Red bacteria on a white background
A. Tularemia D. Chain like bacilli of violet colour
B. Anthrax E. Thin red bacteria on a blue background
C. Plague 246. Mycobacteria tuberculosis were not

28
revealed in the smear from a phlegm of E. To give BCG injection
patient with tuberculosis stained by Ziehl- 251. A 6-year-old child with active
Neelsen. With the help of what methods the tubercular process suspected had a
probability of the bacterioscopic revealing of diagnostic Mantoux test carried out. What
pathogen can be increased? immunobiological preparation was used?
A. To prepare preparation of hanging drop A. Tularin
B. To stain by other method B. BCG vaccine
C. To prepare preparation of thick drop C. APDT vaccine
D. To use the luminescent microscopy of D. Tuberculin
hanging drop E. ADТ vaccine
E. By the methods of flotation and 252. A vaccination against tuberculosis is
homogenization planned in a maternity hospital. Which
247. For prevention of what disease (from preparation of listed below must be used?
listed below) a vaccine of alive attenuated A. BCG vaccine
bacteria is used? B. APDT vaccine
A. Tetanus C. Tuberculin
B. Botulism D. ADТ vaccine
C. Cougher E. STI vaccine
D. Tuberculosis 253. The sputum of a patient with
E. Diphtheria tuberculosis was delivered to bacteriological
248. The pulmonary form of disease was laboratory. What staining method should be
revealed in anamnesis of a patient with used for microscopic examination of the
tuberculosis. Microscopic examination of smears for revealing tuberculosis
phlegm was carried out with the purpose of mycobacteria?
pathogen determination. What method of A. Ziehl-Neelsen
staining was used? B. Burri-Gins
A. Neisser C. Zdrodovsky
B. Gram D. Gram
C. Peshcov E. Romanovscy-Giemsa
D. Romanovscy-Giemsa 254. Centrifuge urine of a patient with
E. Ziehl-Neelsen renal tuberculosis suspected was used to
249. Hard phlegmonal infiltrate of dark blue- prepare a smear. What method of staining
purple color with numerous fistulas, which should be used for pathogen detecting? A
excrete a pus with unpleasant smell, was preparation for microscopy was prepared
revealed in a neck-jaw region of patient. For from centrifugate of urine allowance, which
confirmation of diagnosis “actinomycosis” was taken from a patient who is suspected on
during microscopic examination of pus the renal tuberculosis. Which method of filling
bacteriologist must reveal: the preparation is used for determining the
A. Druzes causative agent?
B. Gram-positive streptococci A. Loeffler's.
C. Gram-negative diplobacteria B. Burri.
D. Acid resistance bacilli C. Gram’s.
E. Gram-negative diplococci D. Ziehl-Neelsen's
250. A 16-year-old patient from the E. Ojeshko
countryside has a negative Mantoux test. 255. In the sputum smear stained by Ziehl-
What should the doctor do? Neelsen's method are revealed single or
A. Repeat the reaction in a month grouping red acid-fast rods. The first
B. To carry out tuberculosis serodiagnosis growth signs were appeared on nutrient
C. To quarant ine t he young man from media in 14 days. What species do the
the collective urgently microorganisms belong to?
D. To conduct the accelerated tuberculosis A. Mycobacterium tuberculosis
diagnostics by Price method B. Yersinia pseudotuberculosis

29
C. Histoplasma dubrosii E. Cord-factor
D. Klebsiella rhinoscleromatis 260. The medical examination of the 1-st
E. Coxiella burnettii form children included Mantoux. 15 children
256. Mycrobacteria tuberculosis (tbc) stained out of 35 had negative reaction. What action
by Ziehl-Neelsen were not revealed in the should be taken against children with the
smear from a phlegm of patient with negative Mantoux test?
tuberculosis. With the help of what methods A. Repeat the test
the probability of the bacterioscopic B. Inject anti-toxic serum
revealing of pathogen can be increased? C. Inject rabies vaccine
A. Enzyme assay D. Inject BCG vaccine
B. Biological method E. Examination of the blood serum
C. Inoculation of materials on the media of 261. Druses were revealed in a dark blue-
enrichment purple color phlegmonal infiltrate with
D. Methods of enrichment of material numerous fistulas of patient neck-jaw region
(homogenization and flotation) after Gram staining and microscopy. They
E. Serological methods were gram-positive in the center and gram-
257. Examination of the 36-yeared patient’s negative bulb-like. What disease causative
sediment was made. Previous diagnosis is agent is it?
kidney tuberculosis. Cord-factor was not A. Fusobacteriosis
revealed by the Price method during the B. Candidiasis
microscopy, but bacteria with resistance to C. Tularemia
acids were presented. What examination D. Brucellosis
would confirm or disprove the previous E. Actinomycosis
diagnosis? 262. The first form pupils went through a
A. Laboratory animals infection medical examination aimed at selection of
B. Toxigenicity study children needing tuberculosis revaccination.
C. Phagotyping of the isolated culture What test could be used?
D. Serological identification of the causative A. Mantoux
agent B. Sheek
E. Skin allergy test C. Skin test with tularin
258. A child underwent Mantoux test. 24 D. Burnett
hours after allergen injection there appeared a E. Anthraxin test
swelling, hyperaemia and tenderness. What 263. Mantoux test (with tuberculin) was made
are the main components of in the to a 10-year-old child. A papule (8 mm in
development of this reaction? diameter) appeared on the place of injection
A. Granulocytes, T-lymphocytes and Ig G in 48 hours. What type of hypersensitivity
B. Mononucleare leukocytes, T-lymphocytes reaction has developed after injection of
and lymphokines tuberculin?
C. Plasmatic cells, T-lymphocytes and A. Hypersensitivity reaction type IV
lymphokines B. Artuse phenomenon type reaction
D. B-lymphocytes, Ig M C. Serum disease type reaction
E. Macrophages, B-lymphocytes and D. Atopic reaction
monocytes E. Hypersensitivity reaction type II
259. Red rod-like bacteria formed twisted 264. The immunity to tuberculosis is
rodings were revealed by Price method preserved until there are live bacteria of
examination of patient’s sputum. What vaccine strain in the body after BCG
substance provides conglutination and wisp- vaccination of infants. Name this kind of
like growth of these bacteria? immunity.
A. RRD A. Non-sterile
B. Alttuberculin B. Humoral
C. Phosphatit (Phtyonic acid) C. Type specific
D. Tuberculostearic acid D. Innate

30
E. Crossed E. A. bovis
265. A patient is curing from chronic 270. The tubercular blebs were appeared on
pneumonia for a long time. Red, singly the pulmonary tuberculosis patient`s gum jaw
situated (sometimes in small clusters) length region. What method of staining was used for
about 0.25-0.4 mcm bacilli were revealed acid fast bacteria revealing?
during the microscopy of the sputum stained A. Neisser
by Ziehl-Neelsen. What disease does the B. Gram
patient have? C. Ziehl-Neelsen
A. Lung tuberculosis D. Romanovscy-Giemsa
B. Pneumococcal pneumonia E. Peshcov
C. Lung actinomycosis 271. It`s need to stain the slide with patient`s
D. Flue pneumonia sputum by Ziehl-Nilsen method for
E. Candidiasis “tuberculosis” diagnosis confirmation. The
266. Mantoux test was used for medical following reagents were prepared for this
examination of pupils. What specific factor purpose: carbolic fucsine and methylene blue.
the positive reaction cause? Which reagent is needed too?
A. T-lymphocytes A. 5% sulphuric acid
B. B-lymphocytes B. 3% hydrogen peroxide
C. Antibodies C. 70% ethyl alcohol
D. Erythrocytes D. Iodine solution
E. Leukocytes E. Vezuvin
267. A patient has diagnosis “leprosy” after 272. The patient’s sputum with suspicion on
complex examination. What skin-allergic test tuberculosis has been delivered to the
was used for the diagnostics? bacteriological laboratory. The smears need to
A. Coombs' test be stained by Ziehl-Nilsen. Ziehl`s fuchsine,
B. Molonnie’s test 5% solution of H2SO4 and methylene blue
C. Deeck’s test were used. Name the aim of sulphuric acid
D. Mittsude’s test using.
E. Shick’s test A. Neutralization of alkaline staining agent
268. The 44 yeared milkmaid with complaints B. Smear staining
to changes in the skin of the neck was C. Decolourisation of the acid sensitive
consulted by the doctor. The dense blue-claret bacteria
infiltrate fistula with unpleasant smell pus D. Increasing the perception of
was formed. The fibrous structure granular mycobacterium to staining agent
formations in diameter 20-50 microns were E. Dilution of the sputum
revealed in the pus during microscopy. What 273. The patient’s sputum with suspicion on
disease can be suspected? tuberculosis has been collected and delivered
A. Actinomycosis to the bacteriological laboratory for
B. Tularemia bacteriological method. When should these
C. Microsporia results be ready?
D. Anthrax A. In 3-4 weeks
E. Cryptococcosis B. In 3-4 months
269. Arranged in clusters like the pack of C. In one week
cigars bacillus have been revealed during D. In 2 days
microscopic examination of biopsy material E. On the next day
from the damaged area of oral cavity mucosa. 274. Calmet and Geren used media with
What type of causative agent was in biopsy unfavourable substance (bile) for vaccinal
material? strain of tubercular bacillus attenuation. What
A. Mycobacterium tuberculosis bacteria properties did they want to change in
B. A. israili this way?
C. Mycobacterium leprae A. Tinctorial
D. Mycobacterium avium B. Antigenic

31
C. Morphological D. Method of Romanowsky-Giemsa
D. Virulent E. Method of Neisser
E. Cultural 280. Tuberculosis can be treated by means of
275. Slide stained by Ziehl-Neelsen was combined chemotherapy that includes
prepared from a patient sputum with substances with different mechanisms of
suspiction on tuberculosis. What properties of action.What antituberculous medication
the causative agent will be important for its inhibits transcription of RNA into DNA in
identification? mycobacteria?
A. Toxigenicity and immunogenicity A. Streptomycin
B. Cultural and enzymatic B. Rifampicin
C. Pathogenic and virulence C. Isoniazid
D. Biological and antigenic D. Ethionamide
E. Morphological and tinctorial E. Para-aminosalicylic acid
276. The patient’s sputum with tuberculosis 281. At the study of phlegm, taken from a
was sent to a laboratory. Ziehl-Nilsen method patient with suspicon on tuberculosis,
was used. For this purpose carbolic fucsine, preparation was made and stained by Ziehl-
5% H2SO4 and methylene blue were used. Neelsen. What microscopic picture is seen at
What method was used? confirmation of the supposed diagnosis?
A. Ziehl-Neelsen A. Red bacilli on a green background
B. Gins-Burri B. Microorganisms with the kernel of ruby-
C. Gram red colour and blue cytoplasm
D. Peshkov C. Red bacteria on a white background
E. Neisser D. Chain like bacilli of violet colour
277. The proper sequence of staining by E. Thin red bacteria on a blue background
Ziehl-Neelsen`s method are: 282. For prevention of which from the
A. Crystal violet, Alcohol, Safranin enumareted diseases is a vaccine from
B. Ziehl`s Fuxin, H2SO4, Water, Methylene living attenuated bacteria are used?
blue A. Tetanus
C. Methylene blue, H2SO4, Ziehl`s Fuxin B. Sausage-poisoning
D. Ziehl`s Fuxin, Water, Methylene blue, C. Whooping-cough
Alcohol D. Tuberculosis
E. Ziehl`s Fuxin, H2SO4, Methylene blue, E. Diphtheria
Water 283. The pulmonary form of disease was
278. While enrolling a child to school revealed in anamnesis of a patient with
Mantoux's test was made to define whether tuberculosis. Microscopic research of phlegm
revaccination was needed. The test result is was carried out with the purpose of
negative. What does this test result mean? determination of pathogen. What method of
А.Absence of antitoxic immunity to the staining was used?
tuberculosis A. Neisser
В. Presence of antibodies for tubercle bacillus B. Gram
С. Presence of cell immunity to the C. Peshcov
tuberculosis D. Romanovscy-Giemsa
D. Absence of cell immunity to the E. Ziehl-Neelsen
tuberculosis 284. In a sick man hard phlegmonal infiltrate
E. Absence of antibodies for tubercle bacillus of dark blue-purple color with numerous
279. A consumptive patient has an open fistulas, which excrete a pus with unpleasant
pulmonary form of disease. Choose what smell, was revealed in a neck-jaw region. For
sputum staining should be selected for finding confirmation of diagnosis “actinomycosis”
out the tubercle (Koch's) bacillus? during microscopic research of pus the
A. Method of Ziehl-Neelsen bacteriologist must reveal:
B. Method of Burry-Gins A. Druzes
C. Method of Gram B. Gram-positive streptococci

32
C. Gram-negative diplobacteria singularly. When inoculated onto the
D. Acid resistance bacilli nutrient media, the signs of their growth show
E. Gram-negative diplococci up on the 10-15 day. These bacteria relate to
285. Specimen of a patient`s sputum was the following family:
stained with the following dyes and reagents A Yersinia pseudotuberculosis
Ziehl`s solution, methylene blue solution, 5% B Micobacterium tuberculosis
solution of sulphuric acid. What staining C Histoplasma dubrosii
method was applied? D Klebsiella rhinoscleromatis
A. Peshkov`s E Coxiella burnettii
B. Burri`s 290. A 15 y.o. boy from a countryside entered
C. Ziehl-Neelsen an educational establishment. Scheduled
D. Gram`s Manteux test revealed that the boy had
E. Niesser`s negative reaction. What are the most
286. Microscopical examination of an reasonable actions in this case?
infiltrate removed from the submandibular A To repeat the reaction in a month
skin area in a 30 y.o. man revealed foci of B To perform BCG vaccination
purulent fluxing surrounded by maturing C To perform serodiagnostics of tuberculosis
granulations and mature connective tissue, the D To isolate the boy temporarily from his
pus contains druses consisting of multiple mates
short rod-like elements with one end attached E To perform rapid Price diagnostics
to the homogenous centre. What disease is it? 291. Microscopy of stained (Ziehl-Neelsen
A. Angina staining) smears taken from the sputum of a
B. Syphilis patient with chronic pulmonary disease
C. Candidosis revealed red bacilli. What property of
D. Actinomycosis tuberculous bacillus was shown up?
E. Tuberculosis A Alcohol resistance
287. A consumptive patient has an open B Alkali resistance
pulmonary form of disease. Choose what C Acid resistance
sputum staining should be selected for finding D Capsule formation
out the tubercle (Koch's) bacillus? E Sporification
A Method of Gram 292. The first grade pupils were examined in
B Method of Romanowsky-Giemsa order to sort out children for tuberculosis
C Method of Ziel-Neelsen revaccination. What test was applied for this
D Method of Neisser purpose?
E Method of Burry-Gins A Schick test
288. While registering the child to the B Mantoux test
school Mantu's test was made to define C Supracutaneous tularin test
whether revaccination was needed test result D Burnet test
is negative. What does this result of the test E Anthraxine test
mean? 293. A 10-year-old child had the mantoux
A Absence of antitoxic immunity to the tuberculin test administered. 48 hours later a
tuberculosis papule up to 8 mm in diameter appeared on
B Presence of cell immunity to the the site of the injection. What type of
tuberculosis hypersensitivity reaction developed after the
C Absence of antibodies for tubercle bacillus tuberculin injection?
D Absence of cell immunity to the A Type II hypersensitivity reaction
tuberculosis B Arthus phenomenon
E Presence of antibodies for tubercle bacillus C Seroreaction
289. Study of bacteriological sputum D Atopic reaction
specimens stained by the Ziel-Neelsen E Type IV hypersensitivity reaction
method revealed some bright-red acid- 294. Planned mass vaccination of all newborn
resistant bacilli that were found in groups or 5-7 day old children against tuberulosis plays

33
an important role in tuberculosis prevention. 298. While examining a patient an
In this case the following vaccine is applied: otolaryngologist noticed hyperaemia and
A TAB significantly edematous tonsils with a grayish
B Diphteria and tetanus toxoids and pertussis film upon them. Microscopical examination
vaccine of this film revealed some gram-positive
C Diphtheria and tetanus anatoxin vaccine bacilli placed at an angle with each other.
D Adsorbed diphtheria vaccine What disease might be suspected?
E BCG A Diphtheria
B Angina
Microbiological diagnostics of diphtheria. C Scarlet fever
Microbiological diagnostics of diseases D Meningococcal nasopharyngitis
caused by bordetellas E Epidemic parotitis
295. A patient with suspected diphtheria went 299. From pharynx of a child with
through bacterioscopic examination. suspected diphtheria a pure culture of
Examination of throat swab revealed rod- microorganisms was
shaped bacteria with volutin granules. What isolated. Their morphological, tinctorial,
etiotropic preparation should be chosen in this cultural and biochemical properties appeared
case? to be typical for diphtheria causative agents.
A. Interferon What study should be conducted in order to
B. Bacteriophage drow a conclusion that this is a pathogenic
C. Diphtheria antitoxin diphtheria bacillus?
D. Antidiphtheric antitoxic serum A Estimation of toxigenic properties
E. Eubiotic B Estimation of proteolytic properties
296. Microorganism which is identical to C Estimation of urease activity
Corynebacterium diphtheriae according to D Estimation of cystinous activity
morphological and biochemical signs was E Estimation of ability to decompose starch
isolated from the nasopharynx of a 5-year-old 300. Inoculum from pharynx of a patient ill
child. But this microorganism did not produce with angina was inoculated into blood-
exotoxin. As a result of what process can this tellurite agar. It resulted in growth of grey,
microorganism become toxigenic? radially striated (in form of rosettes) colonies
A. Passing through the organism of the 4-5 mm in diameter. Gram-positive bacilli
sensitive animals with clublike thickenings on their ends placed
B. Cultivation in the telluric environment in form of spread wide apart fingers are
C. Phage conversion visible by microscope. What microorganisms
D. Chromosome mutation are these?
E. Growing with antitoxic serum A Diphtheria corynebacteria
297. In order to estimate toxigenity of B Botulism clostridia
diphtheria agents obtained from patients the C Diphtheroids
cultures were inoculated on Petri dish with D Streptococci
nutrient agar on either side of a filter paper E Streptobacilli
strip that was put into the centre and 301. A sample taken from the pharynx of a
moistened with antidiphtheric antitoxic patient with angina was inoculated on the
serum. After incubation of inoculations in blood-tellurite agar. This resulted in growth of
agar the strip-like areas of medium turbidity grey, radially striated (in form of rosettes)
were found between separate cultures and the colonies up to 4-5 mm in diameter.
strip of filter paper. What immunological Microscopically there can be seen gram-
reaction was conducted? positive rods with club-shaped ends arranged
A Precipitation gel reaction in form of spread fingers. What
B Coomb's test microorganisms are these?
C Agglutination reaction A. Diphtheroids
D Rings precipitation reaction B. Streptococci
E Opsonization reaction C. Corinebacteria diphtheriae

34
D. Clostridium botulinium D. Cystine activity
E. Streptobacilli E. The starch fermentation
302. To determine the toxigenicity of 306. For diphtheria causative agent isolation
diphtheria causative agents isolated from the material must be inoculated on such
patients, cultures are inoculated onto a media:
Petri dish with nutrient agar on both sides A. Ploskirev
of the centrally located filter paper strip B. Roux, Loeffler, blood-tellurite agar
sodden with antidiphtheric antitoxic serum. C. Levin (Endo)
What must be revealed after cultures D. Kitt-Tarocci
incubation in the agar between separate E. Hiss
cultures and filter paper strip? 307. A specific pathogenetic treatment is
A. Merging precipitation lines recommended for a patient with diphtheria.
B. Crossing precipitation lines Name this medication.
C. Zones of diffusion opacification A. Antibiotics
D. Precipitation lines absence B. Bacteria-phages
E. Precipitation ring C. Anatoxin
303. In order to estimate toxigenicity of D. Anti-toxic serum
diphtheria agents obtained from patient the E. Sulfanilamides
cultures were inoculated on Petri dish with 308. Among the children of a boarding
nutrient agar on either side of a filter paper school there are cases of angina.
strip that was put into the centre and Microscopy of tonsil smears stained by
moistened with antidiphtheric antitoxic Neisser method revealed yellow rods with
serum. After incubation of inoculation in agar dark-brown terminal granules arranged in the
the strip-like areas of medium turbidity were form of V, W, and X letters. What infection
found between separate cultures and the strip can be suspected in this case?
of filter paper. What immunological reaction A. Infectious mononucleosis.
was conducted? B. Diphtheria.
A. Agglutination C. Listeriosis.
B. Coomb`s test D. Tonsillitis.
C. Precipitation in gel E. Scarlet fever.
D. Ring-precipitation 309. What localization of diphtheria is mostly
E. Opsonization wide-spread now?
304. The patients usually receive accurately- A. Membranous nasal diphtheria
calculated doses of antitoxic serum. In what B. Diphtheritic conjunctivitis
activity units the antitoxic antidiphtherial C. Ear diphtheria
serum is defined? D. Faucial diphtheria
A. Flocculation units E. Surgical diphtheria
B. International units 310. Anti-toxic serum is used for treatment
C. Lethal units of:
D. Bacteriostatic units A. Gonorrhea
E. Hemolytic B. Cougher
305. The pure culture of microorganisms was C. Dysentery
isolated from a child with suspicion on D. Diphtheria
diphtheria. The morphological, tinctorial, E. Tuberculosis
cultural and biochemical properties which 311. The toxoid was injected to 20-year-old
were typical for diphtheria agent were man. For what disease prevention it was
studied. What examination must be made for done?
the pathogenicity of the diphtheria bacillus A. Tuberculosis
revealing? B. Diphtheria
A. Urease activity C. Scarlet fever
B. Proteolytic properties D. Meningitis
C. Toxigenicity properties E. Cougher

35
312. The 5-yeared boy was vaccinated E. Ozheshko
according to plan and get an injection of 317. A dentist has revealed the grey films on
1/40 Dlm diphtheria toxin in his forearm tonsils of the examined child with suspected
during diphtheria epidemy. There was no atypical form of diphtheria. Slide and
reaction after 24-48 hours in the place of inoculation on nutrient mediums were made
injection and doctor, who cured the child, and toxigenicity of the isolated pure culture
was satisfied of this result. What was the was established. What test was used for
purpose of this test? diphtheria bacillus toxigenicity study?
A. Revaccination A. Hemolysis
B. Anti-toxic immunity estimation B. Agglutination
C. Skin allergic test C. Complement fixation test
D. Desensitization D. Precipitation in gel
E. Bacteria carrier’s revealing E. Ring precipitation
313. Vaccine for diphtheria prevention and 318. A child with diphtheria diagnosis was
antitoxic immunity providing consist of: delivered into a hospital. What medication for
A. Diphtheria toxoid specific therapy you will use?
B. Small dose of diphtheria toxin A. Anti-diphtheria anti-toxic serum
C. Diphtheria toxin and anti-diphtheria serum B. Diphtheria anatoxin, antibiotic
together C. The “Codivak” vaccine, sulfanilamide
D. Alive weakened diphtheria causative D. Diphtheria vaccines: APDT, ADT, AD
agents E. Diphtheria bacteria-phages
E. Killed diphtheria bacillus 319. The otolaryngologist has revealed
314. A 5 yeared girl has high temperature and hyperemia, edema and the grey films on
sore throat. Objectively: soft palate edema, tonsils of the examined patient. Gram-positive
tonsils are covered with grey films that can be arranged angularly to each other bacilli have
hardly removed and leave deep bleeding been revealed at microscopy. What disease is
tissue injuries. What disease is the most it?
probable? A. Meningonasopharengitis
A. Lacunar tonsilitis B. Angina
B. Pharingeal diphtheria C. Scarlet fever
C. Infectious mononucleosis D. Diphtheria
D. Vincent`s preudomembranous tonsilitis E. Epidemic parotitis
E. Necrotic tonsilitis 320. A group of students must be injected for
315. In the smear from the tonsils of a diphtheria prevention. What preparation must
patient with diphtheria suspected there have be used for creation of the artificial active
been revealed blue rods with thickenings immunity?
on the ends. What method of staining A. APDT vaccine
has been used? B. Anti-diphtheria serum
A. Loeffler's. C. Specific immunoglobulin
B. Ziehl-Neelsen. D. Diphtheria anatoxin
C. Gins'. E. Vaccine with inactivated bacteria
D. Gram's. 321. Antidiphtherial immunity must be
E. Neisser's. examined for necessity of collective
316. Bacillus with thickenings on the ends vaccination. What examination must be done
similar to C.diphtheria stained with in this case?
methylene-blue were revealed during A. Establish the antitoxins title in IHAT
microscopy. What method must be used for B. Check all members for diphtheria bacillus
confirming the suggestion additionally? carrying
A. Neisser C. Establish an antibody level against
B. Kozlovskiy diphtheria
C. Ziehl-Neelsen D. Check all medical documentation about
D. Zdrodovsky vaccination

36
E. Check the immune status against diphtheria 327. An antitoxic antidiphtherial serum was
bacillus inoculated to a child with diphtheria. A rash
322. A passive hemagglutination test was on a skin, an itch, turgidities and a pain in
used for level of the anti-diphtheria immunity joints were appeared in 10 days after injection
of child determining. What must erythrocytes and protein was revealed in urine. What are
should be sensitized by to solve the task? the reasons of these phenomena?
A. Hemolytic serum A. Serum disease
B. Diphtheria anti-toxin B. Anaphylactic reaction
C. Antigens of diphtheria bacillus C. Atopy
D. Anti-diphtheria serum D. Hypersensitivity of the delayed type
E. Diphtheria toxoid E. Contact allergy
323. A boy has received the diagnosis 328. Examining a 6-year-old child a doctor
“diphtheria” in the children department of has noticed on pharyngeal tonsils a grayish
infectious hospital. What preparation should "pseudo-membrane". Attempt to remove it
be injected first? causes moderate bleeding. Bacterioscopy of
A. Anti-diphtheria anti-toxic serum tonsils swabs has shown Gram-positive
B. Diphtheria anatoxin club-shaped bacteria. What symptoms can
C. APDT appear within the next few days without
D. ADP specific treatment?
E. TABte A. Papular rash on skin
324. Dark blue bacilli with thickenings on B. Pneumonia (Lungs edema)
poles were revealed in the dab made from C. Very strong attack-like cough
tonsills incrustation of patient with suspicion D. Toxic affection of the cardiac muscle,
on diphtheria. What smear staining method liver, kidneys
was used? E. Intermittent fever
A. Loeffer 329. The microorganisms which is identical to
B. Burri Corynebacterium diphtheriae according to
C. Gins morphological and biochemical signs were
D. Gram isolated from the tonsills of a two-year-old
E. Neisser child without planned APDT injection. Gel
325. The doctor has noticed a grey-yellow precipitation reaction with anti-toxic serum
films on the examined child tonsils that can had shown negative result. What form of the
be hardly removed. Previous diagnosis is infectious process can this agent cause in a
“diphtheria?” has made considering child’s organism if the treatment will not start
complaints of the patient to pains during at the moment?
swallowing and high temperature. With the A. Unsymptomal carrying of the bacteria
help of what method this diagnosis can be B. Light nontoxic disease type
confirmed or disproved? C. Toxic and complicated disease type
A. Biological D. Persist infection
B. Serological 330. Rod-like bacillus with volutin granules
C. Bacteriological were revealed during the patient’s smear
D. Allergic test process examination (patient is suspected on
E. Microscopic diphtheria). What ethiotropic preparation
326. A pure culture of C.diphtheria was must be chosen in this case?
isolated from the patient. What immunologic A. Bacteria-phage
test should be used for toxigenicity of the B. Anti-diphtheria anti-toxic serum
bacteria study? C. Diphtheria anatoxin
A. Complement fixation test D. Eubiotic
B. Agglutination E. Interferon
C. Precipitation in agar 331. The doctor has suspected a diphtheria in
D. Hemagglutination inhibition test a sick child who had a rise in temperature of a
E. Indirect hemagglutination test body to 38 degrees, a pain in a throat at

37
swallowing, a face turgidity, adynamy and 336. The microorganisms, which
grey-white films on tonsils,. What morphological and biochemical properties
microbiological methods can confirm the looked like diphtheria bacillus were revealed
previous diagnosis? at bacteriological examination of a material
A. Microscopic and allergy test from an oral cavity of the 5 yeared boy which
B. Microscopic + bacteriological in 5 months have made a planned an APDT
C. Microscopic + serological injection and revaccination when he was 2
D. Allergic + serological years. How can the causative agent’s presence
E. Biological + serological inside the immunized child be explained?
332. From a nasopharynx of the child with A. Only anti-toxic anti-diphtheria immunity
suspicion on a diphtheria the pure culture of was formed
microorganisms is allocated with B. The APDT vaccine creates unsterile
morphological, tinctorial, cultural and immunity
biochemical properties which are typical for C. After APDT injecting immunity lasts not
all diphtheria causative agents were revealed. more than for 2 years
But isolated culture was not toxic. As a result D. Child is suffering from immunodeficiency
of what process this culture can become E. Isolated bacteria is a nonpathogenic
toxic? diphtheroid
A. Phage conversion 337. A patient has intoxication, headache,
B. Cultivation on the tellurite media hyperemia and a pharynx swelling. There are
C. Putting through the organisms of sensible grey films on tonsils which may be hardly
animals deleted. The diagnosis is Diphtheria. What is
D. Fermentation in anti-toxic serum presence the main factor of pathogenicity of the
E. Chromosome mutation causative agent identifies the described
333. Doctor has suspected diphtheria in the symptoms of the disease?
sick child. A bacteriological examination was A. Neuraminidase
completed and this diagnosis was confirmed. B. Intracellular toxin
What nourishing media is used for C. Hyaluronidase
C.diphtheria? D. Exotoxin
A. MPB E. Protease
B. Endo 338. A 20-year-old man was injected with
C. Ploskirev anatoxin in purpose of prophylactics. What
D. MPA disease was the injection against?
E. Roux A. Meningitis
334. A pat ient was admit t ed to a clinic B. Tuberculosis
in grave condition, fever, heavy breathing. C. Scarlet fever
Preliminary diagnosis of diphtheria croup was D. Diphtheria
made by the microscopy of throat specimen. E. Cougher
What method of staining was used? 339. A 7 y.o. girl was admitted to the
A. Peshkov's. infectious diseases hospital with fever, sore
B. Ziehl-Neelsen's. throat, common weaknees. A doctor
C. Burri-Gins'. suspected diphtheria. What (from listed
D. Neisser's. below) is decisive for the diagnosis
E. Ozheshko's. confirming after the pure culture of causative
335. A specific prophylactics of diphtheria agent isolation?
must be made in a kindergarten. Which A. Phagolysability
preparation must be used for this? B. Cystinase test
A. Anatoxin C. Toxicity test
B. Antibiotics D. Detection of volutin granules
C. Probiotics E. Hemolytic ability of the causative agent
D. Corpuscular vaccine 340. A pure culture from a pharynx of the
E. Immune serum sick 9 years old child with suspicion on

38
diphtheria was isolated. It was identified as C. A. Spores
diphtheriae, nontoxic strain. APDT B. Plasmids
vaccination was done earlier accordingly to a C. Volutin granules
calendar of injections. What is the most D. Capsule
possible reason for the child’s anti-bacterial E. Flagella
anti-diphtherial immunity absence? 345. In order to determine toxigenicity of
A. A postvaccinal immunity is anti-toxic diphtheria bacilli a strip of filter paper
B. Child has primary immunodeficiency impregnated with antitoxic diphtherial
C. Child has secondary immunodeficiency serum was put on the dense nutrient medium.
D. Child has phagocytic immunodeficiency There were also inoculated a microbal culture
E. Revaccination was made by off-grade under examination and a strain that is known
vaccine to be toxigenic. If the microbal culture under
341. A toxic culture of causative agent was examination produces exotoxin, this wil result
isolated from a patient C. with the previous in formation of:
diagnosis – diphtheria. It is known, that this A Haemolysis zones
activity is connected with diphtheria bacteria B Precipitin lines
lysogenicity. What factor determines the C Zones of diffuse opacification
lysogenicity of bacteria? D Zones of lecithovitellinous activity
A. Antibodies E Precipitin ring
B. Antibiotics 346. In order to estimate toxigenity of
C. Phagocytes diphtheria agents obtained from patients the
D. Factors of self-lysis cultures were inoculated on Petri dish with
E. Temporal phages nutrient agar on either side of a filter paper
342. A fauces smear was taken from patient strip that was put into the centre and
K. and microslide was stained by Neisser. moistened with antidiphtheric antitoxic
Which structure components of the diphtheria serum. After incubation of inoculations in
causative agents can be discovered during the agar the strip-like areas of medium turbidity
microscopy of the preparation? were found between separate cultures and the
A. Inclusions strip of filter paper. What immunological
B. Sarments reaction was conducted?
C. Capsules A Agglutination reaction
D. Pili B Coomb's test
E. Spores C Precipitation gel reaction
343. C. diphteriae was isolated in the teacher D Rings precipitation reaction
during inspection of bacteria carriers in the E Opsonization reaction
children's institutions. C. diphteriae strain did 347. Inoculum from pharynx of a patient ill
not produce exotoxin. What reaction was used with angina was inoculated into blood-
for toxigenicity diphtheria bacillus tellurite agar. It resulted in growth of grey,
examination? radially striated (in form of rosettes) colonies
A. Precipitation in agar gel 4-5 mm in diameter. Gram-positive bacilli
B. Ring precipitation reaction with clublike thickenings on their ends placed
C. Immunofluorescence reaction in form of spread wide apart fingers are
D. Complement fixation reaction visible by microscope. What microorganisms
E. Agglutination reaction are these?
344. A child is presumably ill with diphtheria. A Diphtheria corynebacteria
A specimen of affected mucous membrane of B Botulism clostridia
his pharynx was taken for analysis. The smear C Diphtheroids
was stained and microscopical examination D Streptococci
revealed yellow rods with dark blue E Streptobacilli
thickening on their ends. What structural 348. In order to establish the level of anti-
element of a germ cell was revealed in the diphtheria immunity in a child it was decided
detected microorganisms? to use a passive hemagglutination test. This

39
task should be completed by the A. Dissociation
sensibilization of erythrocytes by: B. Transformation
A. Diphtheria antitoxin C. Phage conversion
B. Diphtheria anatoxin D. Adaptable change ability
C. Antigens of diphtheria bacillus E. Mutation
D. Anti-diphtheria serum 353. A patient had high temperature and
E. Hemolytic serum cough attacks for 10 days. Doctor proposed
349. In purpose of active prophylactics of the inoculation of material from rhinopharynx
diphtheria, tetanus and cougher an APDT onto CCA media. What microorganism is
vaccine is used. What does this vaccine supposed to be isolated?
include and what protects organism from A. Influenza bacillus
cougher? B. Cougher bacillus
A. Attenuated Bordetella petrussis C. Listeriosis
B. Cougher exotoxin D. Staphylococcus
C. Inactivated Bordetella petrussis E. Klebsiella
D. Cougher intracellular toxin 354. A 4-year-old child has clinical symptoms
E. Anatoxin of cougher. For serological diagnostics of the
350. A 5-year-old boy was delivered into a whooping cough it was made large-scale
city infectious hospital with the previous reaction with parapertussis and pertussis
diagnosis “cougher”. Sputum was taken for a diagnosticums. At the bottom of the test-tubes
bacteriological examination. Which with diagnosticum of Bordetella pertussis
nourishing media are used for Bordetellas grain-like sediment formed. What antibodies
cultivating? have this reaction revealed?
A. Serum agar A. Agglutinins
B. Levin media, Endo media B. Precipitin
C. MPB C. Bacteriotropins
D. Casein-charcoal agar D. Bacteriolysins
E. Tumanskiy media E. Anti-toxins
351. A 5-year-old child is having a cougher 355. There is a vaccination against the
clinic symptoms. The pathological material cougher planned in a kindergarten. What
(scrape from pharynx) was inoculated on preparation is used for this?
sugar meat-pepton agar for this diagnosis A. Type-specific serum
confirming. No signs of growth appeared in B. BCG vaccine
one day. Mark the most possible reason for C. APDT vaccine
this. D. Normal (common) gamma-globulin
A. Wrong pathologic material E. ADP anatoxin
B. Wrong nourishing media 356. A 6-year-old child has clinical symptoms
C. A long period of time from taking the of cougher (convulsive period). What method
material to seeding of microbiological diagnostics can prove to be
D. Colonies of cougher causative agent grow the most effective and be used?
only on the 4-th day A. Serological
E. This disease is not a cougher, it is a virus B. Bacteriological
respiratory infection C. Biological
352. The cougher causative agents cultures D. Allergic
were isolated from patients. They gave E. Microscopic
colonies of different types: I phase – smooth, 357. There is an infective cougher episode in
consist of virulent capsulated the kindergarten. On the background of
microorganisms, IV phase – rough, consist of episode a 4-year-old child got catarrhal
avirulent non-capsulated microorganisms, II phenomenon. Child is sick since second day.
and III phases have average properties. What What microbiological method is most
term characterizes this phenomenon most possibly able to be used for confirming the
accurately? “cougher” diagnosis in this case?

40
A. Allergic was delivered into the surgical department.
B. Serological The etiology was not revealed. What specific
C. Microscopic treatment should be used for this patient?
D. Biological A. To use the polyvalent antitoxic antigangren
E. Bacteriological serum
358. For serological diagnostics of the B. To make surgical treatment of wound
whooping cough it was made large-scale C. To prescribe the high doses of sulfanilamid
reaction with parapertussis and pertussis preparations
diagnosticums. At the bottom of the test-tubes D. To make vaccination
with diagnosticum of Bordetella E. To prescribe the high doses of antibiotics
parapertussis grain-like sediment formed. 363. During the patient’s examination of the
What antibodies have this reaction revealed? jaw-facial necrotic phlegmone a doctor
A Precipitins suspected a gas gangrene. The rod shape
B Agglutinins grampositive microorganisms were revealed
C Opsonins during microscopy of wound’s festering
D Bacteriolysins allocation. What media is used for the clean
E Antitoxins culture isolation of the infectious agent?
359. A patient has been suffering from A. Media Kitt-Tarozzi
elevated temperature and attacks of typical B. Media Endo
cough for 10 days. Doctor administered C. Media Levin
inoculation of mucus from the patient's D. Meat-pepton agar
nasopharynx on the agar. What E. Milk-salt agar
microorganism is presumed? 364. The case of anaerobic infection after the
A Pertussis bacillus planned surgical operation took place in a
B Pfeiffer's bacillus hospital. What material should be sent for
C Listeria bacteriological examination for establishment
D Klebsiella of this case?
E Staphylococcus A. Blood
360. In a kindergarten vaccination against B. Urine
pertussis is planned. Which preparation is used C. Bandaging, stitch material (silk, catgut)
for the immunization? D. Pieces of staggered tissue
A. Type specific serum. E. Tissue liquid
B. BCG vaccine. 365. The microorganisms that grew on
C. APDT vaccine. glucose-blood agar in anaerobic conditions
D. Normal γ-globulin. for 7-10 days, were isolated during
E. ADT vaccine. bacteriological examination from the pus
operation wound. They were S-form black
brilliant colonies with an unpleasant smell.
Microbiological diagnostics of anaerobic Polymorphic gramnegative rods were
infection. revealed during the microscopy. What
361. A doctor suspected possible microorganisms could cause this process?
development of wound’s anaerobic infection A. Bacteroides
in a patient suffered in accident. What B. Clostridia
preparation was most expediently to apply for C. Fuzobacteria
specific treatment before the establishment of D. Veilonella
laboratory diagnosis? E. Colon bacilla
A. Bacteriophage 366. The gas anaerobic infection of the left
B. Polivalent specific serum lower extremity was suspected at 39 yeared
C. Typospeсific immune serum mechanization expert. Clostridium
D. Native plasma perfringens was isolated from a wound. The
E. Placental gammaglobulin determination of what enzyme has the
362. A patient with the shin’s gas gangrene deciding value during the identification?

41
A. Lecitinase C Microbiological diagnostics of tetanus and
B. Dezoxyribonuclease botulism
C. Collagenase 372. Microscopical examination of a microbal
D. Proteinase culture revealed fusiform spore-forming
E. Hyaluronidase microorganisms that get violet-blue Gram's
367. A patient with the anaerobic infection of stain. What microorganisms were revealed?
lower extremity in the defeat area has the A Clostridia
edema, gasification, necrosis of tissues, B Streptococci
general intoxication. What factors of C Spirochaete
pathogenicity were absent in the infectious D Actinomycete
agent and do not take part in development of E Diplococci
pathology? 373. A specimen stained by Ozheshko
A. Endotoxin method contains rod-like microorganisms
B. Patogenic enzimes stained blue with round terminal components
C. Hemotoxin stained red. What are these components
D. Necrotoxin called?
E. Neurotoxin A Spores
368. A patient, who had been injured in car B Cilia
accident, was suspected by the doctor to be C Flagella
suffering from a possible wound anaerobic D Capsules
infection. What kind of medication for E Mesosomas
specific treatment should be applied prior to 374. Patient with vomiting, dizziness,
laboratory diagnosis determination? sensation of dubble vision, difficult
A. Polyvalent specific serum swallowing was admitted to the hospital.
B. Bacteriophage Doctor suspects botulism. What diagnostic
C. Typespecific immune serum methods should be used for diagnosis
D. Native plasma approving?
E. Placenta hammagloboulinum A Allergic test, bacteriological
369. Antitoxic sera is used for treatment of: B Allergic test, serological
A. Tuberculosis C Bacteriological, mycological
B. Brucellosis D Protozoological, microscopical
C. Gonorrhea E Biological test, bacteriological
D. Dysentery 375. Bacteriological laboratory examines
E. Gas gangrene canned meat whether it contains botulinum
370. Vaccine is used for the prophylaxis of gas toxin. For this purpose an extract of test
anaerobic infection. It consists of: specimen and antitoxic antibotulinic serum of
A. Anatoxin A, B, E types were introducted to a group of
B. Small dose of toxin mice under examination; a control group of
C. Sera mice got the extract without antibotulinic
D. Living gas anaerobic infection agent serum. What serological reaction was
E. Killed gas anaerobic infection agent applied?
371. A laboratory received a material from a A Neutralization
patient's wound. Preliminary diagnosis is B Precipitation
gaseous gangrene. What microbiological C Complement binding
method should be applied to determine D Opsono-phagocytic
species of causative agent? E Double immune diffusion
A Allergic 376. What group of the vaccine, applied for
B Bacteriological the tetanus prevention, this preparation does
C Bacterioscopic belong to?
D Serological A. Inactivated vaccine
E RIA B. Attenuated vaccine
C. Chemical vaccine

42
D. Anatoxin 382. For the victim in a motor-car accident
E. Genoingeneering vaccine with the lower jaw trauma the antitetanus
377. After canned food use a sick man N had serum was inoculated urgently to a patient.
diplopia, headache, dysphagia, infregements However 2 months later a patient was
of breathing, muscular weakness. A delivered into infectious department with the
diagnosis is botulism. What factor of symptoms of ‘later’ tetanus. How would it be
pathogenicity did these clinical displays of the correctly to do the prevention of tetanus to
disease connected with? avoid the noted complications?
A. Plasmacoagulase A. To use antitetanus human gamagloboulin
B. Agressines B. To the large dose of antitoxic serum
C. Endotoxin C. To conduct the active-passive prevention
D. Exotoxin of tetanus
E. Fibrinolysine D. Blood transfusion
378. The symptoms of bulbar paralysis E. To do autohemotherapy
appeared in a patient after canned mushrooms 383. A jerked fish of domestic preparation
meal: ptosis, diplopia, aphonia and dysphagia. which caused serious food poisoning was
Previous diagnosis was botulism.What examined in a bacteriological laboratory.
reaction helps to determine the type of toxin? Microscopy of the culture isolated on
A. Reaction of agglutination Kitt-Tarozzi medium revealed
B. Reaction of neutralization microorganisms akin to a tennis-racket.
C. Reaction of precipitation What diagnosis is possible?
D. Reaction of complement fastening A. Tetanus
E. Reaction of imunofluoresсent B. Salmonellosis
379. The diagnostics of tetanus were made in C. Cholera
a laboratory. What method of sterilization can D. Dysenteria
we use for the killing of the agent of tetanus? E. Botulism
A. Pasterization 384. A patient N. was delivered to the
B. Boiling hospital with complaints on vomiting,
C. Tindalization doubling in eyes, headache, violation of
D. Dry heat swallowing, hard breathing, muscular
E. Autoclaving weakness. A doctor suspected botulism. What
380. At a student C. appeared the symptoms methods of diagnostics should be used for
of the food poisoning after the canned meat confirmation of diagnosis?
meal in domestic terms: diplopia, violation of A. Biological test, bacteriological
language and breathing paralysis. What such B. Allergic test, serologic
symptoms of botulism are conditioned by? C. Bacteriological, mycologic
A. By neurotoxin D. Protozoological, microscopic
B. By histotoxin E. Bacteriological, serologic
C. Secretion of enterotoxin 385. During patient’s examination a dentist
D. Enterotoxic shock marked a tonic abbreviation of masticatory
E. Activation of adenilatacyclise muscles and opening of mouth was limited.
381. A sick man is hospitalized with What infectious disease has such symptoms?
complaining on vomiting, dizziness, doubling A. Diphtheria
in eyes, violation of swallowing. A doctor B. Flu
suspected botulism. What methods of C. Tetanus
diagnostics should be expediently used for D. Leptospirosis
confirmation of diagnosis? E. Cholera
A. Protozoologic, microscopic 386. A patient with the opened break of lower
B. Allergic test, serologic jaw was delivered into the stomatology-
C. Biologic test, bacteriological surgical department. What preparation should
D. Bacteriological, serologic be used with the purpose of active
E. Bacteriological, micologic immunization against tetanus?

43
A. Tetanus anatoxin A. Serum illness
B. Antitetanus immunogloboulin B. Hives
C. Antitetanus serum C. Anaphilactic shock
D. APDT D. Disbacteriosis
E. Antitetanus vaccine E. Quink`s edema
387. A patient was delivered to a hospital 391. The grampositive rods with the terminal
with a previous diagnosis «botulism». What located spore that had the appearance of
serologic reaction should be used for the «drumstick» were isolated during the
botulin toxin’s revealing in the examined bacteriological examination of stitch material.
material? What type of bacteria has such morphological
A. Reaction of precipitation description?
B. Reaction of agglutination A. C.tetani
C. Reaction of complement binding B. C.botulinum
D. Reaction of neutralization C. B.anthracis
E. Reaction of immunofluorescent D. C.perfringens
388.7 days later after the plastic operation E. Y.pestis
made by a dentist, a tetanus appeared at a 392. The infectious agent of tetanus
patient. There was suspicion, that a reason produces exotoxin with different biological
was contaminated by the tetanus infectious action effects. What clinical symptoms can
agent stitch material which delivered in a this toxin cause?
bacteriological laboratory for examination. A. Disorders of vision
What nourishing media should be used for the B. Lackjaw
primary inoculation? C. Diarrhea
A. Kitt-Tarozzi D. Rash on a skin
B. Loeffler E. Nausea
C. Levenstain-Iensen 393. A patient after infected product use had
D. Endo got the urgent prevention of botulism.
E. Ploskirev Indicate, which from the noted preparations
389. A bacteriological laboratory conducts a should be used?
research of canned meat for the presence of A. Polivalent antitoxic serum
botulinum toxin. An extract from the B. Interferon
researched material with antitoxic C. Monovalency antitoxic serum
antibotulinic serum of A, B, E types was D. Anatoxin
injected to the experimental group of mice. E. Placent gammaglobulin
An extract without antitoxic serum was 394. A 20 dayed baby has dead after a tetanus
injected to the control group. What serologic infection. Where this infectious agent can be
test has been used? revealed?
A. Opsono-phagocytal A. Blood
B. Precipitation B. Spinal cord
C. Complement’s C. Umbilical wound
D. Neutralization D.Gastro-intestinal tract
E. Doubled immune diffusion E. Muscles
390. The tetanus symptoms appeared at a 395. A patient with botulism’s symptoms was
patient in 7 days after motor-car accident delivered to a hospital. C.botulinum toxin was
trauma. The course of treatment by an revealed from food stuffs. It is known few
antibotulinum serum was appointed for him people used that meal too. Which
and a patient began to convalesce. In two preparations must be used for urgent
weeks a temperature has risen, limphatic prevention of disease?
nodes were multiplied, swelled joints, rash, A. Antitoxic serum
itch and violation from the side of heart B. Anatoxin
vascular system were appeared in a patient. C. Killed vaccine
How the state of a patient is named? D. Antibiotics

44
E. Normal immunoglobulin E. Nausea.
396. A patient addressed to the dentists with 401. A bacteriological laboratory conducts a
difficulty of chewing. The doctor guesses research of canned meat for the presence of
tetanus, cause 3 weeks ago the patient unjured botulinum toxin. An extract from the
his leg with a rusty nail. What material is researched material with antitoxic
necessary to send to bac. laboratory to antibotulinic serum of A, B, E types was
confirm the diagnosis? injected to the experimental group of mice.
A. Serum of the patient An extract without antitoxic serum was
B. Blood of the patient injected to the control group. What serologic
C. Dab from the surface of wound test has been used?
D. Washings from the nail A. Complement fixation.
E. Necrotic tissue pieces B. Precipitation.
397. A patient with the previous diagnosis C. Neutralization.
botulism has been hospitalised. What D. Opsonophagocytic.
serological reaction should be used to E. Double immune diffusion.
determine botulinum toxin in test 402. A student, having eaten meat, tinned
specimens? in domestic conditions, has symptoms of
A. Complement fixation test. food poisoning, caused by Clostridium
B. Agglutination. botulinum: diplopia, dysphrasia,
C. Neutralization. respiratory paralysis. What caused such
D. Precipitation. symptoms of botulism?
E. Irnmunofluorescence test. A. Enterotoxin secretion.
398. After eating canned mushrooms a patient B. Botulinum invasion into intestinal
has bulbar paralysis symptoms: ptosis, epithelium.
diplopia, aphonia, and dysphagia. Botulism C. Neurotoxin action.
was previously diagnosed. What test should D. Endotoxic shock.
be used to determine the toxin type? E. Activation of adenylate cyclase.
A. Neutralization. 403. Jerked fish, being the cause of food
B. Agglutination. poisoning, has been delivered to a
C. Precipitation. bacteriological laboratory. The sample was
D. Complement fixation test. cultu red under anaerobic condition. After
E. Immunofluorescence test. inoculation into Kitt-Tarozzi medium a
399. A jerked fish of domestic preparation bacteriologist has revealed microorganisms
which caused serious food poisoning was akin to a tennis-racket. What disease do
examined in a bacteriological laboratory. they cause?
Microscopy of the culture isolated on A. Typhoid fever.
Kitt-Tarozzi medium revealed B. Salmonellosis.
microorganisms akin to a tennis-racket. C. Dysentery.
What diagnosis is possible? D. Staphylococcal toxic infection.
A. Botulism. E. Botulism.
B. Salmoncllosis.
C. Cholera. Microbiological diagnostics of Syphilis
D. Shigellosis. 404. A patient had Syphilis 2 years ago. On
E. Typhoid fever. reproductive organs an ulcer (shancer) was
400. The causative agent of tetanus produces revealed again. What form of infection is it?
exotoxin with different biological action A. Reinfection
effects. What clinical symptoms can this toxin B. Secondary infection
cause? C. Relapsis
A. Disorders of vision. D. Superinfection
B. Lackjaw. E. Autoinfection
C. Diarrhea. 405. Wassermann test was positive in 8-
D. Skin rash. monthes pregnancy woman blood serum

45
during preventive examination. How does the B. Brucellosis
authenticity of serologic examination of C. Tuberculosis
Syphilis should be confirmed? D. Syphilis
A. Test must be repeated with 10-15 days E. Influenza
interval 411. The laboratory assistant has taken the
B. Again examination after the got result blood serum from a patient with previous
C. Again examination after treatment diagnosed «Syphilis» for the immune test
D. To use the sedimentary Kann`s test based on antibodies which inhibit the motility
E. To use the sedimentary Zax-Vitebsky test of treponemes and kill them revealing. What
406. A vein blood was taken for Wassermann test has been used for diagnostics?
test at the planned examination of pregnant A. Immobilization
woman in maternity welfare centre. The test B. Complement binding
was positive. The pregnant woman and her C. Agglutination
husband deny extramarital sexual relations. D. Precipitation
What should be done for Syphilis E. Neutralization
confirmation or refutation of diagnosis? 412. The pinky spirohaetes were revealed
A. To use the treponema immobilization test during the microscoping of dabs from
B. To take a dab from an urethra shancer of a patient. What method of dab
C. To repeat the Wassermann test staining was used in this case?
D. To use the sedimentation reactions A. Romanovsky-Giemza
E. To use the complement binding test B. Burri-Gins
407. A patient with Syphilis has passed C. Ziehl-Nilsen
complete course of antibiotic therapy and was D. Morozov
fully cured. After some time he was infected E. Gram
again with Treponema pallidum. How can be 413. A patient was delivered to a hospital with
named this form of infection? suspicion on Syphilis. What method should
A. Complication be used for detecting the infectious agent in
B. Recidive hard shancer?
C. Secondary infection A. Microscopy of material stained by Gram’s
D. Superinfection method
E. Reinfection B. To isolate of pure culture
408. What method of serologic examination is C. Darkfield microscopy
not used for Syphilis diagnostics? D. Wasserman test
A. Treponema imobilization test E. Agglutination
B. RMP 414. The laboratory assistant prepared such
C. Wassermann test reagents for Wassermann test: cardiolipid
D. Agglutination antigen, alcoholic extract of lipids from the
E. Indirect IF cardiac muscle of bull with cholesterol,
409. Thin spiral pink microorganisms with inactivated by an ultrasound treponema
12-14 scrolls and sharp ends were revealed in antigen, hemolytic system, NaCl, examined
a regionar lymphatic node punctat of patient serums. What component should be used for
stained by Romanovscy-Giemza. What Wasserman test at serologic diagnostics of
disease the infectious agent in this case cause? Syphilis?
A. Leptospirosis A. Complement
B. Typhus B. Live treponema
C. Campilobacteriosis C. Ram`s erythrocytes
D. Sodocu D. Diagnostic serum
Е. Syphilis E. Antiglobulin serum
410. A 30-year-old patient's Wassermann 415. The dark-field microscopy of the 28
test is positive (++++). What infectious yeared patient shancer was used in the
disease diagnostics is this test used for? laboratory of dermatovenerologic dispanser.
A. Poliomyelitis What property of Syphilis treponema was

46
studied? C. Agglutination.
A. Active motility D. Precipitation.
B. Spiral shape E. Neutralization
C. Number of scrolls 421. The patients which have delivered to
D. Primary character of scrolls surgical stomatology department need to give
E. Secondary character of scrolls blood for Wassermann test. For what purpose
416. What type of serological tests is not used this reaction are used?
for Syphilis diagnostics? А. Detection of antibodies to Treponema
A. Microprecipitation test В. Detection of antibodies to Typhus agent
B. Wasserman С. Detection of antibodies to HIV
C. Immobilization D. Detection of syphilis agent
D. Indirect IF Е. Detection of antibodies to hepatitis agents
E. Agglutination 422. The patients of surgical department need
417. Microscopy of the specimen of a to give blood for Wassermann test. What type
regional lymph node punctate stained by of reaction this test belong to?
Romanowsky-Giemsa showed thin A. Immobilization
microorganisms with 12-14 regular curls and B. Complement binding
pointed ends of light rose color 10-13 C. Agglutination
micrometers long. What disease could this D. Precipitation
agent cause? E. Neutralization
A. Relapsing fever.
B. Toxoplasmosis. Microbiological diagnostics of recurrent
C. Leptospirosis. typhus and leptospirosis
D. Syphilis. 423. In the micropreparation made from
E. Leishmaniasis. patient's regional lymph node punctate and
418. A patient with syphilis suspected has stained by Romanovsky-Giemsa method, the
arrived to a hospital. What method should be doctor found out thin microorganisms with
used for detecting the causative agent in hard 12-14 equal ringlets and pale-pink sharp
chancre? pointes 10-13 mkm in length. Name the
A. Microscopy of material stained by Gram's pathogen agent.
method. A Bartonella bacilliformis
B. Isolation of pure culture. B. Borrelia dutlonii
C. Using dark-field microscopy. C. Calymmatobacterium granulomatis
D. Wassermann test. D. Leptospira interrogans
E. Agglutination test. E. Rickettsia mooseri
419. Wassermann test is positive in a 20- 424. In the endemic region of leptospirosis
yeared man. What infectious disease the population is ill with this dangerous
diagnostics is this test used for? illness. What source of the infection is the
A. Brucellosis. most dangerous?
B. Syphilis. A. Rodents.
C. Tuberculosis. B. Dairy products.
D. Poliomyelitis. C. Cattle.
E. Influenza. D. Meat products.
420. A laboratory assistant has taken blood E. Ticks.
serum from a patient with preliminary 425. A patient was hospitalized on the 5th
diagnosed syphilis for the immune reaction day of illness with jaundice signs, pain in
based on detecting antibodies, which inhibit muscles, rigor, nose bleeding. A bacteriologist
the motility of treponemes and lead to their used dark-field microscopy. In the patient's
death. What test has been used for blood there have been revealed motile
diagnostics? spiral microorganisms. Name the causative
A. Complement fixation. agent.
B. Immobilization. A. Bartonella bacilioformis.

47
B. Borrelia duttoni. А. Meat products
C. Calymmatobacterium granulomatis. В. Milk products
D. Leptospira interrogans. С. Horned cattle
E. Rickettsia mooseri. D. Rodents
426. A patient with a periodically Е. Pincers
repeating fever is hospitalized to an 431. The veterinary surgeon with suspicion on
infectious department. In blood specimen (a brucellosis was arrived to an infectious
thick drop), stained by Romanovsky-Giemsa diseases hospital. During his examination the
method, spiral microorganisms with sharp diagnosis «Relapsing tick typhus» has been
ends of blue-violet color are revealed. Which put. What way of transmission the patient
disease is caused by this agent? could be invaded?
A. Relapsing fever. А Through malarial mosquito sting
B. Typhoid fever. В. Through a dog tick sting
C. Malaria. С. Through a rural tick sting
D. Epidemic typhus. D. Through a mosquito sting
E. Leptospirosis. Е. Through an ixodes tick sting
427. A man died from an acute infectious 432. The patient was hospitalized after the
disease accompanied by fever, jaundice, fifth day of illness with displays spleen and
haemorrhagic rash on the skin and mucous pains in muscles fever and nasal bleeding.
membranes as well as by acute renal The bacteriologist has performed the dark
insufficiency. Histological examination of field microscopy of the patients blood. Name
renal tissue (stained by Romanovsky-Giemsa the infection agent.
method) revealed some convoluted bacteria A.Bartonella bacilliformis
looking like C und S letters. What bacteria B. Borrelia dutlonii
were revealed? C. Calymmatobacterium granulomatis
A. Treponema D. Leptospira interrogans
B. Leptospira E. Rickettsia mooseri
C. Spirilla 433. The doctor has revealed blue-violet
D. Borrelia thread microorganisms with several big twist
E. Campilobacteria long from 10 to 30 microns and more at
428. A sick man with high temperature and a microscopy of blood micropreparation
lot of tiny wounds on the body has been stained by Romanovsky-Giemza. For what
admitted to the hospital. Lice have been found infectious disease such microscopic picture is
in the folds of his clothing. What disease can characteristic?
be suspected in the patient? А. Recurrent typhus
A. Scabies В. Syphilis
B. Malaria С. Leptospirosis
C. Plague D. Tripanosomosis
D. Tularemia Е. Leshmaniosis
E. Relapsing typhus 434. Recurrent typhus, caused B. caucasica,
429. A patient with the final diagnosis meets in the definite areas only. Electorobius
recurrent typhus had delivered to an tick is transmitter. How is such infection
infectious diseases hospital. What material called?
must be taken first of all? А. Exotic
A Liquor В. Endemic
В. Urina С. Sporadic
С. Blood D. Pandemic
D. Excrements Е. Epidemic
Е. Smear from a nasopharynx 435. A sick man with a fever which
430. The population has leptospirosis in the sometimes repeated been has admitted to the
endemic zone of this dangerous disease. What infectious hospital. Blue-violet helical
source of this infection can be most probable? bacteria with sharp ends were revealed in the

48
blood preparation (thick drop) stained by D. Bed-bugs
Romanovsky-Giemza. Name the infection E. Cockroaches
agent. 441. A sick man with high temperature and a
А Bartonella bacilliformis lot of tiny wounds on the body has been
B. Borrelia dutlonii admitted to the hospital. Lice have been found
C. Calymmatobacterium granulomatis in the folds of his clothing. What disease can
D. Leptospira interrogans be suspected in the patient?
E. Rickettsia mooseri A. Scabies
436. Doctor made clinical diagnosis recurrent B. Malaria
typhus to patient. The microscopy was C. Plague
prescribed for laboratory diagnosis D. Tularemia
confirmation. In which material the infectious E. Epidemic typhus
agent could be revealed? 442. Parents with ill child came to the
А. In urine infectionist. They worked in one of the Asian
В. In sweat countries for a long time. Child has eathy
С. In blood colored skin, loss of appetite, laxity, enlarged
D. In excrement liver, spleen, peripheral glands. What
Е. Smear from a nasopharynx protozoan disease can this child have?
437. Examined patient has diagnosis “tick A. Visceral leishmaniasis
typhus”. How the patient could be infected? B. Balantidiasis
А Through malarial mosquito sting C. Amebiasis
В. Through a dog tick sting D. Toxoplasmosis
С. Through an ixodes tick sting E. Lambliasis
Д. Through a mosquito sting 443. Patients with similar complaints applied
Е. Through a rural tick sting to the doctor: weakness, pain in the intestines,
438. The specific prevention of leptospirosis disorder of GIT. Examination of the faeces
is planned to members of a society “Hunting revealed that one patient with four nucleus
and fishing”. What preparate should be used? cysts should be hospitalized immediately. For
А. Inactivated vaccine of several sera-groups what protozoa are such cysts typical?
В. Live vaccine of several sera-groups A. Dysenteric amoeba
С. Chemical vaccine B. Intestinal amoeba
D. Anatoxin C. Balantidium
Е. Antitoxic serum D. Trichomonas
439. Blue-violet thread spiral microorganisms E. Lamblia
stained by Romanovsky-Giemza at 444. 2 weeks since the blood transfusion a
microscopy of a material from patient with recepient has developed fever. What protozoal
fever. They were identified as Borrelia disease can it be?
recurrentis. What pathological material was A. Trypanosomоsis
taken? B. Malaria
А. Pus C. Amebiasis
В. Blood D. Toxoplasmosis
С. Urine E. Leishmaniasis
D. Sputum 445. A malarial plasmodium (haemamoeba) -
Е. Lymph nodes punctate the pathogene of vivax malaria - has two
440. A patient with suspicion on epidemic strains: southern and northern. They differ by
typhus was admitted to the hospital. Some the duration of their incubation period: the
arachnids and insects have been found in his southern has short and the northern - long
flat. Which of them may be a carrier of the one. What selection works in this case?
pathogen of epidemic typhus? A. Cutting
A. Lice B. Stabilizing
B. Spiders C. Artificial
C. Houseflies D. Moving

49
E. Sexual 450. A patient has been brought to the
446. A patient who came to the doctor hospital with the complaints of headache, pain
because of his infertility was administered to in left hypochondrium. He has been ill for 1,5
make tests for toxoplasmosis and chronic weeks. The sudden disease began with the
gonorrhoea. Which reaction should be increase of body temperature up to 39,90C. In
performed to reveal latent toxoplasmosis and 3 hours the temperature decreased and
chronic gonorrhoea in this patient? hydropoiesis began. The attacks repeat
A. RIHA - Reverse indirect hemagglutination rhythmically in 48 hours. The patient had
assay visited one an African country. The doctors
B. IFA - Immunofluorescence assay have suspected malaria. What method of
C. RDHA - Reverse direct hemagglutination laboratory diagnostics is necessary to use?
assay A. Urine examination
D. Immunoblot analysis B. Examination of vaginal and urethral
E. (R)CFT- Reiter's complement fixation test discharge
447. A journalist’s body temperature has C. Stool examination
sharply increased in the morning three weeks D. Blood examination
after his mission in India, it was accompanied E. Immunological tests
with shivering and bad headache. A few hours 451. A woman who was infected with
later the temperature decreased. The attacks toxoplasmosis during the pregnancy has a
began to repeat in a day. He was diagnosed child with multiple congenital defects.This is
with tropical malaria. What stage of a result of:
development of Plasmodium is infective for A. Teratogenesis
anopheles-female? B. Chemical mutogenesis
A. Sporozoites C. Biological mutogenesis
B. Shizontes D. Recombination
C. Gametocytes E. Cancerogenesis
D. Merozoites 452. A duodenal content smear of a patient
E. Microgamete with indigestion contains protosoa 10-18 mcm
448. Slime, blood and protozoa 30-200 large. They have piriform bodies, 4 pairs of
microns of length have been revealed in a filaments, two symmetrically located nuclei in
man's feces. The body is covered with cilias the broadened part of body. What kind of the
and has correct oval form with a little bit lowest organisms is it?
narrowed forward and wide round shaped A. Balantidium
back end. On the forward end a mouth is B. Intestinal ameba
visible. In cytoplasm there are two nucleuses C. Dysentery ameba
and two short vacuoles. For whom are the D. Trichomonas
described attributes typical? E. Lamblia
A. Balantidium 453. Slime, blood and protozoa 30-200
B. Lamblia microns long have been revealed in a man's
C. Intestinal amoeba feces. The body is covered with cilias and has
D. Trichomonas correct oval form with a little bit narrowed
E. Dysenteric amoeba anterior and wide round shaped posterior end.
449. A businessman came to India from South At the anterior end a mouth is visible. In
America. On examination the physician found cytoplasm there are two nucleui and two short
that the patient was suffering from sleeping- vacuoles. What are the described features
sickness. What was the way of invasion? typical for?
A. Through dirty hands A. Lamblia
B. With contaminated fruits and B. Balantidium
vegetables C. Trichomonas
C. As a result of mosquito's bites D. Dysenteric amoeba
D. After contact with a sick dogs E. Intestinal amoeba
E. As a result of bug's bites 454. A woman with complaints

50
characteristic of inflammatory process in A. Trichomonas
vagina has referred to a gynecolo gist. B. Lamblia
Which protozoa can it be caused by? C. Amoeba dysenteric
A. Trichomonas vaginalis. D. Balantidium
B. Toxoplasma gondii. E. Amoeba intestinal
C. Plasmodium malariae. 460. Protozoa were revealed during an
D. Entamocba coli. examination of the patient’s Giemsa stained
E. Lamblia intestinalis blood smear. The doctor put diagnosis: Crus-
455. A duodenal content smear of a patient Chagas disease. What is the pathogen of the
with indigestion contains protosoa 10-18 mcm disease?
large.They have piriform bodies, 4 pairs of A. Leishmania tropica.
filaments, two symmetrically located nuclei in B. Toxoplasma gondii.
the broadened part of body. What kind of the C. Leishmania donovani.
lowest organisms is it? D. Trypanosoma crusi.
A Lamblia E. Trypanosoma brucci.
B Dysentery ameba 461. The woman with symptoms similar to
C Trichomonas the inflammatory process of the vagina has
D Intestinal ameba addressed to the gynecologist. What protozoa
E Balantidium can be a pathogen of the disease?
456. Giemsa stained protozoa, which have A. Trichomonas vaginalis.
half moon shape with acuminate ends, blue B. Toxoplasma gondii.
cytoplasm and ruby-red C. Plasmodium malariae.
nucleus, were revealed during microscopical D. Entamoeba coli.
examination of spinal liquid smears. What E. Lamblia intestinalis.
protozoal disease can it be? 462. The pregnant woman with complains,
A. Leishmaniasis. similar to the toxoplasmosis, has addressed to
B. Malaria. the doctor. The blood examination was
C. Toxoplasmosis. proposed to confirm a clinic diagnosis. What
D. Trypanosomosis. serological reaction must be done in this case?
E. Amoebiasis. A. Complement fixation test.
457. What are the ways of infection at B. Precipitation test.
malaria? C. Neutralization test.
A. Alimentary. D. Widal reaction
B. Droplett-aerogenic. E. Wassermann reaction
C. Transmissive. 463. Giemsa stained protozoa, which have
D. Contact. half moon shape, protoplasm is colored blue,
E. Dusty-aerogenic. nucleus is colored red, were revealed during
458. 32 yeared woman with asymptomatic microscopical examination of the blood
disease had second dead child was borned smear. What protozoa can be a pathogen?
with microcephaly. What disease can it be? A. Toxoplasma.
A. Listeriosis B. Trypanosome.
B. Brucellosis C. Leishmania.
C. Histoplasmosis D. Lamblia.
D. Toxoplasmosis E. Balantidium.
E. Syphilis 464. A patient has addressed to the doctor.
459. Patients had come to the doctor with The doctor detected painless ulcers, covered
similar symptoms: weakness, intestinal pain, by brown-red peels on the opened parts of the
alimentary tract disorders. One of the patients body. A surface, covered by granulation was
needs urgent hospitalization, because after under the ulcers. Round and oval
feces examination four nuclear cysts were microorganisms were revealed during Giemsa
revealed. For which protozoa these cysts are stained micropreparation microscopical
typical? examination. The disease lasts more then

51
year. What protozoa can be a pathogen of the 469. Indirect Immunofluorescence reaction
disease? was used to toxoplasmical antibodies
A. Trichomonas hominis. detecting in the patient’s blood serum. The
B. Leishmania tropica var. major. first step is to cover fixed toxoplasmical
C. Leishmania donovani. smear by experimental serum. By what will
D. Lamblia intestinalis. you cover your preparation at the second
E. Leishmania tropica var. minor. step?
465. Round form microorganisms which A. Fluorescein solution.
cytoplasm has erythrocytes and small-shaped, B. Fluorescent serum against human
four nucleus cysts were revealed during immunoglobulin.
microscopical examination of native C. Human immunoglobulin of normal
preparation from patient’s feces. Feces have serum.
bloody-mucus consistency. What is the D. Diagnostical serum ,which has
pathogen of the disease? toxoplasmic antibodies.
A. Entamoeba histolytica. E. Fluorescein labeled toxoplasmical
B. Entamoeba coli. antibodies serum.
C. Lamblia intestinalis. 470. A patient with a liver suspicion has
D. Trichomonas intestinalis. entered the surgical department of the
E. Leishmania donovani. hospital. The patient was at the business trip
466. A patient has addressed to the doctor in the one African country for a long period
because of his infertility. The patient was and he had acute forms of alimentary tract
appointed to toxoplasmosis and chronical diseases many times. What protozoal disease
gonorrhea examination. What reaction must can it be?
be done to detect hidden toxoplasmosis and A. Leishmaniasis.
chronical gonorrhea in this patient? B. Trypanosomosis.
A. Immunofluorescence reaction. C. Malaria.
B. CBT. D. Amoebiasis
C. Immunoblotting. E. Toxoplasmosis.
D. Reversed passive hemagglutination test 471. Protozoa, which size are 10-18 mcm,
E. Countercurrentimmunoelectrophoresis. were revealed in the duodenum content
467. Patient H., 40 years old came back home smear. The body has pear shape, four pairs of
after some month swimming in Western flagella. There are two symmetrically located
Africa. He feeled weakness, headache, fever nucleuses in the front enlarged part of the
15 days. The doctor put diagnosis: malaria. protozoa body. What protozoa can it be?
What method of laboratory examination can A. Amoeba dysenteria
confirm this diagnosis? B. Balantidium.
A. Microscopical, serological. C. Amoeba intestinal
B. Bacteriological, allergic. D. Lamblia.
C. Bacteriocsopical, biological. E. Trichomonas.
D. Serological, biological. 472. Patient C., referred to the infectional
E. Microscopical, cultural. department of the hospital with his skin ulcer,
468. A patient with inflammation of bile ducts which appeared 3 month ago during his trip to
has entered the gastroenterological Middle Asia country. Lanceolated protozoa
department. Lively protozoa which have pear with flagellas were revealed after ulcer
shape, two nucleuses and have supporting material on blood defibrinated agar
rod-axostyle were revealed in bile portions. inoculation. What protozoa can it be?
What protozoa disease can it be? A. Trichomonas vaginalis.
A. Lambliasis. B. Leishmania donovani.
B. Amoebiasis intestinal. C. Toxoplasma gondii.
C. Balantidiasis intestinal. D. Lamblia intestinalis.
D. Dysentery E. Leishmania tropica.
E. Trichomoniasis.

52
473. Toxoplasmosis was suspected in the 477. Bacteriologic examination of urethral
newborn. Enzyme linked immunosorbent discharge of 40-yeared man with chronic
assay was used for specific antibodies inflammation of urinary-genital system was
detection in the umbilical blood. Presence of performed. After the inoculation of
what immunoglobulin will confirm pathological material onto the serum agar with
intrauterine infection? cholesterol, small colonies with a dense center
A. IgA. ingrown into culture medium and with
B. IgM. semilucent periphery grown were appeared.
C. IgG. What causative agent it was?
D. IgD. A. Chlamydia
E. IgE. B. Gonococci
474. Patient H with complains for prostate C. Mycobacterium
pains and large excretion from urethra has D. Gardnerella
addressed to the infectionist. The doctor E. Mycoplasma
thinks that it is trichomoniasis. What method 478. 11-yeared girl, who cared for the animals
of laboratory examination can confirm this in a school ''live corner”, has a flu like disease
diagnosis? (weakness, fever, loss of appetite and great
A. Enzyme-linked immunosorbent assay, headache).The symptoms of
ELISA, for antibodies detection bronchopneumonia appeared later. The
B. Immunofluorescence reaction for causative agent was revealed after patient’s
antigen detection. blood injection of chicken embryos into a yolk
C. An allergic test sac. Skin test with chlamidin was positive in
D. Excretion smear microscopical this sick girl and in three schoolboys, that
examination. worked in a «alive corner».What animal could
E. Bactereological test. be the most possible source of this infection?
475. Two weeks after hemotransfusion a A. Hamsters
patient had fever. What protozoal disease can B. Rabbits
be suspected? C. Parrots
A. Leishmaniasis. D. Tortoises
B. Trypanosomosis. E. Cats
C. Malaria. 479. The cytoplasmа inclusions in the form of
D. Amoebiasis a typical little “cap” above the nucleus of the
E. Toxoplasmosis. cell were revealed in the epithelial cells during
the microscopy of urethral smears. What
Microbiological diagnosis of chlamidiosis, disease should be surmised?
mycoplasmosis and rickettsiosis A. Chlamydiosis
476. The patient with clinical signs of B. Gonorrhoea
Mycoplasmosis was examined. The diagnosis C. AIDS
was confirmed by bacteriologic method in a D. Siphilis
patient with the clinical signs of E. Genital herpes
mycoplasmosis. Mycoplasma was not 480. The epithelial cells with reticular and
revealed after second time pathological elementary little bodies stained by
material inoculation onto the media. Possible Romanovsky-Giemza were revealed during
reason of negative result is wrong prepared microscopy of scrape of mucous membrane of
culture medium. Absence of what substance a patient with urino-genital infection. Which
in culture medium would be the most possible pathogenic microorganisms is it?
reason of negative result? A. Mycoplasmas
A. Agar-agar B. Chlamydiae
B. Axerophthol (retinol) C. Rickettsia
C Glicerine D. Viruses
D. Glucose E. Fungi
E. Cholesterol

53
481. Chlamydia trachomatis was isolated from C. Mycoplasma produces the enzyme
the genital tract of 35-yeared man with chronic destroying penicillin
urethritis. What other organs can impress this D. Mycoplasma does not have a cellular wall
causative agent? E. Mycoplasma does not form corresponding
A. Gastrointestinal tract transport proteins
B. Kidneys 486. A patient with suspected rickettsiosis was
C. Joints delivered to the hospital. What method of
D. Central Nervous System diagnostics must be applied for confirmation
E. Eyes of diagnosis?
482. What research method is the most A. Cultivation in a chicken embryo
informative at the control of Chlamydia B. Microscopic
infection treatment? C. Cultivation on the artificial nourishing
A. Determination of antibody titer in ELISA media
B. Direct IF D. Serological
C. PCR E. Biological
D. Western blot 487. The method more frequently used in
E. Microscopic smears bacteriological laboratory for diagnostics of
483. Pneumonia of mycoplasma etiology was the epidemic spotted fever is:
diagnosed in a patient. Antibiotic with what A. Serological method
mechanism of action should not be used for B. Infection cells cultures
treatment? C. Infection chicken embryos
A. Antibiotics inhibiting synthesis of cellular D. Bacteriological method
wall E. Allergic method
B. Antibiotics disturbing the permeability of 488. The 73-yeared woman has shallow
plasma membrane descents on a skin, high temperature and
C. Antibiotic disturbing protein synthesis dizziness. Chicken embryos yolk sack
D. Antibiotic disturbing nucleic acids infection with blood of the patient showed the
synthesis presence of ultrafine Gr- bacillary and
E. Antibiotic disturbing the oxidative coccoidal form microorganisms. The reaction
phosphorylation of Weighl-Felix is positive. In childhood the
484. A 30-yeared man has urethritis and patient had typhus, but she does not remember
prostatitis. With the aim of microbiological which one. What method of diagnostics will
diagnostics culture isolation was performed. allow to find out if this disease is epidemic
The growth of causative agent was achieved spotted fever or Brill disease?
only on the culture medium with 10 % urine A. Infection laboratory animals
addition. What group of microorganisms does B. Reaction of complement binding with the
a causative agent belong to? paired sera
A. Chlamydia C. Reaction of microagglutination
B. Neisseria D. Allergic skin test
C. Mycoplasma E. Definition IgM and IgG
D. Gardnerella 489. A patient has diagnosis Brill disease as a
E. Staphylococci result of blood serological examination. What
485. A patient with an urethritis within a week microorganism is the agent of this disease?
independently treated oneself by the penicillin, A. Rickettsia prowazekii
but the state of patient did not improve. The B. Pseudomonas aeruginosa
bacteriological examination showed that a C. Salmonella typhi
causative agent is mycoplasma. Why D. Borrelia recurrentis
medications accepted by the patient were not E. Pneumocystis carini
effective?
A. Membrane of mycoplasma contains Myсological diagnostics of a candidiasis
cholesterol and mycosis
B. Causative agent multiply inside the cell 490. A child with clinical signs of candidiasis

54
was hospitalised to the clinic. What laboratory D. Pneumocystis carinii
examination is necessary for the diagnosis Е. Mucor mucedo
"candidiasis" confirmation? 495. Gram-positive big oval budding cells
A. Microscopical, mycological which are located chaotically and in the form
B. Allergic, mycological of chains were revealed from mucous
C. Histological, mycological membranes and sputum of the patient who
D. Biological, mycological used immunosuppressants long time. What
E. Serological, mycological agent has been isolated?
491. A pregnant woman complained of A. Actinomycetes
itching and genital tracts discharges. B. Streptobacterii
Bacterioscopy of vaginal discharges C. Yersinii
smear showed big Gram-positive oval D. Candida
oblong cells which form pseudomycelium. E. Streptococci
What is the most possible way of 496. The oral cavity candidiasis was
transmission? diagnosed in the child. What preparation is
A. Sexual used for candidiasis treatment?
B. Endogenous infection A. Nystatin
C. Contact indirect B. Gentamycin
D. Transmissive C. Penicillin
E. Wound infection D. Tetracyclin
492. Microscopy of the smear of vaginal E. Cyfran
discharges of a woman with chronic 497. Whitish exudates which remind curds
colpovaginitis revealed round and oval milk was revealed in the child`s cheeks
budding cells 3-6 micrometers in size. mucosa and tongue. Gram-positive, oval yeast
Name the fungal disease, which can be were revealed in preparations-dabs. Name
caused by these microorganisms. this agent.
A. Epidermophytium. A. Fuzobacterii
B. Coccidiosis. B. Staphylococci
C. Candidosis. C. Diphtheroides
D. Microsporia. D. Actinomycetes
E. Cryptococcosis. E. Candida
493. Choose among mentioned below the 498. A child was hospitalized to a clinic.
most probable reasons of acute On the mucous membrane of cheeks,
pseudomembranous candidiasis (milky palate, and tongue there was revealed white
disease) development. and yellowish fur typical of candidiasis.
A. Long antibiotic therapy What material should be taken for
B. Immunodeficiency of B-system immunity examination?
C. Long immunomodulating therapy A. White exudates (plaques) from different
D. Chronically inflammatory processes sites of oral cavity.
E. Atopic status B. Hair and nails.
494. A whitish touch scraping from an oral C. Blood.
cavity mucous membrane was directed to D. Mucus from nasopharynx.
the laboratory. As a result of microscopic E. Urine.
examination it has been found out, that the 499. The scraping of albescent fur from oral
microorganism belongs to the list of cavity mucous membranes was sent to a
opportunistic mycoses agents. What of the laboratory. The material was inoculated
resulted microorganisms is the contagious onto Sabouraud's medium. Sour cream-like
mycosis agent, instead of an opportunistic colonies' growth was revealed.
one? Bacterioscopic method has shown short
А. Microsporum cаnis budding strings. What group of infections
В. Aspergillus niger can we refer this disease to?
С. Candida albicans A. Mycosis.

55
B. Spirochetosis. patient had a white fur on an oral cavity
C. Rickettsiosis. mucous membrane. Microscopic studying of
D. Mycoplasmosis. dabs prepared from fur has revealed large,
E. Chlamidiosis. roundish, Gram positive various sizes
500. The dark purple, oval, roundish, budding microorganisms. What it is necessary to make
cells were revealed at microscopy of scrape for continuation of microbiological
from a tongue surface stained by Gram's diagnostics?
method. What disease agent is it? A. Yolk-salt agar inoculation
A. Diphtheria B. Ploskirev agar inoculation
B. Actinomycosis C. Kitt – Tarozzi media inoculation
C. Streptococcal infection D. Endo agar inoculation
D. Staphylococcal infection E. Saburaud agar inoculation
E. Candidiasis 505. The oral cavity candidiasis is diagnosed
501. After long antibiotic therapy a mouth in the child. What preparation should be used
mucous membrane sites with white fur which for candidiasis treatment?
did not deleted with spatula have been A. Cifran
revealed in patient. For what disease such B. Gentamycin
signs are characteristic? C. Penicillin.
A. Parodontitis D. Tetracyclin
B. Parotitis E. Nystatin
C. Candidiasis 506. The patient had a sensation of burning in
D. Flue a mouth, his tongue was covered wit h
E. Typhus dense whitish cheese like fur. The milk
502. A pediatrician examining a child has disease was diagnosed. What preparation
noticed that the mucous membrane of the should be used for treatment?
oral cavity and t he tongue are covered A. Grizeofulvin
wit h dense whitish fur. In the material B. Nystatin
taken from the site of affection a C. Gentamycin
bacteriologist has revealed yeast. What D. Amfotericin
mycosis will he suspect in this case? E. Tetracyclin
A. Actinomycosis. 507. A child was hospitalized to a clinic.
B. Favus. On the mucous membrane of cheeks,
C. Epidermophytia. palate, and tongue there was revealed white
D. Candidiasis. and yellowish fur typical of candidiasis
E. Trichophytosis caused by Candida albicans. What of the
503. The white creamy plaques which easily listed preparations is used for candidiasis
deleted leaving blood erosion have been treatment?
revealed on a mouth mucous membrane of A. Penicillin
pupil С during routine inspection of B. Cifran
schoolboys by the doctor-stomatologist. The C. Nystatin
doctor has suspected pseudomembranous D. Gentamycin
candidiasis and has appointed to the patient E. Tetracyclin
mycological examination. What of the listed 508. Microscopic examination of a Gram-
nutrient mediums should be used for stained scrape from patient's tongue revealed
revealing of the candidiasis agent? oval, round, elongated chains of dark-violet
A. Saburaud gemmating cells. What disease can be caused
B. Roux by this causative agent?
C. Endo A. Actinomycosis
D. Kitt-Tarozzi B. Candidosis
E. Hiss C. Streptococcic infection
504. Patient Т, 68 years old, has passed a long D. Staphylococcic infection
course of antibiotic. After treatment the E. Diphtheria

56
509. A child was hospitalized to a clinic. What is the most plausible causative agent
On the mucous membrane of cheeks, of the disease?
palate, and tongue there was revealed white A. Staphylococci.
and yellowish fur typical of candidiasis. B. Fungi of Candida genus.
What material should be taken for C. Streptococci.
examination? D. Diplococci.
A. White exudates (plaques) from E. Sarcina.
different sites of oral cavity 514. Candidosis is diagnosed in a child.
B. Hair and nails. What preparation should be used for
C. Blood. treatment?
D. Mucus from nasopharynx. A. Cifran.
E. Urine. B. Gentamycin.
510. A pediatrician examining a child has C. Penicillin sodium salt.
noticed that the mucous membrane of the D. Tetracycline hydrochloride.
oral cavity and t he tongue are covered E. Nystatin
wit h dense whitish fur. In the material 515. Scraps of the mycelium of a fungus,
taken from the site of affection a spores, air bubbles and fat drops were
bacteriologist has revealed psudohypha. discovered on microscopy of the patient's hair
What mycosis will he suspect in this case? excluded from the infected areas. For what
A. Actinomycosis. fungus disease is this microscopic picture
B. Favus. characteristic?
C. Epidermophytia. A. Favus
D. Candidiasis. B. Microspory
E. Trichophytosis C. Trichophytosis
511. The scraping of albescent fur from D. Epidermophytosis
oral cavity mucous membranes was sent to a E. Sporotrichosis
laboratory. The material was inoculated 516. A doctor asked the patient, the skin of
onto Sabouraud's medium. Sour cream-like the scalp which were affected area of hair
colonies' growth was revealed. chippy near the skin surface. Small spores
Bacterioscopic method has shown short were revealed inside the hair at microscopy.
budding strings. What group of infections Which disease is it?
can we refer this disease to? A. Rubromycosis
A. Mycosis. B. Favus
B. Spirochetosis. C. Trichophytia
C. Rickettsiosis. D. Candidosis
D. Mycoplasmosis. E. Microsporia
E. Chlamidiosis. 517. 40 yeared man has the damaged areas
512. On the mucous membrane of cheeks between the toes on his feet: skin get wet,
and tongue of a child albescent stains were peel, crack appeared. When sowing scrapes
revealed. In the prepared smears Gram- skin. The fluffy white top and a greenish-
positive oval yeast-like cells are found. What yellow underneath colonies were grew on
causative agent is this? Saburaud media. Conidium as "cudgel’ with
A. Corinebacterium diphtheriae. 1-5 cells is visible n smears from the top of
B. Staphylococci. the colony. What other organs most likely can
C. Fungi of Candida genus. affect this fungus?
D. Actinomycetes. A. Mucous of genitalies
E. Fusobacteria. B. Hair
513. A pregnant woman complained of C. Under the skin tissue
itching and genital tracts discharges. D. Lymphatic vessels
Bacterioscopy of vaginal discharges E. Nails
smear showed big Gram-positive oval 518. A doctor revealed threads of mycelium
oblong cells which form pseudomycelium. showed double-circuit, as round and square

57
disputes at microscopy of affected areas structures that can be found in the smears
epidermis pieces of skin taken from the from the damaged mucosa, confirm the
interdigital folds and soles of patients (coal preliminary diagnosis?
miners). A fungal pathogen of which disease A. Cells that form buds
may be referred to in this case? B. Ovoid cells formed ascospores
A.Microsporia C. Branching threads
B.Candidosis D. Long cells form psudohypha
C.Trichophytia E. Small spores
D.Epidermophytia 522. Select from the following below the
E.Scabiasis most likely causes of acute
519. A dense whitish fur which easily pseudomembranous candidiasis (thrush).
deleted was revealed in the oral cavity after A. Chronic inflammation
long-term treatment with ceftriaxone. Smear B. B-system immunedeficiency
microscopy revealed large round cells with a C. Long-term immunomodulatory therapy
nucleus. Which media are used for agent D. Prolonged antibiotic therapy
isolation? E. Atopic state
A. MPA 523. Psudohypha was revealed in a smear
B. Nutrient agar of the studied material taken from patients
C. Saburaud with suspected fungal infection. What micro
D. Yolk salt agar organism is it?
E. Roux A. Aspergillus
520. Man 70 y.o. developed prosthetic B. Penicillum
stomatitis. In addition, there was pronounced C. Candida
corners of mouth lesions. What D. Actinomycetes
microorganisms most likely were the leading E. Mycobacterium
etiological factor for this injury? 524. Prolonged use of antibiotics in the
A. Streptococci patient K., 43 years, causing oral mucosa
B. Staphylococci candidiasis. What type of Candida most
C. Neisseria commonly causes oral candidiasis?
D. Candida A. S.aureus
E. Clostridium perfringens B. C. tropicalis
521. Dentist suspected Candida etiology C. С.albicans
stomatitis in children 4 years old and D. S.pyogenes
appointed microscopic study. What are the E. C.pylori
525. After continuous treatment with organisms. What features of living organisms
antibiotics a patient got symptoms of are typical for?
stomatitis. Examination of specimens of oral A. Heredity, variation, capacity for
mucous membrane revealed some oval reproduction
polymorphous Gram-positiv microorganisms B. Parasitic mode of existence, heredity,
arranged in clusters. What microorganism variability, inability to binary fission
may be the causeof such manifestations? C. Parasitic mode of existence, heredity,
A. S.aureus variation, the ability to have only one type of
B. C. tropicalis nucleic acids
C. С.albicans D. Heredity, capacity for reproduction,
D. S.pyogenes parasitic way of existence, the ability to have
E. C.pylori only one type of nucleic acids
E. The ability to have only one type of nucleic
Methods of viruses cultivation, indication acid, variability, capacity for reproduction,
and identification parasitic way of existence
526. Viruses don’t have a cellular structure, 527. Viruses belong to the Vira kingdom .
but they carry the basic functions of living What characteristics distinguish them from
viruses pro-and eukaryotes?

58
A. Do not have a cell structure capable of 533. Most effective in combating viral
growth and binary fission have only one type infections, which have aerogenic transfer
of nucleic acids mechanism are:
B. Do not have a cellular structure, not A. Measures aimed at the source of infection
capable of growth and binary fission,they B. Measures, interrupt transfer mechanism
have only one type of nucleic acids C. Measures to create a specific immunity
C. They have both types of nucleic acids, 534. Material from patient A., 4 years, with
have no cell structure, not capable of binary suspected enterovirus infection were infected
fission with cell culture to virus accumulation. What
D. They able to grow and to binary fission, type of tissue culture is represented by tumor
have cellular structure, have only one type of cells and is able to divide indefinitely?
nucleic acids A. Transplantable cell line
E. Do not have a cellular structure, have both B. Organ culture
types of nucleic acids, capable of growth and C. Culture of primary cells
binary fission D. Diploid cell lines
528. Dimensions of virions different viruses E. None of these
vary widely:
A. From 45 to 500 nm Laboratory diagnostics of Ortomyxoviral,
B. From 0,5 to 500 nm Paramixoviral and Rhabdoviral infections.
C. From 25 to 600 nm Lesions of the oral cavity under conditions
D. From 15 to 400 nm of influenza and measles.
E. From 5 to 200 nm 535. In the laboratory smears were taken of
529. Capsids of virions have a clearly ordered the mucous membrane of the nasal cavity in a
structure. What are the types of symmetries patient with suspicion of influenza. What are
underlying? the methods of investigation must be carried
A. Rhabdoid, combined out to identify the virus?
B. Spiral, spherical A. Immunoassay analyze
C. Spiral, cubic, combined B. The reaction of agglutination
D. Cubic, Spermatozoon-shaped C. The reaction of precipitation
E. Spermatozoon-shaped, spherical D. Reaction of indirect hemagglutination
530. Due to the interaction of virus with the E. The reaction of complement fixation
host cell can develop productive cell type of 536. The high variability of influenza A
virus infection. Thus there: viruses, implemented through the mechanism
A. Integration of viral and cellular genome of the "shift" is associated with features of the
B. Fine virus virus
C. Inhibition of virus reproduction A. The virus has a fragmentirium genome
D. B. The virus has a negative RNA
E. Enhancement of virus reproduction C. The virus will infect both human and
531. The bacteria is called lysogenic: animal
A. In condition of phages’s reproduction D. The structure of the virion consists of
B. Lysed in the allocation of phages lipids and carbohydrates
C. Infected with virulent phages E. The virus has spiral RNA
D. Have on their surface adsorbed phages 537. Infection of chick embryos is the main
E. Have in its genome prophage method of isolation of influenza virus. When
532. To isolate the virus used chick embryos injected into the chicken embryo test material
aged: (nasopharyngeal wash) is added prior to it:
A. From 3 to 10 days A. Streptomycin and penicillin
B. From 6 to 15 days B. Periyodat of potassium
C. From 10 to 15 days C. A solution of Needle
D. From 5 to 20 days D. Ethanol
E. From 10 to 20 days E. Ether
538. On the basis of clinical symptoms and

59
epidemiological data, the doctor put the A.Live vaccine
patient the diagnose of the flu. Which family B. Killed vaccine
contains viruses? C. Anatoxin
A Picornavirus D. Chemical vaccine
B. Herpes E. Antiidiotypic vaccine
C. Gepadnovirus 544. For the specific prophylaxis of influenza
D. Ortomiхovirus among students and those working in
E. Togavirus enterprises in the service is used:
539. A pediatrician, conducting a A. Live vaccine
conversation with parents about measles, B. Killed vaccine
noted that a certain category of children has a C. Anatoxin
natural passive immunity to the disease. D. Chemical vaccine
Which ones children had a physician in mind? E. Antiidiotypic vaccine
A. Over 14 years 545. The composition of the outer shell
B. Newborns processes of spikes flu include:
C. Those who had measles in the first year of A. gp120, gp41
life B. p24, p17
D. Those who received routine immunizations C. Hemagglutinin, neuraminidase
E. Those whose parents had not measles D. Hemolysin, coagulase
540. Pathological material (nasopharyngeal E. Hemagglutinin, hialuronidase
mucus) was taken from a patient with 546. The composition of the outer shell
suspicion of flu, which have infected the processes spikes parainfluenza virus include:
chicken embryos in chorion-alantoisnus A. gp120, gp41
cavity. Through which the reaction is the B. p24, p17
most expedient to prove that HAP actually C. hemagglutinin, neuraminidase
accumulated the influenza virus and to D. hemolysin, coagulase
determine the type of virus? E. hemagglutinin, hialuronidase
A. Precipitin 547. For the specific prophylaxis of
B. Double immunodiffusion parainfluenza is used:
C. Immunofluorescence A Live vaccine
D. Нemagglutination B. Killed vaccine
E. Neutralization C. Anatoxin
541. Typical influenza virus belonging is D. Chemical vaccine
determined: E. Is not developed
A. The neutralization reaction 548. Mumps virus belongs to the family:
B. The reaction of complement fixation A Picornavirus
C. The reaction of agglutination B. Rhabdovirus
D. The reactions of precipitation C. Ortomyxovirus
E. The reaction of hemagglutination - D. Paramyxovirus
inhibition E. Rhinoviruses
542. The serological reactions are used to 549. Transmission of mumps:
establish the subtype of the hemagglutinin of A. Alimentary
influenza virus: B. Sexual
A. The reaction of agglutination C. Respiratory
B. The reaction of precipitation D. Parentheral
C. The reaction of complement fixation E. Tranplacental
D. The reaction of neutralization 550. Entrance gate of infections with mumps:
E. The reaction of hemagglutination - A. The mucous membrane of the nasopharynx
inhibition B. The salivary glands
543. For the specific prophylaxis of influenza C. The mucosa of the intestine
in people aged 65 and older with chronic, D. Damaged skin
cardiovascular disease is used: E. The mucosa of genital organs

60
551. For mumps the complications are 558. For a specific treatment for measles
characterized by: apply:
A cardiovascular system, pancreas, testes, A. Live vaccine
B. Liver, thyroid, testicular B. Killed vaccine
C. The nervous system, mammary glands, C. Anatoxin
testes, D. Immunoglobulin
D. The thyroid and pancreas, liver E. Is not developed
E. Cardiovascular system, mammary glands, 559. Cytopathic effect of mumps virus to
testes, sensitive tissue culture is characterized by:
552. For the specific prevention of mumps A. The appearance of simplast with multiple
shall apply: cores
A. Live vaccine B. destruction and formation of granulation in
B. Killed vaccine the infected cells
C. Anatoxin C. Improving the refraction of affected cells,
D. Chemical vaccine which are then rounded and separated from
E. Is not developed the
553. The measles virus belongs to the family: glass
A. Ortomyxovirus D. The formation of giant cells, which have
B. Рaramyxovirus intranuclear inclusions
C. Picornaviruses E. Stimulation of cell proliferation
D. Poxviruses 560. In a study of the sick 5 years old child by
E. Rhabdovirus the dentist on the basis of clinical data has
554. The main route of transmission of been diagnosed with mumps. Which of the
measles is: following methods can be transmitted the
A. Respiratory disease?
B. Alimentary A. Airborne
C. Parentheral B. Transmissible
D. Contact-home C. Animals through the bite
E. Tranplacental D. Through fecal-oral
555. Infectious period for measles is: E. Parentheral
A. The last 5 days of incubation, prodrome, 561. To prevent seasonal flu rise in hospitals
the period of eruption of the city Sanitary Epidemic Station obliged
B. The last day of incubation, prodrome to immunize health care workers. What of the
C. Prodromal period, the period of eruption following drugs should be immunized?
D. The last day of incubation, prodrome, the A. Subunit vaccine
period of eruption B. Interferon
E. The last 5 days of incubation, prodrome C. Gamma globulin
556. An early pathogenesis symptom for D. Remantadin
measles is to identify: E. Amantadine
A. Cells of Babesh- Negri 562. In the infectious diseases hospital the
B. Spots of Bielski-Filatov-Koplik patient admitted with signs of pneumonia,
C. Granule of Babesh-Ernst which developed on the 6th day of the flu.
D. Cells of Guarnieri Which method is most authentically
E. Listing of Prowazeki-Galbertshedter confirmed influenza etiology of pneumonia?
557. For the specific prophylaxis of measles A. Detection of influenza virus antigens in
apply: sputum by ELISA
A. Live vaccine B. The study paired sera
B. Killed vaccine C. Infection of chick embryos
C. Anatoxin D. Immunofluorescent study smears from the
D. Chemical vaccine nasal passages
E. Is not developed E. Detection of antibodies against influenza
virus hemagglutinin

61
563. During an outbreak of acute respiratory hemagglutination properties and causes the
infection in order to establish the diagnosis of formation of simplast in cell culture. What
influenza rapid diagnosis is carried out, based other organs are most likely to be affected as
on the identification of specific viral antigen a consequence of infection with this virus?
in the test material (nasal wash). What A. Lungs
serological tests used for this? B. Liver
A. Immunofluorescent assay C. Gonads
B. The reaction of complement fixation D. pharyngeal tonsils
C. The reaction of agglutination E. Brain
D. The reaction of precipitation 568. The epidemic of flu is in the city. Which
E. The reaction of opsonization of the following drug can be recommended to
564. Serological diagnostics of influenza people for non-specific prevention of the
involves the identification of growth titer of disease?
antibodies to the patient serum. How many A. Leukocyte interferon
times should the antibody titer increase in B. Flu vaccine
paired sera,that results are considered C. Penicillin
credible? D. Influenza immunoglobulin
A. In 3 times E. Flu serum
B. In 2 times 569. To prevent seasonal flu which rises in
C. In one time the educational institutions of the city-station
D. In 4 or more times must hold an active immunization of children
E. In a half-titer and adolescents. What of the following drugs
565. The influenza virus contains an internal should be immunized?
antigens - the nucleoprotein (NP), polymerase A. Killed vaccine
(P1, P2, P3), matrix protein (M) and external B. Normal human immunoglobulin
antigens - hemagglutinin (H) and C. Rimantadine
neuraminidase (N). Some of them a key role D. Oxolin
in establishing immunity to influenza E. Interferon
infection? 570. In the kindergarten routine measles
A. Hemagglutinin and neuraminidase vaccination were conducted. What method
B. Nucleoprotein antigens can verify the formation of postvaccinal
C. Matrix protein immunity?
D. polymerase proteins A. Serological
E. Neuraminidase B. Virological
566. Post-mortem department of infectious C. Bacteriological
diseases hospital the corpse of a dead person D. Bacterioscopic
was suddenly sent with a clinical diagnosis E. Allergic
"flu." What research is needed to confirm 571. When virusoscopy cell monolayer,
specified for a reliable diagnosis? infected with infectious material,
A. Electron microscopic detection of viruses physician assistant diagnosed the respiratory-
B. Isolation and identification of influenza syncytial virus infection. What changes will
virus this virus cause in cell culture?
C. Detection of intracellular inclusions at light A. Total destruction of the cell monolayer
microscopy B. Round cellular degeneration
D. Isolation of high titers of antibodies in the C. The formation of polynuclear cells
reaction of hemagglutination -inhibition D. The presence of cell Babesh- Negri
E. Isolation of antibodies in the complement- E. Detachment of the monolayer
fixation 572. The department has increased
567. The boy of 6 years old of moderate dramatically the incidence of newborn
fever, enlarged parotid glands. A virus of children SARS, caused by the different
saliva was isolated that replicates in chicken groups of viruses. In order to prevent the
embryos or cell cultures, has spread of infection are recommended

62
appointment of human leukocyte interferon. inhibition (RGGA)
Which way is this product introduced? B. The reaction of agglutination (PA)
A. Subcutaneously C. The reaction of indirect hemagglutination
B. In the nasal passages (Phragmites)
C. Inhalation D. The reaction of hemagglutination (RHA)
D. Oral E. The reaction of immune hemolysis
E. Intramuscularly 578. In the diagnosis of salivary gland
573. What strain of influenza virus with a inflammation in the patient was isolated
pandemic antigenic structure is caused now? mumps virus. Which family is this a virus?
A H1N1 A Paramyxovirus
B. H3N4 B. Ortomyxovirus
C. H1N2 C. Picornavirus
D.H2N3 D. Herpes simplex virus
E. H2N2 E. Adenoviruses
574. Boy, 1.5 years old, who did not receive 579. For effective vaccination is necessary to
routine vaccinations in contact with the provide probable virus that causes epidemic.
measles. For the purpose of emergency As a rule, the agent is the variant of the virus
specific prevention of child was entered donor against which most people have no
gamma globulin. What type of immunity was antibodies. What kind of reaction is advisable
established at the same time? to apply for the detection of antibodies in the
A. The natural sera of people?
B. Passive A Complement fixation
S. antitoxic B. The indirect hemagglutination
D. Post-vaccination C. Passive hemagglutination
E. Local D. Neutralization of the cytopathogenic
575. What type of influenza virus has the E. Нemagglutination inhibition
greatest antigenic variation? 580. From individuals with respiratory
A. A infection clinic, residents of south-east Asia,
B. B has been isolated influenza virus from sharp
C. C changes in the surface glycoproteins -
D. D hemagglutinin and neuraminidase. The
E. E laboratory staff regarded the virus as a virus
576. A child 7 years has acute illness. During with pandemic potency. In the result of which
the inspection it was revealed: that the process could form an antigenic variant of
hyperemia and edema of the pharyngeal influenza virus?
mucosa, which is covered with lots of mucus. A. Antigenic shift
On the mucous membrane of cheeks whitish B. Antigenic drift
spots. Then a blotchy rash appeared on the C. Conjugation
child’s face, neck and body. What is the D. Transduction
disease most likely evolved in this case? E. Transformation
A. Meningococcemia 581. In a 32-year-old man 'flu-like syndrome"
B. Measles developed with fever, headache, sore throat
C. Atopic dermatitis and muscle aches. Which of the following is
D. Scarlet fever the best studied material for virus isolation -
E. Diphtheria the agent of this infection?
577. From the patient's clinical material (flush A. Feces
with the nasopharynx), with a preliminary B. Nasopharyngeal wash
diagnosis of influenza epidemiology, conduct C. Urine
virus isolation in chicken embryo. Which of D. Blood
these reactions can be used to detect the virus E. Saliva
in the chick embryo? 582. The kindergarten child got sick of
A. The reaction of hemagglutination measles. What medication can prevent

63
measles outbreak in a group? D. By Turevich
A Human immunoglobulin, normal E. By Peshkov
B. Measles vaccine 589. For specific rabies is applied:
C. Sulfonamides A. Killed vaccine
D. Interferon B. Live vaccine
E. Antibiotics C. Anatoxin
583. The patient has the flu where a defeat of D. Chemical vaccine
the epithelial cells of the nasal mucosa is, E.Antiidiotypic vaccine
which helps activate the body autoflora and 590. The patient turned to the clinic because
the emergence of secondary bacterial of dog’s bites. The dog managed to catch it,
infection. What kind of bacteria can cause and it turned out that the animal is sick with
complications? rabies. Which vaccine should be used for
A. Coli bacillus specific prophylaxis of rabies in humans?
B. Pseudomonas aeruginosa and Proteus A. Chemical
C. Bacillus and Clostridium B. Anatoxin
D. Staphylococcus and Streptococcus C. Living
E. Fuzobacteries and Treponema D. Recombinant
584. The incubation period is a minimum in E. The synthetic
the penetration of the agent of rabies through 591. A man turned to the surgery who was
the damaged skin: bitten by unknown dog. Extensive lacerations
A. Head are located on the face. What is the treatment
B. Neck and prophylaxis care is required to provide for
C. Upper limb the prevention of rabies?
D. Body A. Post immunized with rabies vaccine
E. The lower extremities B. Assign a combined antibiotic
585. The incubation period is the maximum C. Urgently introduce DTP
penetration of the agent of rabies through the D. To hospitalize and keep under the
damaged skin: supervision of a physician
A. Head E. Extra enter the normal gamma-globulin
B. Neck 592. For which of these diseases are
C. Upper limb characterized by the cell of Babesh- Negri in
D. Body host cells?
E. The lower extremities A. Aseptic meningitis
586. For the laboratory diagnostics of rabies B. Rabies
the methods are used: C. Congenital Rubella
A Histological, allergic, biological D. mumps
B. Immunofluorescent, histological, E. Infectious mononucleosis
biological 593. The rabies virus belongs to the family:
C. Serological, biological, and allergic A. Reovirus
D. Biological, allergic, histological B. Retrovirus
E. Allergic, serological, immunofluorescent C. Togavirus
587. Specific for rabies is to identify the cells: D. Picornavirus
A. Paschen E. Rhabdovirus
B. Babesh- Ernst 594. In Ukraine, the main source of human
C. Prowazek infection with rabies is:
D. Gvarnier A Sick Man
E. Babesh-. Negri B. Dogs
588. To detect the cells of Babesh-Negri by S. Wolf
histological method the staining is used: D. Fox
A. By Neisseria E. Cattle
B. By Gins 595. Entrance gates in rabies are:
C. By Morozov A. Upper respiratory tract

64
B. The intestinal tract of illness should think about?
C. Damaged skin A. Influenza
D. Reproductive organs B. Fever
E. The salivary glands C. Measles
596. The body of a sick man rabies virus D. Scarlet fever
stands out: E. Meningococcal disease
A. From the saliva 602. The patient has the flu and there is a
B. From the feces defeat of the epithelial cells of the nasal
C. From urine mucosa, which helps activate the body
D. From the nasopharyngeal mucus autoflora and the emergence of foci of
E. Not released secondary bacterial infection. What kind of
597. In the patient's body is fixed rabies virus bacteria can cause complications?
in the cells: A. Bacillus and Clostridium
A. Cardiovascular System B. Pseudomonas aeruginosa and Proteus
B. Nervous System C. Staphylococcus and Streptococcus
C. Liver D. Coli bacillus
D. Blood E. Fuzobacteries and Treponema
E. Skin 603. From individuals with respiratory
598. In setting up the FTA with the slices of infection clinic, residents of south-east Asia,
dog’s brain died because of the rabies has been isolated influenza virus from sharp
observed luminosity. What is defined for a changes in the surface glycoproteins -
given reaction? hemagglutinin and neuraminidase. The
A. The cells of Paschen laboratory staff regarded the virus as a virus
B. Antibodies with pandemic potency. As a result of what
C. The cells of Babesh- Negri process could form an antigenic variant of
D. Corn Fly influenza virus?
E. Corn volutin A. Transformation
599. The patient with a torn wound after B. Antigenic drift
being bitten by rabid animal admitted to the C. conjugation
hospital. Which vaccine should be introduced D. Transduction
for prophylaxis of rabies? E. Antigenic shift
A. Rabies vaccine 604. In a 32-year-old man 'flu-like syndrome
B. DTP "developed with fever, headache, sore throat
C. DT and muscle aches. Which of the following is
D. BCG the best material under study to isolate the
E. TABte virus agent of this infection?
600. Hunter turned for medical help because A. Nasopharyngeal wash
of being bitten by the wounded fox. What B. Feces
kind of help should be given to him in order C. Urine
to prevent rabies specific? D. Blood
A. Introduction of rabies vaccine E. Saliva
B. Surgical treatment of wounds 605. The kindergarten child got sick of
C. The introduction of antibiotics measles. What medication can prevent
D. Washing the wound with soap and water measles outbreak in a group?
E. Treatment of wounds with an alcoholic A measles vaccine
solution of iodine B. Human immunoglobulin normal
601. A child of 2 years of catarrhal C. sulfonamides
conjunctivitis, pharyngitis, D. Interferon
larengotraheobronhitis appeared., The white E. Antibiotics
spots have appeared on the mucous
membrane of the cheeks, and later spotted Laboratory diagnostics of HIV infection.
rash on the face, trunk, extremities. What kind The defeat of the oral cavity at AIDS

65
606. In a study of young man in the middle of antibody against the virus in the first stage?
the fight against AIDS has been obtained A. RIA
positive results with ELISA antigens of HIV. B. Immunoblotting
Complaints about the state of health are not C. ELISA
available. What may indicate a positive result D. FTA
of ELISA? E. RNTA
A. About HIV 613. What kind of protein of HIV is bounded
B. About AIDS the mechanism of reverse transcription?
C. About the HBV infection A. Endonuclease
D. About the disease of AIDS B. Protease
E. About the persistence of HBV S. Neyrominidaza
607. The diagnostics of HIV infection D. RNA polymerase
examined serum for detection of specific E. Revertaze
antibodies by ELISA. What enzyme labeled 614. Specify the transmission of HIV.
antibodies are used in this case? A. Respiratory
A. Against the gp120 protein B. Contact
B. Against HIV antigens C. Alimentary
C. Against the human immunoglobulin D. The parenteral
D. Against protein gp 17 E. Transmissible
E. Against protein gp 41 615. At the initial examination of blood
608. Human immunodeficiency virus differs donors at blood transfusion stations ELISA to
from other viruses by: detect antibodies to human immunodeficiency
A. The complexity of the structure virus with serum from one of them produced
B. The presence of reverse transcriptase a positive result. Which method is
C. The ability to integrate into the genome of recommended to confirm the diagnosis of
the cells HIV infection?
D. The presence of two types of nucleic acids A. Electron Microscopy
- DNA and RNA B. Immunoblot (immunoblotting)
E. The ability to grow in chick embryo C. Enzyme Immunoassay
609. The main method of laboratory D. Immunofluorescence
diagnostics of HIV infection is E. Clinical and immunological study
A. Allergic 616. At autopsy of the deceased in the skin of
B. Virusological distal lower extremities revealed crimson-red
C. Biology patches, plaques, and nodes (Kaposi's
D. Serological sarcoma). Identified as an acute pneumonia
E .Electron microscopy caused by pnevmotsistami. What disease is
610. Target cells for HIV are: characterized by the changes?
A. T-killers A. Influenza
B. T-suppressor B. Diphtheria
C. T-helper C. Measles
D. B-lymphocytes D. Anthrax
E. NK-cells E. AIDS
611. After infection with HIV the antibodies 617. HIV belongs to the family:
appear in serum in: A Picornavirus
A. 2-3 weeks B. Reovirus
B. 3-6 months C. Rhabdovirus
C. 10-12 months D. Retroviruses
D. 3-6 years E. paramyxovirus
E. 8-10 years 618. HIV belongs to the subfamily:
612. The patient was informed that A. Oncoviruses
serological testing for AIDS consists of two B. Lentiviruses
stages. What reaction is used to study serum C. Spumavirus

66
619. LTR of proviral HIV DNA encodes: C. 2, 5-3, 2
A. Internal structural proteins D. 3, 5-4, 2
B. Regulatory proteins E. 4, 5-5, 0
C .Type-specific protein of the outer shell 627. In AIDS patients are observed:
D. Do not encode any proteins A. The involution of parathyroid gland,
E. Virus specific enzymes inhibition of B-lymphocytes, reducing the
620. The gene env of HIV proviral DNA level of interleukin-2 and interferon
encodes: B. The involution of parathyroid gland,
A. Internal structural proteins activation of B cells, reducing levels of
B. Regulatory proteins interleukin-2 and interferon
C.Type-specific protein C. The involution of parathyroid gland,
D. Do not encode any proteins activation of B-lymphocytes, increased levels
E. Virus specific enzymes of interleukin-2 and interferon
621. The gene pol of HIV proviral DNA D. Hyperfunction of parathyroid inhibition of
encodes: B-lymphocytes, reducing the level of
A. Internal structural proteins interleukin-2 and interferon
B. Regulatory proteins E. The involution of parathyroid gland,
C .Type-specific protein of the outer shell activation of B-lymphocytes, increased levels
D. Do not encode any proteins of interleukin-2 and interferon
E. Virus specific enzymes 628. In AIDS patients are observed:
622. The greatest variability of proviral DNA A. Reduced levels of IgG and IgA, the
are the genes of HIV: number of immune complexes, interleukin-2
A env B. Reduction of IgG and IgA, increased
B. pol number of immune complexes, interleukin-2
C. gag C. Increased levels of IgG and IgA, the
D. sor number of immune complexes, interleukin-2
E. tat D. Increased levels of IgG and IgA, reduction
623. Very high level of variability of HIV is of immune complexes, Interleukin-2
associated with proteins: E. Increased levels of IgG and IgA, the
A. p24 number of immune complexes, reduction of
B. p18 interleukin-2
C. p13 629. The structure of HIV processes include:
D. gp120 A. gp120, gp41
E. gp160 B. Hemagglutinin, neuraminidase
624. Major role in the interaction of HIV with C. Hyaluronidase, coagulase
target cells is: D. Hemolysin, coagulase
A. p24 E. p24, p17
B. p18 630. For specific prophylaxis of AIDS is
C. gp41 used:
D. gp120 A. Live vaccine
E. gp160 B. Killed vaccine
625. In healthy people the ratio of T4/T8 is as C. Toxoids
follows: D. Chemical vaccine
A. 0, 2-0, 5 E. Not developed
B. 0, 7-1, 2 631. High risk for HIV infection include:
C. 1, 9-2, 4 A. Patients with chronic medical conditions
D. 2, 5-3, 2 B. Recipients of blood
E. 3, 5-4, 2 C. Patients with tuberculosis
626. In patients with AIDS the ratio of T4/T8 D. Parturient
is: E. Blood Donors
A. 0, 2-0, 9 632. If any clinical signs of disease and
B. 1, 9-2, 4 showed an HIV test:

67
A. Acute Diarrhea periodically for signs of revitalization
B. Acute pneumonia process. What are the most significant sign
C. The increase in groin that points to the transition of HIV infection
D. Increase in cervical lymph nodes to AIDS.
E. The increase in lymph nodes of several A. Kaposi's sarcoma. Number of T-helper
locations cells below 200 cells / mm
633. For the first time AIDS was registered as B. Reducing the number of neutrophils
an infectious disease in man: C. Reducing the number of T-helper
A. 1977 D. Number of T-helper cells below a critical
B. 1979 level
C. 1981 E. Detection of antibodies to gp 41
D. 1983 639. In a specialized clinic patients assigned
E. 1985 the combination of drugs that inhibit the
634. The results of the allocation of AIDS reproduction of HIV. Indicate which group
pathogen group of French scientists were first includes drugs that are necessarily
published in: comprehensive antiviral treatment.
A. 1977 A. Crixivan
B. 1979 B. Broad-spectrum antibiotics
C. 1981 C. Interleukin
D. 1983 D. nucleoside analogues
E. 1985 E. Biseptol
635. Priority in allocation of AIDS pathogen 640. A patient was treated for a long time for
belongs to a group of researchers led by: pneumonia of unknown etiology, resistant to
A. P. Frosch and T. Francis standard therapy. Anamnesis revealed that he
B. P.Parkmen and T. Uellerg was a long time on a business trip in the U.S..
C. G. Doldorf and G. Sikls While traveling he was injured, was treated in
D. L.Montan’ye and R. Gallo hospital after his recovery he returned home.
E. D. Enders and E. Popper Evaluating the medical history, clinical
636. In a patient with fever of unknown picture, the doctor suspected AIDS in
etiology, immunodeficiency, patients. Results of what laboratory diagnostic
damage to the nervous and digestive systems methods let confirm that pre-diagnosis in this
previously diagnosed with AIDS. What are patient?
the diagnostic methods must be used to A. Widal reaction
confirm the diagnosis? B. Immunosorbent assay
A. Immunofluorescence assay, C. Complement fixation test
immunoblotting, radioimmunoassay D. Electron Microscopy
B. Complement fixation test E. HAI-hemagglutination inhibition test
C. Agglutination test 641. The patient had been diagnosed AIDS
D. Reaction gemadsorbtion for 20 years -. What is the cell population
E. The reaction of hemagglutination most sensitive to human immunodeficiency
637. When testing blood donors for blood virus?
transfusion station in the serum of one of A. Epithelial cells
them had antibodies to human B. Hepatocytes
immunodeficiency virus. Which method is C. Endothelial cells
recommended to confirm the diagnosis of D. T helper
HIV infection? E. B-lymphocytes
A. Immunofluorescence 642. It is known that human
B. Electron microscopy immunodeficiency virus belongs to the family
C. ELISA retroviruses. Specify the main features which
D. Immunoblot (immunoblotting) characterized the family.
E. Immunoradiometric assay A. The presence of the enzyme reverse
638. HIV-infected patient is examined transcriptase

68
B. Contain less RNA A. HIV infection
C. Simple viruses that infect only human B. NLTV-1 infection
D. The nucleic acid does not integrate into the C. TORCH-infection
host genome D. HB - infection
E. The reaction of enzyme immunoassay E. ECHO-infection
antigen detection 648. After laboratory examination of patients
643. In carrying out Western blot test in with recurrent common viral, bacterial and
serum was found protein gp120. What illness fungal opportunistic infections diagnosed
occur this protein? "HIV infection". The results of investigations
A. Syphilis made it possible to put such a diagnosis?
B. Viral hepatitis B A. Immunosorbent assay.
C. Tuberculosis B. Complement fixation test.
D. HIV infection C. Gel precipitation test.
E. Polio D. The reaction of hemagglutination
644. The patients turned to the dentist with inhibition.
manifestations of oral candidosis, which E. The reaction of passive hemagglutination.
always recurs and no cure. When questioning 649. In a patient with clinical signs of
revealed that the patient has a long time there immunodeficiency were conducted
is fever, weight loss. What research should be immunological studies. Revealed a significant
done to the patient? decrease in the number of cells that form
A. Immunological and serological tests for rosettes with sheep erythrocytes. What
HIV infection conclusion should be based on data analysis?
B. Bacteriological studies on dysbiosis. A. Reduction of comlement
C. Select a pure culture of the pathogen and to B. Reduction of B-lymphocyte
investigate the sensitivity to antibiotics. C. Reduction of natural killer (NK-cells)
D. Check the status of humoral immunity. D. Reduction of T-lymphocytes
E. Investigate the level of specific antibodies E. Lack of effector cells of humoral immunity
to the fungus Candida 650. The patient has 25 years of multiple skin
645. Acquired immunodeficiency syndrome pustules planted Staphylococcus aureus in
(AIDS) is the human immunodeficiency association with Staphylococcus epidermidis,
virus. Which cells are targets for this virus?: the study of sputum revealed pnevmotsista
A. Eosinophils Carini in the faeces - cryptosporidium, vulgar
B. Natural killer cells Proteus and fungi of the genus Candida.
C. T-helper lymphocytes When a disease is found multiple infections
D. Basophiles opportunistic microorganisms?
E. Plasma A. AIDS
646. Human immunodeficiency virus, which B. Diabetes
has on its surface antigens gp 41 and gp 120 C. Sepsis
interacts with the target cells of the body. D. dysbacteriosis
Choose among these antigens of human E. Medical agranulocytosis
lymphocytes, which complementarily binds 651. Retroviruses are unique enzyme, which
gp 120 virus. distinguishes them from other viruses. Which
A. CD 3 of the following enzymes of retroviruses
B. CD 4 replicate?
C. CD 8 A. RNA-dependent DNA polymerase
D. CD 19 B. DNA-dependent RNA polymerase
E. CD 28 C. DNA polymerase
647. In the study of blood serum in patients D. RNA polymerase
with signs of immunodeficiency were E. Restrictase
identified antibodies of proteins gp 120 and 652. To the city center of research an
gp 41. What infection in a patient confirms anonymous citizen N. has addressed to
this result? conduct a survey on HIV infection, after

69
which the patient's blood was sent to the C. The reaction of hemagglutination
laboratory for serological diagnosis. Which of inhibition
these reactions must take place in this case? D. The reaction of hemagglutination
A. Enzyme Immunoassay 657. In the dental clinic, the Laboratory was
B. The reaction of indirect hemagglutination provided for diagnosis of HIV infection. What
C. The reaction of complement fixation diagnostic tools must a laboratory be
D. Immunofluorescence reaction equipped with?
E. The reaction of neutralization A test system for detection of specific
653. Patient at the age of 22 was hospitalized antibodies by ELISA
at the clinic complaining of a sharp weight B. Cell cultures for virus isolation
loss, general weakness, swollen lymph nodes. C. Sets of special culture media
Over the past few months suffering from D. Standard diagnostic sera
diarrhea. Laboratory researches established E. Antigen and hemolytic system to
radiation in the faeces in large numbers is complement-fixation
found Cryptosporidium. In a detailed clinical 658. AIDS patient asked the dentist about the
study of AIDS, diagnosis was proved. Which destruction of the oral mucosa. The defeat
of the immunological characteristics are was manifested with white plaques, which are
important for this disease? merged into a continuous "layering cheese",
A Сhange in the correlation between T-and B- usually near the corners of the mouth and on
lymphocytes the back of the tongue. What is the diagnosis
B. The sharp decline in activity of the of such lesions of the oral mucosa of AIDS
complement system posed a doctor?
С. Lymphopenia A. Candidosis
D. Polyclonal activation B - lymphocytes B. Herpetic stomatitis
E. The absolute reduction in the number of T C. Necrotizing stomatitis Vincent
- helper cells D. Foot and mouth disease
654. In a patient with clinic pneumonia E. Gonococcus stomatitis
physician suspected HIV - infection. What 659. In the dental patient, a man at the age of
laboratory tests can confirm a diagnosis of 49, has been observed for necrotizing
"HIV - infection" in this patient? gingivitis, the defeat of the "thrush" caused by
A. ELISA screening of serum Candida, dry mucous membranes, and
B. Bacteriological examination of sputum bilateral tumors salivary glands. Consequence
С. Mycological examination of sputum of which immune disorders most likely to be
D. Virological examination of sputum that defeat?
E. Electron examination of sputum A. AIDS
655. A young man at the age of 25 was first B. Primary immunodeficiency combinable
applied to the study of trust with the request C. Allergic disease
to delete his HIV infection. What reaction can D. Autoimmune disease
be used to diagnose HIV infection in this E. Selective gammaglobulinemia
patient? 660. In connection with cases of diagnosis of
A. Enzyme immunoassay (ELISA) HIV infection in the dental clinic a meeting
B. Reaction immunoblotting was held, at which the recommendations were
С. Radioimunny analysis (RIA) made by doctors, epidemiologists, AIDS
D. Immunoelectronnic microscopy (IEM) Prevention Center. What preventive measures
E. Immunofluorescence reaction (IFA) could dentists recommend preventing the
656. After the examination of patients with possibility of professional infection?
recurrent aphthous stomatitis and candidosis A. The requirement of each patient's
associated physician decided to exclude HIV presenting medical certificate
infection. What research will help clarify the B. Wash your hands after each patient
situation and a preliminary diagnosis? C. For each patient, using a new sterile set of
A. Enzyme Immunoassay instruments
B. Gel precipitation test D. To work in personal protective equipment

70
(gloves, goggles) A Picornavirus
E. To quartz the room B. Ortomyxovirus
661. What method of testing blood donations C. Poxvirus
should be applied to ensure the absence of her D. Filovirus
immunodeficiency virus? E. Flavivirus
A. Electronic microscopic study of T4 666. For serological diagnosis of polio
lymphocytes examined the paired sera of the patient. What
B. Detection of the virus by infecting cultures should be used as an antigen in the
of lymphocytes neutralization of the cytopathogenic?
C. Detection of IgM antibodies A. Antigens - haemagglutinin
D. To conduct nucleic acid agent B. Complement-antigens
E. Identification of pathogen antigens C. The antigens inactivated with formalin
662. In an observational branch of the D. Living the three types of viruses
hospital HIV-infected women gave birth to a E. Antigens of the capsid proteins of the virus
child. The use of which of the following 667. A child at the age of 5 hospitalized at the
methods of diagnosis of HIV infection in the 2nd day of illness with complaints of fever,
newborn is the most appropriate? weakness, gastrointestinal upset. In history:
A. RIA contact with patients with poliomyelitis.
B. ELISA Preliminary diagnosis: polio. What is the
C. RVN pathological material taken for laboratory
D. DCs testing and what method is used for
E. PCR microbiological diagnosis?
663. At what stage of HIV infection is a A Virological, stool.
source of human infection? B. Liquor. Virological
A. At all stages of HIV infection C. Blood. Biological
B. Acute HIV infection D. Stools.
C. Chronic (latent) HIV infection E. Blood. Virological
D. Persistent generalized HIV 668. For the specific prevention of polio are
E. AIDS - associated complex used:
A. Ribosomal vaccine
Laboratory diagnosis of enterovirus, B. Live vaccine
aftovirus and coronavirus infections. The C. Toxoids
defeat of the oral cavity in terms of D. Bacteriophage
enterovirus infection, foot and mouth E. Chemical vaccine
disease 669. Enteroviruses belong to the family:
664. In the infectious diseases hospital A Picornavirus
admitted the patient with clinical signs of B. Poxvirus
encephalitis. In the history – the tick bite. In C. Paramyxovirus
hemagglutination-inhibition tests revealed D. Ortomyxovirus
antibodies against the pathogen in tick-borne E. Reovirus
encephalitis dilution of 1:20. Enter the 670. Polio viruses belong to the genus:
following steps after receiving the doctor's A. Rhinoviruses
results: B. Cardiovirus
A. Use a more sensitive reaction C. Aftovirus
B. Use this serum again D. Rubivirus
C. Repeat the study with serum taken after 10 E. Enterovirus
days 671. What is the serotype of the virus of polio
D. Repeat the study with other diagnostic often causes epidemics:
E. Confirm the diagnosis of tick encephalitis A.I
665. Circulation in the nature of vertebrates B. II
and arthropods transmissible transmission C. III
characteristic of the man: 672. Poliomyelitis virus has organ tropism:

71
A. Cardiovascular System C. The respiratory system
B. Nervous System D. Gastrointestinal tract
C. Salivary glands E. Multiorgan tropism
D. Liver 678. For the specific prophylaxis of diseases
E. Blood caused by Coxsackie virus is used:
673. Properties of poliovirus vaccine Sebina: A. Live vaccine
A. Provide general humoral immunity but do B. Killed vaccine
not provide local immunity of the intestine, is C. Toxoids
introduced parenterally D. Chemical vaccine
B. Do not provide a general humoral E. Not developed
immunity, but provide local immunity of the 679. ECHO viruses belong to the genus:
intestine, enter per os A. Rhinoviruses
C. Do not provide a general humoral B. Enteroviruses
immunity, but provide local immunity of the C. Cardiovirus
intestine, is introduced parenterally D. Aftovirus
D. Provide a common humoral and local E. Rubivirus
immunity of the intestine, is introduced 680. For the specific prophylaxis of diseases
parenterally caused by viruses ECHO apply to:
E. Provide a common humoral and local A. Live vaccine
immunity of the intestine, enter per os B. Killed vaccine
674. The properties of the Salk polio vaccine: C. Toxoids
A. Provide general humoral immunity but do D. Chemical vaccine
not provide local immunity of the intestine, is E. Not developed
introduced parenterally 681. The main route of transmission of
B. Do not provide a general humoral diseases that are caused by viruses of ECHO:
immunity, but provide local immunity of the A. The parenteral
intestine, enter per os B. Alimentary
C. Do not provide a general humoral C. Contact and household
immunity, but provide local immunity of the D. Sexual
intestine, is introduced parenterally E. transplacental
D. Provide a common humoral and local 682. ECHO viruses have tropism for Organ:
immunity of the intestine, is introduced A. Cardiovascular System
parenterally B. Nervous System
E. Provide a common humoral immunity but C. The respiratory system
do not provide local immunity of the D. Gastrointestinal tract
intestine, enter per os E. Multiorgan tropism
675. Coxsackie viruses belong to the genus: 683. What is the mechanism of transmission
A. Rhinoviruses of polio:
B. Enteroviruses A. Spray
C. Cardiovirus B. Fecal-oral
D. Aftovirus C. Transmissible
E. Rubivirus D. Contact
676. The main route of transmission of E. Vertical (transplacental)
diseases caused by Coxsackie virus: 684. The patient has 5 year old boy on the
A. Alimentary fifth day after the onset of nasal mucosa and
B. The parenteral tonsils isolated agent of poliomyelitis. What
C. Sexual kind of reaction should be used to determine
D. Contact-home the serotype of agent?
E. Transplacental A. The reaction was neutralized viral
677. Coxsackie virus has tropism for Organ: cytopathic effect
A. Cardiovascular System B. Haemagglutination-inhibition tests
B. Nervous System C. Reaction inhibition gemadsorbtion

72
D. Indirect hemagglutination reaction E. None of these
E. Complement fixation test 689. The kindergarten children performed
685. The stools are sick with suspected vaccination to prevent polimielita. The
intestinal virus infection treated with vaccine was administered orally. Synthesis of
antibiotics for one hour at 40C. Then a antiviral immunoglobulins which class will
suspension of infected primary and dominate the response to the introduction of
continuous cell culture. After 2-3 days in the this vaccine?
infected cells showed cytopathic effect of A. Іg A
cultures. How is the identification of B. Іg M
enteroviruses held? C. Іg E
A. Immunofluorescence reaction D. Іg D.
B. Through neutralization of the cytopathic E -----------
type-specific enterovirus sera 690. Child, 5 months, carried out vaccination
C. Through the haemagglutination-inhibition for prevention of polio vaccine by oral
tests administration. What does the vaccine have in
D. Use of agglutination its composition?
E. With the precipitation reaction A. Formalin Killed viruses
686. From the patient with acute intestinal B. Live viruses
infection the virus was divided which is C. heat-killed virus
referred to the genus Enterovirus. To establish D. Elements of the viral capsid
the serotype of the virus diagnostic serum is E. Virus reverse transcriptase
used. Indicate which of these antibodies 691. As a consequence of contamination of
should contain them? cell cultures of pathological material, which is
A. Against viral enzymes taken from a sick child, isolated type I
B. Against the protein supercapsid shell poliovirus? What is characteristic of this type
C. Against non-structural proteins of the virus of virus is responsible?
D. Against the capsid proteins A .DNA containing a simple virus
E. Against viral haemagglutinin B. Retrovirus
687. In the children's infectious disease C. RNA - containing a simple virus
section of the city hospital was hospitalized D. DNA - containing complex virus
group of children with pre-set clinical E. RNA - containing complex virus
diagnosis of "gastroenteritis rotavirus." What 692. Go to the dentist asked the patient (the
is the basic principle laboratory diagnostic milkmaid), with precipitation in the oral
applies in this case? mucosa in the form of the AFL. The doctor
A. Detection of viral antigen in the test revealed skin rash hands around the nail plate.
material What pathogen caused the disease?
B. Identification of specific inclusions in the A. The virus Coxsackie B
cells. B. Herpesvirus
C. Infection of chick embryos C. Vesicular stomatitis virus
D. Isolation of the virus in cell culture with D. FMDV
subsequent identification E. Cytomegalovirus
E. Infection of laboratory animals
688. Material from the patient A. 4y.o. with Laboratorial diagnostics of hepatitis A, B,
suspected enterovirus infection was infected C, D, E
with cell culture for the purpose of 693. In epidemiology of which from viral
accumulation of the virus. What type of tissue hepatitis is there the expressed seasonality?
culture cells was presented tumor and is able A. Hepatitis G
to divide indefinitely? B. Hepatitis B
A. Inculcate cell lines C. Hepatitis C
B. Organ Culture D. Hepatitis D
C. Culture of primary cells E. Hepatitis A
D. Lines of diploid cells

73
694. The main way of transmission of e. Salivary glands
hepatitis A is: 700. The viral hepatitis A usually damage:
A. Inoculable a. Children before 14 years old
B. Airborne b. Adults outrank 30 years old
C. Parenteral c. Young people and adults (15-30 years
D. Contact old)
E. Alimentary d. Elderly people (much than 60 years old)
695. The virus of hepatitis A belongs to the e. Age dependence does not exist
family: 701. Mechanism of HAV action to
A. Picornaviruses hepatocytes and clinical manifestation of
B. Poxviruses viral hepatitis A are:
C. Раrаmyxoviruses a. Has direct cytopatical action to
D. Orthomyxoviruses hepatocytes, serious forms of illness -
E. Reoviruses acute or chronic with early formation of
696. The virus of hepatitis A belongs to the hepatic cirrhosis
genus: b. Has direct cytopatical action to
A. Rhinoviruses hepatocytes. Often disease is over with
B. Cardioviruses full recovery
C. Aphthoviruses c. The mechanism of action is unknown, 50
D. Enteroviruses % of all forms of a hepatitis pass in the
E. Rubiviruses chronic form, and usually it leads to
697. Resistance of virus of hepatitis A to the cirrhosis and to a liver cancer
factors of environment is: d. The mechanism of action is unknown; the
A. Unstable to the high temperature, to the serious form of infection may be
low temperature, to the action of chlorine especially at pregnancy
B. Unstable to the high temperature, to the e. Has meditational immunological action to
action of chlorine, proof to the low hepatocytes; usually chronically carriage
temperature is characterized as a persistent or
C. Proof to the high temperature, to the low chronically active hepatitis; may be
temperature, unstable to the action of chlorine without symptoms
D. Proof to the high temperature, unstable to 702.A patient with hepatitis A is hospitalized
the low temperature, to the action of chlorine to the infection diseases hospital. What
E. Proof to the high temperature, to the low types of antibodies will be synthesized
temperature, to the action of chlorine first during the reproduction of this virus?
698. In what periods of disease a a. IgA
patient on viral hepatitis A is most dangerous b. IgM
as a source of infection? c. IgE
a. At the end of latent period d. IgG
b. In full play of illness with the period of e. IgD
jaundice 703. 703.There is a hepatitis eruption is fixed in a
c. At the end of latent period and in full play settlement. The water factor of transmission is
of illness before jaundice appearance the main. What kind of viruses can be a cause
d. At the end of latent period and in full play of eruption?
of illness with the period of jaundice a. Hepatitis D
e. At the end of latent period, during all b. Hepatitis C
illness c. Hepatitis B
699.The infection atrium for hepatitis A virus d. Hepatitis A
is: e. No one of hepatitis A or hepatitis B
a. Mucous membrane of respiratory tracts 704.Three samples of blood sera were
b. Mucous membrane of digestive tract delivered to the laboratory: from the
c. Mucous membrane of urinary system patient with chronic hepatitis B, second
d. Skin one recovered after hepatitis B and third

74
one without hepatitis B symptoms. What b. Alimentary
kind of antigens will be revealed in all of c. Contact
samples? d. Sexual
a. HBsAg e. Transplacental
b. HBeAg 711.Patient after a motor-car accident was
c. HBcAg delivered to the hospital. He needs blood
d. HBcAg and HBeAg transfusion. What reaction is it possible
e. HBcAg and HBsAg to use for a presence of the hepatitis B
705.The main way of hepatitis D transmission virus antigen in a donor blood revealing?
is: a. Inhibition of hemagglutination test
a. Transplacental b. Immunoenzymatic test
b. Alimentary c. Inhibition of hemadsorption test
c. Contact d. Indirect-hemagglutination test
d. Sexual e. Complement-connecting test
e. Parenteral 712.During an operative intervention a
706.The main way of hepatitis E transmission patient had blood transfusion. In his
is: blood the presence of the following
a. Parenteral causative agent should be checked:
b. Alimentary a. Hepatitis A virus
c. Contact b. Enteroviruses
d. Sexual c. Hepatitis B virus
e. Transplacental d. Hepatitis C virus
707.The main way of hepatitis C transmission e. Adenoviruses
is: 713.The antigen structure of hepatitis B virus
a. Parenteral consists of antigens:
b. Alimentary a. HBaAg, HBbAg, HBcAg
c. Contact b. HBaAg, HBcAg, HBsAg
d. Sexual c. HBcAg, HBeAg, HBsAg
e. Transplacental d. HBaAg, HBcAg, HBdAg
708.The causative agent of hepatitis D (delta- e. HBaAg, HBeAg, HBsAg
agent) is a defective virus. It can 714.HBsAg of viral hepatitis B is detected in:
replicate only in ceils already infected a. Patient’s excrements
with one of the viruses. With which one? b. Saliva
a. By virus of hepatitis B c. Mucous of nasopharynx
b. By virus of hepatitis A d. Urine
c. By Epstein-Barr virus e. All biological fluids
d. By virus of hepatitis E 715.Resistance of hepatitis B virus to factors
e. By HIV of external environment is:
709.A pat ient with a very grave course of a. Nonresistant to high and low temperature,
hepatitis B has been administered the to disinfectant substances
examination for detecting the possible b. Nonresistant to high temperature and
concomitant agent, which complicates disinfectant substances; resistant to low
the course of the basic disease. What temperature
agent is this? c. Resistant to high and low temperature, to
a. Virus of hepatitis A disinfectant substances
b. Δ-virus d. Resistant to high temperature;
c. Virus of hepatitis G nonresistant to low temperature and
d. Virus of hepatitis E disinfectant substances
e. HBs-antigen e. Resistant to high and low temperature;
710.The main way of hepatitis B transmission nonresistant to disinfectant substances
is: 716.The infection atrium for hepatitis B virus
a. Parenteral is:

75
a. Mucous membrane of respiratory tracts b. Adults outrank 30 years old
b. Mucous membrane of digestive tract c. Young people and adults (15-30 years
c. Mucous membrane sexual ways old)
d. Blood d. Elderly people (much than 60 years old)
e. Skin e. Age dependence does not exist
717.The patient with hepatitis B is dangerous 722.RNA-viruses causes the:
in: a. Smallpox
a. The end of incubation period b. Chickenpox
b. The end of incubation period and height c. Cytomegaly
of disease before occurrence of jaundice d. Hepatitis A
c. The end of incubation period and height e. Hepatitis B
of disease with an jaundice period 723.The viral hepatitis B DNA presence is
d. The height of disease with an jaundice revealed for laboratorial diagnostics
period during last years. What type of reaction is
e. The end of incubation period and during it?
disease a. Chain-polymerization test
718.The viral hepatitis B usually damages: b. Indirect-hemagglutination test
a. Children before 14 years old c. Complement-binding test
b. Adults outrank 30 years old d. Immunoenzyme test
c. Young people and adults (15-30 years e. Inhibition of hemagglutination test
old) 724.High-sensitivity methods are used for
d. Elderly people (much than 60 years old) checking the presence of virus hepatitis
e. Age dependence does not exist B antigens in donors' blood. Which
719.Mechanism of HIB action to hepatocytes test should be used?
and clinical manifestation of viral a. Solid-phase Immunoenzyme test
hepatitis B are: b. Immunoelectrophoresis
a. Has direct cytopatic action to hepatocytes, c. Indirect-hemagglutination test
serious forms of illness - acute or chronic d. Complement-binding test
with early formation of hepatic cirrhosis e. Neutralization test
b. Has direct cytopatic action to hepatocytes. 725.A patient with signs of hepatitis is
Often disease is over with full recovery hospitalized to the infectious
c. The mechanism of action is unknown, 50 compartment. She is a stomatologist.
% of all forms of a hepatitis pass in the What methods of laboratorial diagnostics
chronic form, and usually it leads to must be use for “Viral hepatitis”
cirrhosis and to a liver cancer diagnosis?
d. The mechanism of action is unknown; the a. Determination of HBs-antigen in blood
serious form of infection may be serum
especially at pregnancy b. Virological examination of excrements
e. Has meditational immunological action to c. Virological examination of urine
hepatocytes; usually chronically carriage d. Determination of functional activity of
is characterized as a persistent or hepar (bilirubin and cholesterol in blood)
chronically active hepatitis; and without e. Determination of enzymes activity
symptoms (aldolase, transamilase and other)
720.The most sensible method for HBsAg 726.The donor who for a long time didn't
revealing is: donate the blood was investigated with
a. Precipitation in gel test ELISA method.
b. Counter electrophoresis 1. Anti-HBs antibodies were revealed.
c. Passive hemagglutination test What does positive result of ELISA in
d. Inhibition of hemagglutination test this case mean?
e. Immunoenzyme test a. Acute hepatitis B
721.The viral hepatitis E usually damages: b. Have been ill a hepatitis B
a. Children before 14 years old c. Acute hepatitis C

76
d. Chronic hepatitis B c. Hepatitis C virus
e. Chronic hepatitis C d. Epstein-Barr virus
727.HBs-antigen has been revealed during e. Cytomegalovirus
enzyme immunoassay in serum. What 732.The patient is passing the clinical
infection can be diagnosed by this antigen examination. Diagnostics of hepatitis
detecting? viruses was done. Antibodies to HBs
a. Tuberculosis were revealed in a blood serum. This
b. Viral hepatitis A positive result means:
c. Viral hepatitis B a. Tolerate hepatitis B
d. AIDS b. Acute hepatitis B
e. Syphilis c. Acute hepatitis C
728.A patient is hospitalized with the d. Chronic hepatitis B
previous diagnosis “hepatitis B”. To e. Chronic hepatitis C
diagnose the disease a serological 733.A child with complains what serve for a
reaction based on interaction of antigen doctor as a base for diagnosis “viral
and antibody, chemically linked to hepatitis A” was hospitalize to infectious
peroxidase or alkaline phosphatase, was department In a month after returning
carried out. Name this reaction. from summer camp. What is the most
a. Immobilization possible infection mechanism of invasion
b. Radio-immune assay by viral hepatitis A?
c. Immuriofluorescence test a. Contact
d. Enzyme immunoassay b. Fecal-oral
e. Complement fixation test c. Airdust
729.The gene of hepatitis B virus was d. Transmissible
integrated to a genome of variolovaccine e. Parenteral
virus, what provide the HBsAg creation. 734.There are one of markers of hepatitis B
The recombined virus will be use like a was been detected at the blood analysis.
vaccine. This vaccine is refer to: Detecting what of markers of this
a. Synthetic hepatitis don’t allow t use donor’s blood
b. Combined for a hemotransfusion?
c. Associate a. HBc-antigen
d. Chemical b. HBs-antigen
e. Gene-engineering c. HAv
730.The results of laboratorial tests of patient d. HBe-antibodies
with jaundice are: HBsAg “-”, HBeAg “- e. HBe-antigen
”, anti-HBs(IgM) “-”, anti-HBs(IgG) “+”, 735.Patient M. was hospitalized to infectious
HCcAg “+”. What diagnosis is confirmed department with the next complains:
by these results? general malaise, rise in temperature of
a. Relapsis of hepatitis B body to 38ºC and jaundice. The doctor
b. Hepatitis C suspect a viral hepatitis B considering a
c. Chronic hepatitis B with low replicative hemotransfusion what had be done a few
activity months ago. What are main methods for
d. Hepatitis C, hepatitis B in anamnesis hepatitis B diagnostics?
e. Reinfection of hepatitis B a. Serological and genediagnostics
731.Testing of donor’s blood to HBs antigen b. Allocation of virus on a cells culture and
is not completely exclude a problem of identification by CPA
posttransfusional hepatitis. Which of c. Revealing of virions in blood by electron
these viruses exclude hepatitis B virus microscopy
most possibly can cause posttransfusional d. Isolation of virus in laboratorial animals
hepatitis? e. Isolation of virus in chicken embryo
a. Hepatitis E virus 736.At control of blood, the HBsAg was
b. Hepatitis A virus detected at one of the donors. Thus he

77
said that recently he has a course of phase of viral infection?
treatment with a parenteral entering of A. Ig M
infusional solutions. Estimate a blood of B. Ig A
this patient for transfusion. C. Ig E
a. Is possible to use a packed red cells only D. Ig D
b. Blood is suitably to transfuse E. Ig. G
c. Is possible to use a blood plasma only 742. The risk of congenital malformations is
d. Blood is no-suitably to transfuse 80% when infected pregnant women in the
e. Blood can be used after processing first trimester of pregnancy by the virus:
737.There is B and D viruses of hepatitis were A. Flu
detected at patient with a serious disease. B. Mumps
As we know, Δ-virus can’t reproduce in C. Rubella
hepatocytes independence. About what D. Hepatitis A
process the reproduction of hepatitis D E. Coxsackie
virus is possible in the presence of 743. Which of the following infections in the
hepatitis B? human cause adenoviruses?
a. Interference of viruses A. Myocarditis
b. Complementation of viruses B. Hepatitis
c. Genetic reactivation of defective viruses C. Faringconjunctivitis
d. Phenotypic mixing D. Poliomyelitis
e. Mutations in Δ-virus genome E. Hemorrhagic fever
738.HBsAg was detected at a patient’s blood 744. The patient, who received
which had jaundice 10 weeks ago. What immunosuppressive therapy for systemic
of pathology it is typical for? disease, signs of activation of CMV infection.
a. Viral hepatitis C Which method should be chosen to confirm
b. Viral hepatitis A the diagnosis?
c. Viral hepatitis B A. RAC, RN with paired sera
d. Viral hepatitis E B. The biological method
e. Viral hepatitis D C. Allergotest
739.There are the vaccine among of D. Investigation of cellular immunity
biological drugs, what created by gene- E. Investigation of nonspecific resistance
engineering way. What of infection it is 745. To the hospital a woman at the age of 23
prescribed for? was admitted who during pregnancy has not
a. Hepatitis A passed routine microbiological and
b. Poliomyelitis virological survey. Her baby has immediately
c. Measles after birth the skin and mucous small bubbles
d. Parotitis on the face of limited swollen spots. Later
e. Adenoviral infection developed the damage of central nervous
740.About what infection the detection of system and the baby was not saved. When
HBsAg in blood is display? sowing the contents of the bubbles in the
a. Herpetical infection blood agar growth of the pathogen have been
b. Smallpox identified, and during infection of tissue
c. Viral hepatitis A culture they observed CPE (simplasty,
d. Poliomyelitis intranuclear inclusions, degeneration of cells).
e. Viral hepatitis B Which organism is the most likely causative
agent of neonatal infections?
Laboratory diagnosis of diseases caused by A. Staphylococcus
DNA - viruses B. Gonococcus
741. For the diagnosis of generalized herpes C. Chlamydia
infection was studied blood serum to detect D. Herpes simplex virus
specific antibodies of specific class. E. Cytomegalovirus
Antibodies of which class indicate the acute 746. The family Herpesviridae subfamily

78
Alphaherpesvirinae belongs to: C. Toxoids
A. Varicella-zoster virus D. Chemical vaccine
B. The virus of smallpox E. Idiotypic vaccine
C. Coxsackie virus 754. CMV has tropism for cells:
D. Cytomegalovirus A. The spinal cord
E. Epstein-Barr virus B. Salivary glands
747. The main route of transmission of herpes C. Lymphocytes
simplex virus: D. Airway epithelial cells
A. Alimentary E. Epithelial cells of the gastrointestinal tract
B. Contact 755. In temperate countries Epstein-Barr virus
C. Parenteral usually causes:
D. Air and dust A. Hemorrhagic fever
E. Transplacental B. Infectious mononucleosis
748. The family Herpesviridae subfamily C. Lymphoma of Berkitt
Betaherpesvirinae belongs to: D. Lymphocytic choriomeningitis
A. Herpes simplex virus E. Nasopharyngeal carcinoma
B. Varicella zoster virus 756. In tropical countries the Epstein-Barr
C. Coxsackie virus virus usually causes:
D. Cytomegalovirus A. Hemorrhagic fever
E. Epstein-Barr virus B. Infectious mononucleosis
749. The family Herpesviridae subfamily C. Lymphoma of Berkitt
Gammaherpesvirinae belongs to: D. Lymphocytic choriomeningitis
A. Herpes simplex virus E. Nasopharyngeal carcinoma
B. Varicella zoster virus 757. In Southeast Asia Epstein-Barr virus
C. The virus of smallpox usually causes:
D. Cytomegalovirus A. Hemorrhagic fever
E. Epstein-Barr virus B. Infectious mononucleosis
750. As a rapid diagnosis of herpes infection C. Lymphoma of Berkitt
apply: D. Lymphocytic choriomeningitis
A. Enzyme Immunoassay E. Nasopharyngeal carcinoma
B. The reaction of neutralization 758. The penetration of rubella virus in
C. Immunofluorescence method macroorganism often occurs through:
D. Radioimmunoassay A. Lining of the airways
E. The reaction of hemagglutination B. Intestinal mucosa
751. The main route of transmission of C. Mucosa of genital organs
varicella: D. The salivary glands
A. Alimentary E. Damaged skin
B. Respiratory 759. Herpes simplex virus type 2 has a role in
C. Contact the development of:
D. The parenteral A. Berkitt's lymphoma
E. Transplacental B. Nasopharyngeal carcinoma
752. The center of persistent infection of C. Cervical Cancer
shingles stored in: D. Leukemia
A. Spinal cord E. Breast Cancer
B. Ganglia of spinal cord 760. Genomic DNA viruses cause:
C. Salivary glands A. Mumps
D. Nerve cells hippocampus B. Tick-borne encephalitis
E. Anterior roots of spinal cord C. Rubella
753. For the specific prophylaxis of varicella D. Hepatitis A
developed: E. Hepatitis B
A. Live vaccine 761. In the kindergarten several cases of
B. Killed vaccine children’s illness have been occurred. The

79
clinical picture was characterized by fever and painful when pressed. Appeared after
the appearance in the throat, mouth and skin pneumonia. What disease can be suspected?
rash vesicular. The preliminary diagnosis - A. Herpes simplex
chicken pox. Which of the following B. Shingles
materials should be sent to the virology C. Atopic dermatitis
laboratory for rapid diagnosis? D. Erysipelas
A. The contents of the vesicles E. Hives
B. Sputum 766. The patient with infectious of
C. Swabs from the hands mononucleosis identified the antibodies to
D. Urine antigens of Epstein-Barr virus. Which of the
E. Bile following ways are the most common
762. The contents of the mucous membrane transmission for this pathogen?
of the patient with vesicles smallpox were A. Alimentary
sent to the virology laboratory. Which of the B. Airborne
following changes were detected by smear C. Contact
microscopy? D. Sexual
A. Cells of Paschen E. Parenteral
B. Cells of Babesh - Negri 767. At 17 years old male diagnosed
C. Cells of Guarnieri gingivostomatit caused, as was demonstrated
D. Cells of Babesh- Ernst by the FTA, the herpes virus. What method
E. Syncytium can be established that this disease is the
763. The student of medical school, result of primary or recurrent infection?
hospitalized in the infectious disease clinic, A. Radioimunny analysis
on the 2nd day of the disease is suspected B. RAC with paired sera
infectious of mononucleosis. What is the C. Infection of chick embryos and tissue
result of laboratory tests can confirm the culture
diagnosis of a student on the day of D. Determination ІgG and IgM in serum
admission? E. The immune electron microscopy
A. Detection of Ig M-antibodies to herpes 768. The patient asked the dentist
simplex virus complaining of the appearance of blisters on
B. Detection of Ig M-antibodies to the the lips with the liquid on the verge of the
Epstein-Barr virus skin and mucous membranes. Which
C. Identification of 4-fold rise of antibodies to microorganisms can cause such pathology?
Epstein-Barr virus A. Mycobacteria
D. Isolation (separation) of the herpes virus B. Streptococci
E. Detection of antibodies to cytomegalovirus C. Herpes
764. Laboratory of Diagnostic Center D. Ortomyxovirus
received modern test systems to detect E. Staphylococci
TORSN infection, which makes it possible to 769. A patient has fever and the appearance
diagnose: 1) toxoplasmosis, 2) rubiinfection, of vesicles, which are located on the edge of
3) cytomegaloinfection, 4) herpes infection. the skin and mucous membranes. Through
Which of these diseases are caused by which research methods can confirm the
viruses? presence in the patient of herpes simplex
A. Rubiinfection, cytomegaloinfection, herpes virus?
infection A. Microscopic and allergic
B. Rubiinfection, herpes infection B. Virological and biological
C. Cytomegaloinfection, herpes infection C. Serological and biological
D. All of the above D. Microscopy and biological
E. Rubiinfection E. Virological and serological
765. The patient went to a doctor on the third 770. In the treatment of caries dentist drew
day of illness with complaints of a rash in the attention to the existence of clusters of
form of vesicles on the lips and nose wings, bubbles on the boundary of the skin and

80
mucous membrane of lips. Which micro- conditions
organisms can cause such pathology? 774. In determining the quality of water wells
A. Streptococci , was established that it’s microbial count -
B. Herpes 200, if titer - 100. Determine the index of
C. Mycobacteria water.
D. Ortomyxsovirus A 100
E. Staphylococci B. 10
771. The patient in the mucosa of the mouth, C. 5
nose, lips, appeared vesicular vesicles. Dentist D. 2
suspected vesicular stomatitis. What E .0.5
diagnostic method gives the opportunity to 775. In well water revealed the cholera
confirm the diagnosis? bacteriophage 5x102 in the number of phage
A. Infection with vesicular fluid of animals particles in 1 ml. Which method makes it
B. Statement of allergic tests possible to get these figures in?
C. Isolation of bacteria from the vesicular A. precipitation reaction with antiserum
fluid antiphage
D. Videlenie virus from vesicular fluid B. Electron Microscopy
E. Vesicular fluid microscopy C. The titration method of agar layers
772. Female at the age of 32 years old suffers D. density gradient ultracentrifugation
from recurrent viral infection. The disease E. Planting in culture medium
appears vesicular eruptions on the lips and 776. In the bacteriological laboratory
mouth, which are accompanied by fever and conducted a study of drinking water quality.
general intoxication. Which drug should be Its microbial count was around 100. Which
used for specific prophylaxis of recurrence of micro-organisms considered in this?
the disease? A. All bacteria that have grown in a nutrient
A. Acyclovir medium
B. Interferon B. Escherichia coli
C. Lysozyme C. Bacteria pathogenic to humans and animals
D. Herpetic inactivated vaccine D. Opportunistic microorganisms
E. Donor gamma-globulin E. Enteropathogenic bacteria and viruses
777. In determining the number of microbial
Sanitary-microbiological examination of air in the hospital it turned out that it is 1500
water, air, soil and food cells/m3. Which groups of organisms
773. When the sanitary-bacteriological considered in this?
examination of soil to determine A. Staphylococci and haemolytic streptococci
perfringenstiter inoculation was made B. Bacteria and viruses - pathogens of
dilutions of the suspension of the soil on respiratory infections
blood agar in Petri cups. Crops were grown in C. All bacteria that have grown in a nutrient
an incubator at a temperature of 37C. In a day medium
colonies of clostridia have been identified. D. Agents of hospital infections
What is the most likely cause a result that is E. All pathogenic and opportunistic bacteria
observed? 778. During the sanitary - bacteriological tests
A. For soil bacteria requires a lower of tap water have the following results: the
temperature total number of bacteria in 1.0 ml - 80 cells,
B. The absence of necessary growth factors in the index - 3. How to interpret the result of
a medium the study?
C. Clostridium form colonies only after 3 A. The water quality questionable
days B. Water suitable for drinking
D. The growth of clostridia was suppressed C. Water quality is very questionable
by microbes-antagonists, which are also D. The water is contaminated
present in the soil E. The water is very polluted
E. Have not been provided with anaerobic 779. During the sanitary-bacteriological

81
examination of water using membrane filters E. Reaction imunofluoresence
revealed two red colonies on a membrane 784. In the tray of a private entrepreneur sold
filter (Wednesday Endo), who conceded in the minced meat, according to the seller of
500 ml of water under study. Calculate the beef. However, the sanitary inspector
index and the titer of investigated water suspected that stuffing is made of dog meat.
A. 4 and 250 Through which reaction can be identified this
B. 2 and 500 food?
C. 250, and 4 A. Flocculation
D. 500 and 2 B. Binding of complement
E. 250 and 2 C. Agglutination
780. To assess the suitability of drinking D. Ringprecipitation
water conducted bacteriological research. E. ELISA
Which indicator shows the number of 785. In the infectious ward three students
coliforms, which is contained in 1 liter of were brought who lived in the same room,
drinking water? with a diagnosis of "acute gastroenteritis."
A. Perfringens-titer The disease occurred after ingestion of
B. Coli-titer cooked meat, which remained at room
C. Coli phage titer temperature during the day. What is the most
D. Coli-index probable result of seeding of pathological
E. Microbial number material (gastric washings and feces) on the
781. When checking the condition of air in culture media?
the operating room before surgery A. The blue colonies on agar, alkaline
sedimentation method revealed five small, B. Crimson colonies on Endo medium
round colonies, which are clearly visible C. Uncoated colonies on Endo medium
around the zone of hemolysis. What kind of D. Yellow colonies on VSA
impact has been made the crops on? E. Large slimy colonies on MPA
A. Levine 786. In the laboratory of sanitary-
B. IPA epidemiological station we study the quality
C. Endo of drinking water. What method can be
D. VSA determined if the index and the number of
E. MPA blood water-titer?
782. The study of air in the operating room A. Filtration
revealed that it does not satisfy the sanitary B. Sedimentation
and bacteriological standards. Isolation of C. Serial dilutions
microorganisms which allowed making such a D. The method of titration
conclusion? E. Microscopic
A. Sartsiny 200 / m 3 787. In carrying out sanitary-bacteriological
B. Actinomycetes 100 / m 3 examination of air in a dental office to
C. E. coli 10 / m 3 determine the number of sanitary-indicative
D. Hemolytic streptococcus 5 / m 3 microorganisms (Staphylococcus aureus and
E. Staphylococcus aureus 3 / m 3 hemolytic streptococci) in 1 m3 of air. What
783. In the laboratory of food hygiene of is the culture medium used for this?
regional SES delivered withdrawn from the A. Blood and yolk-salt agar
seller beef mince in the market. The buyer has B. MPA and MPB
doubts about the quality of meat. He believes C. Endo and Ploskirev Medium
that the stuffing is made of dog meat. What is D. The yolk and sugar broth
an immunologic response can check the E. Kitt Tarotstsi and Wilson-Blair Medium
quality of the delivered product?
A. The reaction of precipitation Normal microflora of the human body
B. The reaction of Coombs 788. The patient, who has long taken
C. agglutination antimicrobials, conducted bacteriological
D. Reaction of opsonization examination of vaginal microflora and

82
determination of pH. The absence of C. II degree
lactobacilli and alkaline conditions. What D. III degree
should be assigned the patient to restore E. The resulting data are insufficient for
normal microflora of the vagina? assessing disbiosis
A. Lactic acid bacteria 793. The patient at the age of 68, who was a
B. Suppositories with antibiotics three-week course of antibiotics, bowel
C. A solution of potassium permanganate dysfunction is marked, abdominal pain. The
D. Sulfanilamide drugs study on fecal bacteria overgrowth yielded the
E. Suppositories with antiseptic following results:
789. Cancer patients removed most of the bifidobacteria-5x 105, lactobacilli-102, the
large intestine. Which drugs should be total number of E. coli - 1010, E. coli with
assigned to the patient to replace the function reduced enzymatic properties - 40%;
of colon microflora? hemolytic E. coli - 10%
A. Vitamins opportunistic enterobacteria-105,
B. Antistaphylococcus plasma hemolytic aureus - 102,
C. Polyvalent bacteriophage fungi of the genus Candida - 103. To what
D. Antibiotics extent these data indicate dysbiosis?
E. Sulfonamide A.Norma
790. The patient after prolonged use of B. I degree
antibiotics developed intestinal dysbiosis. C. II degree
Which drugs should be appointed to restore D. III degree
the normal microflora? E. The resulting data are insufficient for
A. Sulfonamide assessing dysbiosis
B. Eubiotics 794. Child of 5 months, which is on artificial
C. Interferon feeding, recovering from kolienterit. Before
D. Antifungal discharge from hospital a study was
E. Cephalosporin conducted quantitative and qualitative
791. When dysbiosis, which is accompanied composition of intestinal microflora. Obtained
by the development of putrefactive flora and the following results: bifidobacteria - 5x 107;
increase the pH of faeces, it is necessary to laktobatsily - 108;
assign biological agents that oxidize medium total coliform-109, E. coli with reduced
and exhibit an antagonistic effect. What enzymatic properties - 10%;
organisms are used for this? hemolytic E. coli - 5%
A. Bifidumbacteria opportunistic enterobacteria - 105; aureus -
B. Klebsiella 104; fungi of the genus Candida - 103. What
C. Azotobacteria tools are most appropriate for corrections of
D. Enterobacteria microbiotsenosis intestine in a child?
E. Sarcina A. Antibiotics that selectively inhibit the
792. There is dysfunction of the intestine at growth of staphylococci and yeasts
10 years old child for three months. B. Inclusion of milk products in the diet
Investigation of fecal material of disbiosis C. Bifidumbacterin
gave the following results: bifidobacteria - D. Colibacterin
5h108; lactobacilli - 109; E. Lactobacterin
the total number of E. coli -107; Escherichia 795. Pregnant woman was diagnosed with
coli with reduced enzymatic properties - 8%; bacterial dysbiosis of vagina. Which drug
opportunistic enterobacteria - 5x10; should be chosen in this case?
staphylococci 104; hemolytic staphylococci A. Bacteriophage
were not identified, fungi of the genus B. Antibiotic
Candida 102. To what extent these data C. Interferon
indicate disbiosis? D. Eubiotics
A.Norma E. Polyvitamins
B. I degree 796. In a patient of 56 years old after

83
prolonged treatment with antibiotics A. You must repeat the study in a few days
developed dysbiosis: weight loss, frequent B. A numerical method for the diagnosis and
stools, a significant amount of fecal hemolytic the correct diagnosis
E. coli, Proteus, Staphylococcus. Which of C. There should also be staging the
the following actions would eliminate the sedimentary response by Cann
imbalance autochthonous microflora: D. Must be microscopic study
A. Replace the antibiotics and to phage E. The need for formulation of an allergic test
therapy 800. At the dental examination of the child of
B. Cancel the antibiotics and sulfa drugs to 3.5 years old of the mucous membrane of
appoint cheeks and palate doctor said raids of white
C. Cancel antibiotics and assign eubiotics and yellow colors that merge to form a cheesy
D. Provide chelators and immunomodulators. film. When removing the film - the mucous
E. Provide drugs and adjuvants. membrane is hyperemic, with a smooth
surface. What picture will most likely be
The microflora of the mouth. observed by microscopy of smears from these
797. 63 years old man turned to the dentist films?
complaining of a complication during A. Gr + cocci, arranged in a cluster of grapes
chewing. The doctor suspected the B. Gr + cells elongate budding
development of tetanus infection. As it was C. Gr + cocci arranged in chains
found a week ago, the patient is deeply D. Gr + rods with swellings at the ends
pierced his leg with a rusty nail. What E. Gr - small coccobacilli
material should be sent to study in the 801. Bacteriological examination of 10 years
bacteriological laboratory to confirm the old boy’s mouth revealed a large number of
clinical diagnosis? β-hemolytic streptococci. What is the most
A serum of patients credible explanation for this phenomenon?
B. Blood patient A. The diet of the boy too much carbohydrate
С. Smear from the surface of the wound B. The child is sick caries
D. Pieces of necrotic tissue C. The child is sick periodontal
E. Flush with subject matter that was the D. The boy observes bad oral hygiene
cause of injury E. β-hemolytic streptococci are the
798. 32 years old man in the lower jaw representatives of normal microflora
formed dense, flushed swelling, pain is 802. From the oral cavity of clinically healthy
virtually nonexistent. After 2 weeks, pus man at the age of 25 years old has identified
appeared on the surface of the hearth, formed the culture of Gr + cocci, which are somewhat
a fistula, a chronic infectious process was elongated, arranged in pairs or short chains
slow and extends to the underlying tissue. form a capsule on blood agar yield of alpha-
Microscopy of pus was found beads that hemolysis. Is this man a carrier of some kind
represent a collection of threads, as well as of pathogen of microorganism?
individual rod-shaped and сoco form. What is A Streptococcus salivarium
the most likely diagnosis? B. Streptococcus pyogenes
A. Actinomycosis C. Streptococcus pneumonia
B. Deep mycosis D. Streptococcus feacalis
C. Candidosis E. Peptostreptococcus
D. Nocardiosis 803. At the patient's buccal mucosa revealed
E. Mixed bacterial infection net shaped ulcer with hard bottom and
799. The man at the age of 21 turned to the smooth edges. Which organisms are most
doctor with a dense rounded ulcer on the likely to be observed by microscopy of fluid
mucous surface of the cheek. The doctor from the surface of an ulcer?
suspected syphilis and gave direction to the A. Thin, spiral, motile
study of serum. Wassermann was negative. B. Acid FAST Sticks
The doctor put the negative diagnosis. How to C. Gr + cocci arranged in chains
evaluate the tactics of a doctor? D. Large spore bacillus

84
E. Bipolar colored ovoid C. Virological
804. Dental instrument has been sterilized by D. Serological
boiling for 5 minutes. Which organisms, have E. Biological
got it out of the mouth or the patient's blood, 809. Microscopy of dental plaque was
are likely to preserve? detected moving spiral-like bacteria. Which
A. Diphtheroids method allowed us to establish the mobility of
B. AIDS virus these organisms?
C. Streptococci A. Fluorescent microscopy
D. Fuzobacteries B. Darkfield Microscopy
E. Hepatitis B virus C. Staining with Romanovsky-Giemsa
805. Dentist analyzed the qualitative D. Staining of Gins-Burri
composition of microflora of the mouth’s 70- E. Electron microscopy
year-old man, and concluded its worst 810. Choose among the normal microrflora
condition. Identify what microorganisms the oral obligate anaerobes.
allowed to make such a conclusion? A. Streptococci, staphylococci
A Streptococcus mutans B. Bacteroids, fuzobacteries
B. Treponema denticola C. Corynebacterium, veylonelly
C. Escherichia colі D. Mycoplasmas, Borel
D.Mycoplasma orale E. Leptospira, Candida
E. Candida albicans 811. Patient D. appealed to the dentist with
806. In the pus taken from a depth of necrotic complaints on halitosis (unpleasant odor from
ulcer of the patient necrotizing ulcerative the mouth). What bacteria dominate the
stomatitis, identified fuzobacteries. What microflora of the oral cavity in this case?
other organisms are involved in the A. Genus Proteus
development of this disease and to stand out B. Type of Bacteroides
in association with fuzobacteries? C. Type of Corynebacterium
A Streptococci D. Type of Esherichia
B. Mycoplasmas E. The genus Clostridium
C. Yeast-like fungi 812. Bacteriological examination of the oral
D. Spirochetes cavity of dental patients revealed a number of
E. Diphtheroids microorganisms. Which of them belong to
807. At 17 years old male diagnosed eukaryotes?
gingivostomatit caused, as was demonstrated A. Mycoplasma orale
by the FTA, the herpes virus. Through a B. Streptpcoccus mutans
method to establish whether the disease is C. Candida albicans
caused by primary or recurrent infection? D. Treponema buccalis
A. Radioimmunoassay E. Bacteroides melaninogenicus
B. RAC with paired sera 813. In the mouth of 55 years old man was
C. Contamination of chicken embryos and found staphylococcus, which forms enzymes
tissue culture hyaluronidase and collagenase. What are the
D. Determination of IgM and IgG in serum most possible damage it may cause the
E. The immune electron microscopy organism?
808. From the dental patient was taken the A. The destruction of the connective tissue
pus from the abscess to investigate, which is B. demineralization of tooth enamel
located on the lower jaw. Pathological C. The formation of tartar
material planted on blood agar for detection D. Change of periodontal tissue trophism
of cocci and on Kitt Tarotstsi medium for E. Necrosis of the epithelium
detection of anaerobes. Which method of 814. In a patient with recurrent inflammatory
microbiological diagnosis was used in this lesions in the oral cavity bacteriological
case? examination revealed a large number of
A. Bacteriological gramnegative bacteria. What conclusion
B. Smear should be drawn regarding the patient's

85
condition? are?
A. Dysbiosis A. Optional aerobes
B. T-cell immunodeficiency B. Aerobes
C. Candidosis C. Anaerobes
D. Recurrent herpes D. Gram-negative bacilli
E. An allergic condition E. Gram-positive cocci
815. The patient has a dental clinic in 819. One of the factors that contribute the
Flushing oral mucosa sowed E. coli. Which development of dental caries in humans is the
members of the microflora of the mouth, formation of a significant number of
these bacteria belong to? microorganisms of lactic acid. Choose among
A. Resident the listed microorganisms that ferment
B. Transient carbohydrates to form a large amount of lactic
C. Causal agent of caries acid.
D. The causative agent of stomatitis A. Staphylococcus, Corynebacterium
E. Pathogen of pulpitis B. Veiloneli, Streptococcus
816. Patient B., aged 59, appealed to the C. Fuzobacteries, lactobacilli
dentist about halitosis. After detailed D. Treponema, Neisser
examination the doctor found that the cause of E. Actinomycetes, fungi
halitosis (halitosis) was associated with a 820. Patient appealed to the dentist with
violation of the microflora of the mouth. initial stage of pulpitis. What organisms are at
Which of these microorganisms may play a this stage of the disease?
central role in the development of halitosis? A. Oral streptococci, lactobacilli
A. Anaerobic bacteria (prevotely, bacteroids, B. Beta-hemolytic streptococci, bacteroides
peptostreptococci, veiloneli) C. Proteus and Clostridium
B. Bifidobacteria D. E. coli
C. The simplest (Entamoeba gingivalis, E. Anaerobic streptococci
Trichomonas elongate) 821. Dentist conducts a conversation among
D. Facultative anaerobic cocci (staphylococci, school children in caries prophylaxis. What
streptococci) are the substances that are contained in foods
contribute the growth and multiplication of
Microbiological and immunological aspects streptococci, and are of high importance in the
of the etiology and pathogenesis of dental occurrence of dental caries?
caries. A. Carbohydrates
817. Caries is the result of a complex of B. Proteins
harmful factors in the oral cavity. The initial C. Fats
phase of caries is a direct consequence: D. Vitamins
A. The provision of the teeth of a film of E. Micro-and macro-
saliva glycoprotein 822. Patient appealed to the dentist
B. Synthesis of Streptococcus mutans glucan complained of pain in the fifth lower right
C. microbial insemination plaque anaerobes tooth, which appears in the use of food as
D. The formation of dental plaque bacteria, well as discomfort when applied to a tooth
organic acids cold or hot. While examination, the doctor
E. IgA class of antibodies against components found a tooth cavity. In probing the bottom of
of the microflora in the oral cavity the cavity particularly painful points were
818. After opening the maxillary abscess, found, indicating a deep cavity. Specify a
which is accompanied by swelling of the group of cariogenic microorganisms.
tissues of the face, the temperature up to 39C, A. S.salivarius, diphtheroids, peptococci
pus was taken and sent to the bacteriological B. S.mutans, lactobacili, actinomycetes
laboratory. The study found that pathological C. S.mitis, Treponema, Neisser
material containing microorganisms of the D. Stomatococci, leptotrihii, Staphylococcus
genus Bacteroides. What are the types of E. Veilonella, Treponema, Borrelia
microorganisms of the usage of oxygen they 823. Lactobacilli are cariogenic

86
microorganisms because they are: S.mutans?
A. Able to survive without nutrients A. It prevents adhesion
B. Stand out from the cavity B. Cause lysis
C. The producers of acid C. Phagocytose
D. In plaque D. Neutralize exotoxin
E. Cytohromoxidaznegative E. Loosen the saccharolytic activity
824. 28 years old patient appealed to the 829. Inoculation of pathological material from
dentist complaining of a throbbing toothache, the patient pulpitis was made on Kitt Tarotstsi
which gives the temple. Found in probing the medium. Which microorganisms are
cavity felt pain all over her bottom, which is committed?
characteristic of acute pulpitis. What are the A. Hemolytic
most common organisms involved in the B. Acid
development of pulpitis? C. Acidophilic
A. Staphylococcus aureus. D. Anaerobic
B. Streptococcus salivarius. E. Aerobic
C. Actinomyces viscosus. 830. While examination the patient's teeth
D. Leptotrichia buccalis dentist found many "white spots" - areas of
E. Prevotella melaninogenica. demineralization of enamel. Indicate which
825. In a patient with multiple caries was microorganizm take part in the development
carried out immunological test that will assess of this process?
the humoral local immunity in the oral cavity. A. Streptococcus mutans
Specify the ratio of IgA, IgG, IgM B. Streptococcus salivarius
characteristic of healthy human saliva? C. Streptococcus pyogenes
A. 4:2:1 D. Veilonella parvula
B. 5:2:2 E. Staphylococcus epіdermidis
C. 1:4:2 831. The first stage of dental caries is the
D. 20:3:1 formation of zones of demineralization of
E. 1:1:2 enamel. What are the microbial factors
826. Patient appealed to the dentist with involved in the development of this process?
multiple cavities. What properties are the A. Coagulase of golden staphylococcus
most common bacteria that cause tooth B. Collagenase anaerobic bacteria
decay? C. Streptococcus mutans acid formation
A Lysogenic D. Letsitinaz of staphylococci
B. Resistance to antibiotics E. Streptococcal exotoxins
C. Lipolytic 832. Dental surgeon had a puncture phlegmon
D. Acidogennye (saccharolytic) in a patient with fever
E. Fagolizabelnye the face. What material should be sent to
827. Dentist conducts preventive conversation baclabolatory?
among students on compliance with the rules A swab from the oropharynx
of hygienic dental care. He told the children B. Blood, purulent exudates
that in violation of hygienic care of teeth C. Cerebrospinal fluid
formed on the surface of dental plaque, which D. Duration of the tonsils
is dangerous to the teeth. It is formed in: E. Blood, urine
A. Three to six months 833. In the occurrence of dental caries is very
B. A few hours important plaque formation. What is
C. One month predominant microflora in the early formation
D. Thirty minutes of dental plaque.
E. A few days A. Obligate anaerobes
828. During a routine inspection of B. Streptococci, veilonella
schoolchildren dentist revealed that the C. Fuzobacteries
majority of carious no damage to teeth. How D. Bacteroids, Candida
secretory IgA to protect against cariogenic E. Leptotrihii

87
834. The patient with pulpitis has edema of agent and find an effective drug to treat a
the soft tissues of the upper jaw. Which factor patient?
is the lead in the occurrence of edema? A. Identification of antigens of the pathogen
A. Tissue gipoosmiya B. Detection of specific antibodies
B. Tissue gipoosmiya C. Microscopic examination of punctate
C. Blood gipoosmiya D. Isolation of pure cultures
D. Blood gipoosmiya E. Complex serology
E. Increased permeability of blood vessels. 840. In the department of maxillofacial
835. For anticaries immunization offered the surgery patient was diagnosed with
vaccine from a suspension of heat-killed suppurative complication. In the
S.mutans. What side effects are possible when bacteriological examination of material from
using this drug? a wound culture was isolated, which formed
A. Hepatic Impairment the pigment of blue-green color. Which of the
B. Cardiac following microorganisms may be the
C. Mucous membranes causative agent of infection in this case?
D. The defeat of the periodontal tissues A. Proteus vulgaris
E. Lung B. Pseudomonas aeruginosa
836. Caries - one of the most common dental C. Bacillus subtilis
illness .Through the development of methods D. Klebsiella pneumonia
of caries prophylaxis in recent years their E. Staphylococcus erіdermidis arius
hopes on the use of anticaries vaccines of S. 841. The patient asked the dentist about the
mutans. The primary defense mechanism of gingival and alveolar gnoeotdelenie caused by
such vaccination is associated with: Gram-positive bacteria. Microorganisms are
A. Education secretory Ig A, which prevents the kind most often cause a purulent-
the adhesion of S. mutans inflammatory processes of maxillofacial area?
B. Formation of delayed-type hypersensitivity A Staphylococcus
C. Activation of saliva lysozyme B. Streptococcus
D. The production of antistreptolysin-O S. Veilonella
E. Stimulation of Ig E production D. Lactobacillus
837. Local immunity in the oral cavity is E. Bacteroides
largely due to the content of antibodies in 842. Of the cavity and dental plaque
saliva. Antibodies are a class of saliva in streptococci isolated several species thought
higher concentration than in blood, whereas to be the main factors of this disease. To what
the content of the saliva and blood about the family in the systematics of bacteria are called
same? Bergey bacteria?
A. Ig G A. Veilonellaceae
B. Ig A B. Streptococcaceae
S. ІgM C. Mycoplasmataceae
D. D Ig D. Bacteroidaceae
E. Ig E E. Micrococcaceae
838. Bacteriological examination of patients 843. From the mouth of 5 years old child, the
with dental caries has been allocated different sick tooth decay, isolated Gr + organisms
microorganisms. Which of the following correct spherical form. In smears, they are in
microorganisms play a leading role in the chains, on blood agar give small translucent
occurrence of caries in these patients? colonies without hemolysis. Which type is
A Streptococcus mutans most likely owned by the microorganisms?
B. Staphylococcus aureus A. Streptpcoccus mutans
C. Candida albicans B. Staphilococcus saprophyticus
D. S. salivarius C. Mycoplasma orale
839. As a result of delayed treatment of D. Candida albicans
pulpitis patient rozvilsya osteomyelitis of the E. Bacteroides melaninogenicus
mandible. What research will identify the 844. 20 years young man caries led to

88
complications: inflammation of the dental A. Lactobacilli and diphtheroids
pulp. Dentist believes that it was caused by B. Actinomycetes and veilonelli
streptococci pulpitis. What kind of fertile C. Escherichia coli and Streptococcus
ground to sow the pathological material (pulp D. Staphylococcus and Streptococcus
content) in order to test this hypothesis? E. Pseudomonas aeruginosa and Proteus
A. IPA 849. In the department of maxillofacial
B. Blood agar surgery patient was admitted with a solid
C. Endo Wednesday phlegmon and multiple fistulas, of which
D. Folded serum stands out the pus with a foul odor. Which
E. The alkaline agar method should be applied to confirm the
845. In a patient with caries tooth surfaces diagnosis of actinomycosis?
have been identified caries white spots in the A. Smear
formation of which microorganisms are B. Biological
involved. Dentist after treatment caries of C. Bacteriological
teeth suggested the patient to take prophylaxis D. Mycological
measures for the prevention of tooth decay. E. Serological
What factors should be sent these preventive 850. After tooth extraction in patients with
measures on? progressive suppurative inflammation.
A. The carbohydrate nutrition and S. mutans Microscopic examination of pus revealed
B. At the protein diet and S.aureus gram-positive nonmotile, thick, surrounded
C. The carbohydrate nutrition and S.aureus by a capsule bacillus. What method is
D. At the protein diet and S.mutans necessary to confirm the preliminary
E. The carbohydrate nutrition and S.sanguis diagnosis - gas anaerobic infection?
846. Based on the examination of the patient A. Allergic
K. doctor of maxillofacial department district B. Bacteriological
hospital has put a preliminary diagnosis of C. Biological
"acute odontogenic osteomyelitis." D. ELISA
Microscopying the drug, made from E. Serological
pathological material, laboratory showed 851. The patient appealed to the dentist with
Gram-positive bacteria spherical shape, which an acute pulpitis. Which microorganisms play
is arranged in a random clusters ("grapes"). a major role in the inflammation of the pulp?
Which organism is the causative agent of A. Streptococci of group D
odontogenic disease? B. Vibrio
A. Staphylococcus C. Spirillum
B. Actinomycetes D. Sarcina
C. E. coli E. Micrococci
D. Streptococcus 852. As a result of delayed treatment of
E. Against pulpitis osteomyelitis of the mandible was
847. In the maxillofacial unit of the regional developed in patient. With some research you
hospital surgery on the submandibular can set the type of agent and find an effective
phlegmon site was performed . With the help drug to treat a patient?
of what method of microbiological studies A. Isolation of pure cultures
can establish the etiology of this disease in B. Appearing of specific antibodies
order to conduct an effective antibiotic in the C. Microscopic examination punctate
out functional period? D. Appearing of pathogen antigens
A. Allergic E. Integrated serology
B. Bacteriological 853. The patient appealed to the dentist with
C. Biological symptoms of inflammation in the oral cavity.
D. Tuberculosis microscopy In smears that are made from the teeth and
E. Serological gums are revealed in the simplest form of
848. Which microorganisms often act as non-constant, which varies due to the
agents of abscesses in maxillofacial area? formation pseudolegs. Sizes - 6 to 30 microns.

89
Indicate the type of protozoa. inflammation?
A. Oral amoeba A. Herpes
B. Giardia B. Mycobacterium leprae
C. Intestinal Amoeba C. Treponema pallidum
D. Intestinal trichomonads D. Bacillus anthracis
E. Dysenteric amoeba E. Actіnomyces isrrailii
858. The patient appealed to the dentist with
Microbiological and immunological aspects complaints of pain, redness, swelling gums.
of the etiology and pathogenesis of Suspected herpetic gingivostomatit. What
periodontal lesions virus could cause disease?
854. The patient appealed to the dentist with A. Herpes simplex virus type 1
necrotizing ulcerative stomatitis. What B. Herpes simplex virus type 2
organisms are naturally in the inflammation? C. Zoster virus
A beta-hemolytic streptococci, bacteroides D. Cytomegalovirus
B. Fuzobacteries Vincent and Treponema E. Epstein-Barr
C. Proteus and Clostridium 859. On examination, the patient's oral cavity
D. E. coli revealed a necrotizing gingivitis.
E. Anaerobic streptococci Bacteriological examination revealed
855. The patient ,36 years old , appealed to Fusobacterium nucleatum, F.necrorum. What
the dentist complaining of pain in the lower are the serious complications of this infection
six tooth when chewing, bleeding from a are at?
tooth while eating and feeling bad smell. A. Severe necrotic processes in the
What caused halitosis? destruction of tissues of the face (noma)
A. Education microorganism’s methyl and B. Gingivostomatity
hydrogen sulfide. C. Periodontal Disease
B. Synthesis of microbial hyaluronidase D. Chronic lip fissures
C. Products of microorganisms of lactic acid E. Bridou
D. The products of the destruction of tissues 860. In patients with chronic periodontitis
by microorganisms scheduled for a study to assess the local
E. The presence of endotoxin microorganisms immunity of the mouth. Where exactly are
856. After examining the patient the dentist localized plasma cells that synthesize
decided that the bacteriological examination immunoglobulins, oral fluid?
of pathological material taken from a patient A. In their own record of the mucosa and
is not feasible. Under what conditions could salivary glands
this be? B. In the lymph nodes
A. To determine the sensitivity of C. In the spleen
microorganisms to antibiotics abnormality D. In the thymus
B. Under the conditions of specific diseases E. In bone marrow
(eg, tuberculosis) 861. It is known that periodontitis is the result
C. Subject to the allergic nature of the of the different species of bacteria and their
pathological process metabolic products. What type of bacteria
D. In the case of purulent processes produces collagenase, which breaks down
E. Under the conditions necessary to clarify collagen - the main protein component of the
the nature of the infectious lesions of the periodontium?
mucous membrane A. Prevotella melaninogenica
857. The patient D. appealed to the dentist B. Treponema vinsenti
with complaints of pain in the gums, swelling C. Veilonella parvula
of pus, redness, formation of granulomas. Pus D. Leptospira dentium
sent in baclabolatory. In blood smears E. Treponema dentium
revealed gram-positive organisms in the form 862. The patient as a result of activation of
of filaments at the ends of which were their own microorganisms that are part of the
controversial. What organisms cause microflora of the oral mucosa, had purulent

90
inflammation of periodontal tissues. Which A. After 3-4 hours after ingestion of food
form of infection disease belongs to? B. After 8-10 hours after ingestion of a meal
A. Exogenous infection C. Before taking food
B. Autoinfection D. Immediately after taking the meal
C. Reinfection E. After the starvation
D. Superinfection 867. During periodontitis the surface structure
E. Relapse of tissues changes. What processes must be
863. In the doctor's office, periodontist for the studied to assess the body's response to the
appointment of strengthening therapy, the changed structure of the periodontium?
patient studied factors of nonspecific A. Autoimmune
resistance of saliva and secretions of the B. Immune
mucous membrane. Which of the following C. Purulent and inflammatory
factors of nonspecific resistance in the first D. Antibody
place will be studied in the material? E. Leukocytosis
A. Complement 868. From a patient with apical periodontitis
B. Secretory IgA material is taken to isolate pure cultures of the
C. Properdin pathogen. Where can patmaterial be sowed
D. Interferon for isolated colonies?
E. Lysozyme A. On the surface of the agar in a Petri cup
864. From the material studied patient B. On the surface slant
periodontium in association with gram- C. The liquid culture medium
negative spirochetes were detected D. Injection into a column of dense medium
microorganisms rod with pointed ends. The E. In the elective medium fluid consistency
bacteria belong to the autochthonous 869. From a patient with a diagnosis of
microflora of the oral cavity. Can form of periodontal disease in the bacteriological
peptone and glucose lactic acid. Which family examination of clinical material were isolated
owns the selected bacteria? gramnegative polymorphic bacilli, which
A. Lactobacillus grow only under anaerobic conditions and
B. Treponema whimsical to the culture medium. Which of
C. Actinomices the following microorganisms may they be?
D. Fusobacterium A. Bacteroids
E. Bacteroides B. E. coli
865. During routine inspection the dentist C. Clostridium
revealed periodontal disease in the pupil. It is D. Mycobacteria
established that periodontal disease is a E. Rickettsia
complication of previously transferred to viral 870. At the time of discharge from the gums
infection, in which damaged small vessels. microscopy patient who is suffering from
The disease is caused by a virus of the family periodontal disease, identified the simplest
Paramyxoviridae genus Morbillivirus. Virus form of pear-shaped, 6-13 mm in length. In
complex, has hemagglutinin, neuraminidase one of the parasite nucleus, on the front end
does not contain very sensitive to sunlight. has 4 flagella, undulating membrane is. What
Outside the body is preserved up to 30 are protozoa found in the patient?
minutes. Which virus caused the disease A. Amoeba
transferred? B. Guardia
A. Mumps virus C. Leishmania
B. The influenza virus D. Balantidium
C. Measles virus E. Trichomonas
D. The paragripp virus
E. Respiratory syncytial virus Microbiological and immunological aspects
866. For microbiological diagnosis of of the etiology and pathogenesis of
odontogenic disease dentist spends fence infectious lesions of the oral mucosa
material. In what period it is expected to do? 871. The young man was found severe

91
gingivostomatita and microbiological study of D. A virus Coxsackie
oral fluid was isolated fusiform bacteria and E. Coxsackievirus B
spirochetes oral. What gingivostomatit arose 876. Patient K., age 67, after a few months
in this patient? after the establishment of a fixed prosthesis
A. Gingivostomatit caused by the anaerobic teeth he asked the dentist about the swelling,
microflora redness and irritation of the mucous
B. Gingivostomatit Vincent membranes of the mouth, appears at the
C. Bacterial gingivostomatit corners of his mouth, " bridous ". Which of
D. Primary gingivostomatit the following organism is the cause of these
E. Viral gingivostomatit complications after prosthesis?
872. Dentist found in the oral mucosa of sick A. Veilonella
girl, 7 years old, ulcer 1.5 cm in diameter with B. Staphylococcus
irregular edges and saped gray bottom. After C. Streptococcus
the paint scrapings from the ulcer by Ziehl- D. Candida
Neelsen in the drug revealed thin rods of E. Treponema
ruby-red color, arranged in random clusters 877. In the dental clinic brought a child with
and alone. For a pathogen is characterized by fever, complaints of painful sores on the
such symptoms? mucous gums that look like blisters with
A. Actinomycosis serous contents. In the smear from vesicles
B. Syphilis stained by Romanovskiy-Giemsa, revealed
C. TB giant multinucleated cells with intracellular
D. Diphtheria inclusions. Which virus caused the disease?
E. Candidiasis A. Alpha-herpes
873. Of pathological material from a sick B. Beta-herpes
child (scraping of the sores on the mucous C. Gamma-herpes
membrane of the mouth) medication prepared D. HIV
and stained by Ziehl-Neelsen. Identification E. KSIP viruses
of the causative agent provided? 878. Patient appealed to the dentist with fever
A. FMD and characteristic small vesicles on the
B. Actinomycosis mucous membrane of cheeks, palate and
C. Syphilis tongue. The doctor suspected herpetic
D. Herpes stomatitis. What additional research is
E. TB necessary to confirm the diagnosis?
874. In the virology laboratory in the study of A. Infection of chick embryos in
fluid from the vesicles on the mucous horionalontois, the introduction of the
membrane of cheeks, patient C was isolated pathological material in the brain of white
vesicular stomatitis virus. Which the family mice
belongs the virus to? B. Inoculation of 199 with the addition of
A. Rinoviridae bovine serum
D. Poxviridae C. Sowing on Wednesday Rappaport
C. Rhabdoviridae D. Statement of the precipitation reaction
D. Reoviridae E. Sowing on Wednesday Needle
E. Togaviridae 879. The dentist diagnosed with aphthous
875. In a study of patients with congenital stomatitis, herpetic in two month child. What
immunodeficiency, S. dentist found on the way was a child infected?
oral mucosa vesicles filled with turbid yellow A. The contact
fluid and diagnosed a viral stomatitis. What is B. Vertical
the most common virus is the causative agent C. The air-dust
of this disease? D. Through fecal-oral
A. Herpes simplex virus type 1 E. At the time of delivery
B. Herpes simplex virus type 2 880. The patient with the ulcer, which is
S. Herpes simplex virus type 3 located on the oral mucosa, staining by

92
Romanovsky-Giemsa revealed thin spiral-like E. Actinomyces
organisms pale pink with 12-14 coils and 885. In studying the microflora of the oral
pointed ends. What pathogen is characterized cavity in a patient with stomatitis E.soli found
by such symptoms? in large quantities. Which drug is advisable to
A. Syphilis apply for adjustment status dysbiotic mouth?
B. Causative agent of leptospirosis A. Bacterial immunomodulators
C. Causative agent of typhoid rotary B. Bacteriophages
D. Agent of campylobacteriosis C. Antibiotics
E. Pathogen Sodoku D. Sulfonamide
881. On the oral mucosa of 20 years old E. Antisera
woman the dentist noticed a round ulcer with 886. Patient appealed to the dentist
a thick bottom and smooth edges, which complained of vesicular rash around the
resembles a chancre. What is the diagnostic mouth. Which method should be applied to
method should be used at this stage of the confirm the diagnosis "herpes"?
disease, to confirm the etiology of syphilis? A. Microscopic examination of smears
A. The biological B. Haemagglutinin
B. Bacteriological C. Seeding on blood agar
C. Smear D. Seeding to Kitt Tarotstsi medium
D. Serological E. Immunofluorescence reaction
E. Allergic 887. In a patient with smear microscopy of
882. In the tested material from a patient with necrotic ulcers in the oral cavity revealed
gingival pockets necrotizing stomatitis spiral organisms, which are stained in blue by
revealed thin spiral bacteria that are 8-14 curls Romanowsky-Giemsa. Specify whether the
and Romanovsky-Giemsa stained a pale pink organism is involved in the development of
color. Which organisms have been identified the necrotic process in this patient?
in the material? A Spirochete Vincent
A. Fusobacterium reriodonticum B. Treponema pallidum
B. Treponema vinsentii C. Obermeyer pallidum
C. Actinomices viscosus D. Leptospira
D. Lactobacillus casei E. Pallidum
E. Bacteroides fragilis 888. A patient with a lesion of oral mucosa
883. For the treatment of odontogenic was diagnosed - necrotizing stomatitis
infections dentist appointed antibiotic therapy. Vincent. What methods of laboratory
What more appropriate antibiotic should be diagnosis should be used to confirm the
used for the prevention of candidosis of the diagnosis?
mouth? A. Biology, allergic
A. Nystatin B. Smear, allergic
B. Kanamycin C. Bacteriological, allergic
C. Levomitsetin D. Bacterioscopy, biological
D. Ristomitsin E. Smear, bacteriological
E. Polymyxin 889. The patient appealed to the doctor (the
884. In a study of preschool children a dentist milkmaid), with a rash on the mucosa of the
found in the mucosa of the mouth coating and mouth in the form of the AFL. The doctor
film of gray-white color. At microscopy found a rash on the skin of the hands around
stained by Gram drug identified the nail plate. What pathogen caused the
microorganisms: Gram-positive, ovoid shape, disease?
thread pseudomycelium. Which A. Cytomegalovirus
microorganisms cause mucosal damage? B. Coxsackie virus
A. C.albicans C. FMDV
B. S.aureus D. Herpesvirus
C. B. anthracis E. Vesicular stomatitis virus
D. Leptotrichia 890. The patient was diagnosed with dental

93
offices - necrotizing Vincent's disease. What
microorganisms are pathogens of this disease?
A. Fuzobacteries and spirochetes
B. Streptococci and staphylococci
C. Actinomycetes
D. Bacteroides
E. Herpes simplex virus
891. During the microscopic examination of
biopsies of the lesion of the mucous
membrane of the mouth sticks were found
arranged in clusters that resemble a pack of
cigars. Ziehl-Neelsen stain in red. What type
of pathogen, the most reliably detected in the
biopsy?
A. A.bovis
B. M.tuberculosis
C. A.israilii
D. M.leprae
E. M. avium
892. Saliva has enzymes that neutralized the
bacteria. Name it.
A. Amylase
B. Phosphatase
C. Lysozyme
D. Karbamoilfosfatsintetaza
E. Glucose-6-phosphate dehydrogenase

94

You might also like